Vous êtes sur la page 1sur 115

S

Universidad La Salle.
Facultad Mexicana de Medicina.
Curso de Extensin Universitaria para la Preparacin del Examen Nacional para
Aspirantes a Residencias Mdicas.

Examen del mdulo I Ginecologa y obstetricia

1.- Femenino de 26 aos G-3, P-1, A-1 con 39 SDG por FUR. Reporta contracciones
uterinas que han sido regulares las ltimas tres horas. Al examen encuentras que las
contracciones son cada tres minutos y duran 50 segundos y son firmes a la palpacin. Tuvo
ruptura de membranas hace una hora y lo demuestras con papel de nitrazina. El examen
digital cervical demuestra una dilatacin de 5 cm, con borramiento del 100% y presentacin
en vrtex en estacin 0. Cul de los siguientes criterios es el ms preciso para decir que
se encuentra en la fase activa del trabajo de parto?
a)
b)
c)
d)

Borramiento cervical ms de 90%


Duracin de las contracciones de ms de 30 seg
Dilatacin cervical mayor de tres centmetros
Ruptura de membranas

FASES DEL TRABAJO DE PARTO


El trabajo de parto se divide en tres fases:
Fase 1 latente
Es llamado as al periodo que sirve para la preparacin uterina del parto, ocurre al final del
embarazo y va hasta el inicio de las contracciones del trabajo de parto. Los aspectos a
destacar en este lapso es el reblandecimiento cervical, el aumento importante en el nmero
de receptores para oxitocina a nivel de las clulas endometriales, un aumento sustancial en
los puentes de unin y el nmero de conexinas a nivel miometrial y por consiguiente una
mayor sensibilidad a los agentes uterotnicos.

Fase 2 activa
Es el lapso que representa el trabajo de parto activo, y se acepta que se inicie cuando
existen 3 cm de dilatacin y las contracciones uterinas son aptas para producir un avance
en el trabajo de parto; se divide en tres periodos:

Primer periodo. Se inicia cuando las contracciones uterinas alcanzan la frecuencia,


intensidad y duracin suficientes para causar borramiento y dilatacin del cuello uterino, y
finaliza
cuando
ste
se
encuentra
en
completa
dilatacin.
El lapso de tiempo que dura es variable, pero se acepta como normal hasta diez horas en
primigrvidas y ocho horas en multigrvidas; pero independientemente de esto, se debe
considerar como adecuado si el borramiento y la dilatacin cervical son progresivos e
ininterrumpidos.
Segundo periodo. Se inicia con una dilatacin cervical completa y termina con la expulsin
del feto; tiene una duracin variable, pero se acepta como normal una hora en pacientes
primparas y 30 minutos en multparas; y tiene como caracterstica que debe de ser
progresivo e ininterrumpido.
Tercer periodo. Este comienza inmediatamente finalizada la expulsin fetal y termina con
la expulsin total de la placenta y las membranas corioamniticas; a este periodo se le
conoce tambin como de alumbramiento y es el ms corto de los periodos del parto; como
norma general se acepta que no debe de extenderse ms all de 10 minutos.
Existen algunos autores que incluyen un cuarto periodo dentro del trabajo de parto, el
cual abarca aproximadamente la hora posterior al alumbramiento, y comprende el lapso de
tiempo cuando ocurre la contraccin y retraccin de las fibras miometriales, as como la
trombosis de los vasos adyacentes, lo cual es un efectivo control de la hemorragia del sitio
de implantacin de la placenta.
Fase
3
Este periodo es el que representa el regreso de la mujer a su estado previo al embarazo, y
se caracteriza por la involucin uterina, la eyeccin lctea y por ltimo la restauracin de la
fertilidad; existen estudios que involucran en esta fase a la endotelina-1 y a la oxitocina
como substancias responsables de estos cambios postparto.

PROGRAMA DE ACTUALIZACION CONTINUA PARA GINECOLOGA Y OBSTETRICIA


PAC GO-1 Libro 3 Obstetricia 2005

2.- Se trata de femenino de 34 aos que inicia tratamiento con sulfato de magnesio por
presentar eclampsia, se presentan
emplear en ste caso es:

a) b) Nitroprusiato.
b) Gluconato clcico
c) Simpaticomimticos.
d) Carbonato sdico.

signos de sobre dosificacin. El antdoto se debe

NIVEL DE PRIMER CONTACTO (ATENCION PRIMARIA)


Se debe instruir a todas las embarazadas que deben acudir inmediatamente a un centro de
salud en cualquiera de los siguientes casos:

edema que se desarrolla rpidamente (en pocos das)


cefalea severa y persistente
dolor en la regin abdominal superior
visin borrosa

Se debe realizar la medicin de la presin arterial y un anlisis de orina para la deteccin


de proteinuria a las mujeres que acudan a centros de salud presentando estos sntomas.
Convulsiones
Si se asiste a una mujer con eclampsia en un centro de atencin primaria,
1. deben mantenerse las vas respiratorias permeables;
2. se debe colocar a la mujer de costado (posicin decbito lateral izquierda) para evitar la
aspiracin del vmito u otras secreciones;
3. si es posible, se debe establecer una va intravenosa;
4. se debe administrar sulfato de magnesio.
Monitoreo de la administracin de sulfato de magnesio: Durante el tratamiento con sulfato
de magnesio, se recomienda realizar un control cada 4 horas, como mnimo, para detectar la
presencia de: Reflejo rotuliano, frecuencia respiratoria superior a 16 por minuto, volumen
de orina >100 ml en las 4 horas previas.
- Sobredosis de sulfato de magnesio: Todo centro de salud que utilice sulfato de magnesio
debe disponer de ampollas de gluconato de calcio (1 g) como antdoto para la sobredosis de
dicho frmaco.

Se sugiere medir la presin arterial y administrar antihipertensivos segn


corresponda.
Convulsiones recurrentes: en caso de convulsiones recurrentes, se administran
otros 2 a 4 g de sulfato de magnesio por va IV en el lapso de 5 minutos, tanto para
el rgimen IM como el IV; la dosis se determina en funcin del peso de la paciente.

El sulfato de magnesio es un frmaco usado en el control de las convulsiones eclmpticas,


para suprimir o controlar las contracciones uterinas sean estas espontneas o inducidas, y
como broncodilatador luego del uso de beta agonistas y agentes anticolinergicos. Tambin
tiene indicacin como terapia de reemplazo en la deficiencia de magnesio, como laxante
para reducir la absorcin de txicos del tracto gastrointestinal. El sulfato de magnesio
est ganando popularidad como tratamiento de inicio en el manejo de algunas arritmias,
particularmente en Torsades de Pointes, y en arritmias secundarias a sobredosis de

antidepresivos tricclicos o toxicidad digitlica. Esta tambin considerado clase Ila


(probable beneficio) para la fibrilacin ventricular refractaria y la taquicardia ventricular,
luego de la administracin de dosis de lidocaina y bretilio.

FARMACODINAMIA
El sulfato de magnesio tiene la capacidad de alterar la excitabilidad de la fibra miometrial,
afecta el acoplamiento excitacin contraccin y el proceso mismo de contraccin, inhibe
la entrada de calcio al sarcoplasma y reduce la frecuencia de los potenciales de accin.
Inhibe tambin la liberacin de acetilcolina. Por ser estas acciones comunes en las fibras
musculares se pueden ver afectadas tambin la musculatura voluntaria e incluso las fibras
miocrdicas.(1)

Bibliografa.
Graves C. Frmacos que contraen o relajan el tero. En: Hardman J, Limbird L, Molinoff P,
Ruddon R, Goodman A, eds. Goodman & Gilman. Las Bases Farmacolgicas de la Teraputica.
9 ed. Mxico DF: McGraw-Hill Interamericana; 1996. pp. 1012-3.

3.- Femenino de 19 aos, atendida en sala de urgencias ginecoobsttricas, Antecedente:


cursa embarazo de 38 SDG. Exploracin Fsica: en trabajo de parto. Repentinamente
presenta sangrado profuso transvaginal y dolor abdominal.
La causa ms probable de la sintomatologa de esta paciente es:

a) Laceracin vaginal por coito


b) Abruptio placentae
c) Cervicitis
d) Placenta previa

DESPRENDIMIENTO PREMATURO DE PLACENTA NORMOINSERTA (DPPNI):


Constituye la separacin de la placenta de su rea de insercin antes del 3 perodo del
parto. La mortalidad fetal es muy alta (superior al 15%) y la materna es tres veces superior
a la esperada.

La aparicin frecuente de SFA, prematuridad, anemia, etc., hace que el nmero de secuelas
tanto sensitivas como motoras sea alto. Desde el punto de vista materno, complicaciones
secundarias a la hemorragia, a las alteraciones de la coagulacin o bien a la embolia
pulmonar tambin tienen una tasa muy alta.
Desde la antigedad se identific al cuadro clnico caracterizado con la trada sintomtica
de hipertona, metrorragia y muerte fetal, con pronstico materno comprometido. En 1775,
Ricci diferenci la separacin prematura de una placenta de insercin normal, de aquella de
insercin baja; a la primera llam hemorragia accidental, y a la segunda hemorragia
imprevisible. Couvelaire introdujo el trmino de desprendimiento prematuro de placenta
y describi la aparicin de la apopleja uterina (tero de Couvelaire)

Obstetricia. Scwarcz, Sala, Duverges. 7 edic. Edit. El Ateneo. (Biblioteca Fac.


Med. UNNE).

4.- Femenino de 23 aos acude al servicio de ginecologa, por referir ciclos opsomenorreicos, desde el inicio de su menarquia, en los ltimos 7 das ha incrementado 15 Kg.
de lo que pesaba habitualmente. Exploracin Fsica: acn facial importante, as como
bigote.
El diagnstico ms probable en esta paciente es:
a)
b)
c)
d)

Sx. De Asherman
Sx. Stein Leventhall
Sx. Amenorrea Galactorrea
Sx. Karman

Sndrome de Ovario Poliqustico (SOP) es uno de los ms comunes trastornos endocrinos


que afectan a las mujeres alrededor del 5% al 10% de las mujeres en edad reproductiva
(12-45 aos) y se piensa que es una de las principales causas de la infertilidad femenina.
Las caractersticas principales son la obesidad, anovulacin (dando lugar a la menstruacin
irregular) o amenorrea, acn, y las cantidades excesivas o los efectos de andrognicos
(masculinizantes) hormonas. Los sntomas y la severidad del sndrome varan mucho entre
las mujeres. Si bien las causas son desconocidas, resistencia a la insulina, la diabetes y la
obesidad estn fuertemente correlacionadas con el SOP.
Bulun SE, Adashi EY. The physiology and pathology of the female reporductive axis.
In: Kronenberg HM, Melmed S, Polonsky KS, Larsen PR, eds. Williams Textbook of
Endocrinology. 11th ed. Philadelphia, Pa: Saunders Elsevier; 2008:chap 16.

5.- Posterior a un trabajo de parto con expulsin normal, y tras una hora aproximada en
periodo de alumbramiento en el que se practic masaje uterino y se increment
moderadamente la dosis de oxitocina, no aprecian signos de desprendimiento placentario,
se indica una extraccin manual de placenta, que resulta imposible por no existir plano de
separacin entre la placenta y la pared uterina. El diagnstico ms probable es:

a) Engatillamiento placentario.
b) Placenta succenturiata con cotiledn aberrante.
c) Placenta circunvalata
d) Placenta adherente por acretismo placentario.

Se denomina a la placenta como acreta cuando sta se implanta en zonas donde la decidua
es deficiente o anormal y por tanto hay una infiltracin del miometrio por vellosidades
coriales; esta infiltracin puede ser focal, parcial o total. A su vez esta condicin se
subdivide en acreta, increta y percreta. La placenta increta y percreta infiltran todo el
espesor de la pared miometrial, en la percreta adems las vellosidades, perforan la serosa
y llegan en algunas ocasiones a infiltrar rganos vecinos, especialmente la vejiga. La PA
est limitada a la superficie miometrial. La frecuencia de presentacin del AP vara entre
10 y 48 por 10.000 partos. (Oishi A 1999, Hung TH 1999, Zaki ZM, 1998).
En las mujeres con acretismo placentario se han visto factores de riesgo, dentro de los
cuales se encuentran:
1.
2.
3.

4.
5.
6.

Edad y multiparidad: La presentacin AP, aumenta con la paridad de la paciente y


la edad, siendo muy rara en primparas.
Placenta previa: esta se ha encontrado en el 30% de los casos de PA. Igualmente
se ha visto PA en el 9.3% de las pacientes con placenta previa (Miller DA, 1997).
Cesrea anterior, o cirugas uterinas previas: Se ha visto este antecedente en el
25% de los casos. En el 29% de los casos la PA estaba implantada en la cicatriz
uterina y solo en el 5% la placenta estaba implantada en otro sitio. (Miller Da,
1997).
Dilatacin y legrado, en el 25% de los casos.
Infeccin uterina previa, remocin manual de la placenta, leiomiomas y otras
anomalas uterinas: La asociacin con estas entidades es inconstante.
Niveles anormalmente elevados de feto-protena y de b-HCG, en el segundo
trimestrre. (Hung TH, 1999).

Una placenta adherente o penetrante no es fcil de diagnosticar antes del alumbramiento.


Despus de ste, se manifiesta como retencin placentaria y sangrado uterino. El
diagnstico generalmente se realiza, despus de intentar la extraccin manual de la
placenta.
Las manifestaciones clnicas propias de la placenta adherente, de la placenta acreta y de la
placenta increta, consisten en una manifiesta dificultad o imposibilidad para la expulsin o
extraccin de la placenta. Como consecuencia de la atona parcial y de la hemostasis
insuficiente en las zonas de despegamiento placentario, se producir una hemorragia ms o

menos grave que en nada se diferenciar de la hemorragia de la atena uterina. Y no ser


solamente al intentar el alumbramiento artificial que se pondran de manifiesto las razones
ntimas de la retencin placentaria; alumbramiento que ser engorroso en. la placenta
ahderente e imposible en las variedades acreta e increta.
En varias ocasiones puede no existir hemorragia y en estos casos la nica manifestacin de
este estado morboso ser la prolongacin del perodo del alumbramiento. La placenta
adherente, como toda placenta retenida, es pronto presa de un proceso infeccioso sin
embargo se han sealado casos de placentas retenidas aspticamente durante muchos
meses. Al cabo de los cuales han sido expulsadas sin causar trastorno alguno; pero hay que
hacer observar que en estas enfermas se ha tratado de retensin de mebranas por abortes
ovulares.

1.
2.
3.
4.
5.
6.
7.
8.
9.

10.

Arredondo-Soberon F, Sabella V, Garza-Leal J, Valente PT. Placenta increta en


primer trimestre de embarazo. Ginecol Obstet Mex 1995; 63: 279-81.
Cantanzarite V, Stanco L, Schrimmer S et al. Managing placenta previa/accreta.
Contemp Obstet Gynecol 1996; 41: 66-95.
Ecker JL, Sorem KA, Soodak L, et al. Placenta Increta Complicating a FirsTrimester Abortion A case report. Journal Reproductive Medicine. 1992; 37-10.
Finberg G, William J. Placenta accreta: prospective sonographic diagnosis in
patients with placenta previa / accreta. Contemp Obstet Gynecol 1996: 41: 66-95.
Gist RS, Voung V, Brody S, Rees P, Landry AD. Placenta increta occurring in a
bligter ovum. South Med J. 1996; 89(5): 545-7.
Harden,MA, Walters MD, Valente PT Postabortal hemorahage due to placenta
increta: A case report. Obstet Gynecol. 1990; 75: 523.
Hudon L, Belfort MA, Broome DR. Dosis and management of placenta percreta: A
review. Obster Gynecol survey 1998; 53: 509-517.
Hung TH, Shau WY, Hsieh CC, et al. Risk factors for placenta accreta. Obstec
Gynecol 1999; 93: 545-50.
Kinoshita T, Ogawa K, Yusumizu T, Kato J. Spontaneous rupture of the uterus due
to placenta percreta at 25-weeks gestation: a case report J Obster Gynaecol Res
1996; 22: 125-8.
Kirkinen P, Helin-Martikainen HL, Vanninen R, Patanen K. Placenta accreta: imaging
by gray-scaleand contrast enhanced color Doppler somography and magnetic
resonance imaging. J Clin Ultrasound 1998; 26: 90-4.

6.-Femenino de 22 aos, que presenta una tumoracin de 2 cm de dimetro en el cuadrante


nfero-externo de la mama izquierda, indolora, de consistencia firme, superficie lisa,
forma ovoidea, mvil y bien delimitada del parnquima vecino, sin antecedentes de
derrame por el pezn, sin piel de naranja ni retraccin del pezn. El diagnstico
presuncional es:

a)
b)
c)
d)

Fibroadenoma.
Carcinoma.
Ectasia de los conductos mamarios.
Quiste solitario.

FIBROADENOMA MAMARIO
Tumor benigno ms frecuente en las mujeres entre los 20 y 35 aos.
ETIOLOGIA
Existen mltiples teoras siendo la ms aceptada la hormonal, generalmente son nicos, solo
el
20% son mltiples o bilaterales. De tamao variable hasta de 10 cm. Ocupa el 13.6% de la
patologa mamaria benigna.
CUADRO CLNICO
Lesin nodular de consistencia dura, de larga evolucin y no dolorosa. Normalmente llegan a
los 3 cm. De dimetro. Durante la fase tarda del ciclo menstrual el tumor suele presentar
un leve aumento de tamao. Durante la menopausia presentan regresin hasta la
calcificacin (signo de palomitas de maz).
DIAGNOSTICO
Es clnico, se presenta como un tumor bien delimitado, desplazable, no adherido a piel ni a
planos profundos, liso o multilobulado en ocasiones. Se localiza frecuentemente en
cuadrantes externos.
EXAMENES DIAGNOSTICOS
ULTRASONIDO MAMARIO .- Identifica un ndulo slido, bien delimitado de bordes
regulares .
TRATAMIENTO.Conservador con vigilancia estrecha dependiendo del tamao y en caso de ser necesario
exresis del ndulo para estudio histopatolgico

hospitalgeneral.salud.gob.mx/
BIBLIOGRAFIA
1. Snchez BC. Tratado de Enfermedades de la glndula mamaria. Ed. Manual Moderno. Cap.
13- 15.
2.- De Vita V. Cancer of the Breast. In Cancer: Principles and Practice of Oncology: Fifth
Ed. Philadelphia: Lippincott-Raven, Chapter 36; pp: 1521-1616.
3.-Consenso Nacional Acerca del Tratamiento de Cncer de Mama. En Tumores de mama:
Diagnstico y Tratamiento. 2 Ed. McGraw-Hill Interamericana; pp: 119-126.
4.-Eberlein T. Current management of carcinoma of the breast. Ann Surgery 1994; 220:
121-136.
5. Encyclopedie Medico. Chirurgicale Praxis Mdica, Editions Techiques de Mexico, tomo 5,
ao 2005.

7.- Mujer que acude por amenorrea de 7 semanas, asintomtica, con antecedentes de
enfermedad plvica inflamatoria tratada anteriormente, sin evidencia mediante ecografa
de tero ocupado. Al determinar la B-HCG, se obtiene cifra de 2,500UI/l. Ante estos
datos, en primer lugar habr que pensar en:
a)
b)
c)
d)

Gestacin de evolucin normal correspondiente a amenorrea.


Gestacin ectpica asintomtica.
Aborto precoz completo con expulsin total de restos ovulares intrauterinos.
Embarazo molar de inicio, sin signos ecogrficos intrauterinos.

FACTORES DE RIESGO
 Anomalas tubarias





Enfermedad plvica inflamatoria 30-50%


Adherencias (endometriosis)
Antecedente de salpingoclasia
Alteraciones anatmicas

EMBARAZO ECTPICO
< 6500mUI

USG normal

Cuantificacin seriada
de HGC

Descenso

Estabilizacin o
incremento

Dilatacin y legrado

Laparoscopia Dx

64

Aumento

Repetir USG
02/07/2013

Diagnstico:




HGC > 1500 U/L


Sin evidencia de saco
Lquido libre en fondo de
saco
 Sensibilidad 63%
 Especificidad 100%

Ankum WM., Mol Bw.,Van der Veen F.,Bossuyt PM.


Risk factors for ectopic pregnancy:a meta-analysis. Fertil Steril 1996:65:1093-9.

8.- Femenino de 45 aos es atendida en consulta externa por presentar desde hace 6
meses flujo transvaginal, mucosanguinolento y sinusorragia importante, refiere dispareunia
de 3 meses de evolucin. AGO. G-5 P-5. Oclusin tubrica bilateral hace 6 aos. E.F.: TA
130/80 MMHG, FC 80 LPM, TEMP. 36.7 C. Laboratorio: HB 9.7 G/DL, HTO 37 %, Se
reporta Papanicolaou clase V.
El siguiente paso para confirmar el diagnstico es:

a)
b)
c)
d)

Histeroscopa.
Ecosonografa.
Colposcopa.
Biopsia dirigida.

El examen citolgico de papanicolau, realizado como examen de tamizaje en ginecologa,


permite establecer las caractersticas de las clulas que se descaman del cuello uterino. La
tcnica de papanicolau ha permitido disminuir la mortalidad e incidencia de cncer invasor
de cuello uterino. Se define examen de papanicolau anormal cuando se presentan
alteraciones en el ncleo, citoplasma y/o la relacin ncleo/citoplasma de las clulas
examinadas. El papanicolau anormal, de acuerdo a la clasificacin de bethesda abarca desde
un diagnstico citolgico de ascus (siglas en ingls de clulas atpicas de significado no
determinado), lesin escamosa intraepitelial (lei) de bajo grado (corresponde a displasia
leve y cambios por papiloma virus), lesin escamosa intraepitelial (lei) de alto grado
(displasia moderada, displasia severa o carcinoma in situ) y clulas de cncer invasor.
4.6. Colposcopa
Ideada por Hinselmann en Alemania en 1924, lleg a Amrica por el cono sur.
Consiste en la visualizacin y amplificacin del cuello uterino mediante un sistema binocular
de lentes, entre 25 y 40 aumentos, lo cual permite la observacin de las estructuras del
cuello uterino mediante la asociacin con imgenes preestablecidas. La colposcopa tiene
una mayor sensibilidad que la citologa, pero su menor especificidad. de conducir a
procedimientos diagnsticos invasivos (biopsias y conizaciones) innecesarios y su mayor
costo, son sus principales limitaciones. Combinadas la citologa y la colposcopia brindan una
seguridad diagnstica que excede EL 95%.
4.6.1. Indicaciones de la colposcopia
La colposcopa est indicada en las siguientes circunstancias (20), (21):
- pacientes con citologa cervical ( papanicolaou) clase III, IV o V, o sus equivalentes en los
otros sistemas de clasificacin.
- pacientes con citologa clase II con atipia inflamatoria, escamosa o endocervical, o cuando
se informe la presencia de coilocitos.
- pacientes con crvix macroscpicamente normal, pero quienes presentan
sinusorragia.
- pacientes con crvix macroscpicamente anormal, en ausencia de carcinoma evidente.
- pacientes con citologa clase II persistente, pese a tratamiento de posibles causas.
(ejemplo: trichomonas).

Tamizaje en cncer ginecolgico autores de la gua.


Dr. Miguel Bueno Montao
Profesor asociado departamento de ginecologa universidad libre gineclogo centro mdico
Imbanaco profesor titular de ginecologa y obstetricia universidad del valle Dr. Jaime
Rubiano universidad del valle Dra. Derry Trujillo

9.- Femenino de 20 aos de edad refiere irregularidades menstruales tipo hipo-opsooligomenorrea desde hace 3 aos. Niega tener vida sexual activa y no recuerda su fecha de
ltima menstruacin. No hay antecedente de galactorrea ni de uso de hormonales exgenos.
Mide 164cm y pesa 60kg. Sin datos de hirsutismo, las mamas, tero y anexos son normales.
El ultrasonido plvico es normal, as como el perfil hormonal. El diagnstico clnico ms
probable es:

a)
b)
c)
d)

Hiperplasia del endometrio


Sangrado uterino disfuncional
Alteracin menstrual fisiolgica
Endometriosis

Hiperplasia endometrial se descarta ya que el usg estpa normal; sangrado uterino


disfuncional no podra ser ya que no hay alteraciones hormonales endgenas ni exgenas
que lo respalden; falla ovrica prematura tampoco por el mismo motivo; endometriosis no
es, ya que no tiene dismenorrea o dolor que es un sntoma muy tpico

Bibliografa: Manual CTO 6 Ed. Ginecologa y Obstetricia, Pgs. 5 y l1.

El ciclo menstrual normal ha cambiado en el curso de los siglos, dependiendo de las


modificaciones en el patrn reproductivo y el estado nutricional de la poblacin.
El debut del sangrado menstrual, definido como menarquia, marca una etapa importante en
la madurez biolgica durante la pubertad.
En el Programa Nacional de Atencin Integral a la Salud de los Adolescentes se mencionan
los trastornos menstruales como una de las primeras causas de consulta o de urgencia en
servicios clnicos de primero y segundo nivel.
Diversas alteraciones menstruales, ya sea las que inducen aumento en la cantidad o
frecuencia o las que se presentan como episodios infrecuentes e irregulares se observan en
la adolescencia y pueden impactar la calidad de vida de las pacientes, con repercusiones
significativas en el mbito reproductivo y metablico.
Por esas razones podemos afirmar que los trastornos menstruales constituyen un problema
de salud pblica y requieren un enfoque ms integral en aspectos preventivos y curativos en
la etapa de la adolescencia.

Aproximadamente la mitad de todas las adolescentes tienen perodos irregulares durante


el primer ao despus de la menarquia. Estos periodos irregulares pueden persistir hasta
cinco aos despus de la menarquia en 20 % de estas adolescentes. La principal
complicacin es la anemia, que puede ser severa y raramente tiene consecuencias fatales.
PATRN MENSTRUAL NORMAL

La Federacin Internacional de Ginecologa y Obstetricia considera el patrn menstrual


normal entre tres y cuatro das, aunque flucta entre dos y siete das. El intervalo entre
menstruaciones es de veintiocho das, considerndose como lmites de veintiuno a treinta y
cinco das.
El volumen de sangre menstrual es de sesenta a ochenta mililitros; no obstante, se
consideran normales los rangos de sangrado entre cincuenta y ciento cincuenta mililitros. El
aspecto de la sangre es rojo oscuro incoagulable.
En la prctica mdica se utiliza un grupo de trminos para hacer referencia a las diversas
alteraciones del ciclo menstrual, que requieren precisin por la frecuencia en que son
diagnosticados.
Segn Schiavon (2000), las alteraciones menstruales ms frecuentes son:
oligoamenorrea: episodios de sangrado infrecuentes, irregulares, con intervalo de ms de
cuarenta das;

polimenorrea: episodios frecuentes pero regulares de sangrado uterino, que ocurren a


intervalos menores de veintin das;

menorragia: sangrado excesivo, tanto en cantidad como en duracin, que ocurre con
regularidad y es sinnimo de hipermenorrea;

metrorragia: sangrado generalmente no excesivo, que ocurre a intervalos irregulares;


menometrorragia: sangramiento generalmente excesivo y prolongado, que ocurre a
intervalos frecuentes e irregulares;

hipomenorrea: sangrado uterino regular, pero disminuido en cantidad;


sangrado intermenstrual: sangrado uterino generalmente no excesivo, que ocurre entre
perodos menstruales regulares.
Las irregularidades menstruales son causa frecuente de consulta en las adolescentes,
siendo 95 % de las veces de naturaleza disfuncional, por inmadurez del eje hipotlamo
hipofisoovrico (HHO).
Uno de los primeros problemas que hay que plantear ante estas irregularidades
menstruales, es la hemorragia uterina disfuncional (HUD). Su definicin guarda relacin con
las caractersticas en cantidad y frecuencia que difieren del sangrado menstrual normal.

Con mayor frecuencia se encuentra en forma de sangrados excesivos y prolongados,


asociados a ciclos anaovulatorios, en ausencia de una patologa o enfermedad existente,
aunque raramente la HUD puede presentarse con ciclos ovulatorios.
De forma prctica, consideramos una hemorragia uterina (HU) como anormal cuando el
sangrado es excesivo, con cualquier desvo o alteracin de su duracin, cantidad o intervalo.
El diagnstico de HUD supone una alteracin de origen endocrino (eje HHO); por lo tanto,
su diagnstico impone haber descartado cualquier patologa orgnica y sistmica que
produzca hemorragia genital. Es un diagnstico por exclusin.
ETIOPATOGENIA DE LA HUD EN LA ADOLESCENCIA
Los ciclos anovulatorios son ms frecuentes en las adolescentes por la inmadurez del eje
HHO en el primer ao tras la menarquia.
En esos casos de HUD, como ya se mencion, se producen ciclos anovulatorios que se
traducen en una proliferacin desorganizada del endometrio por falta de efecto
progestagnico. Una vez que el endometrio alcanza un grosor crtico, comienza a
descamarse en forma irregular, traducindose en un sangrado permanente de cuanta
variable.
Las manifestaciones clnicas de la HUD son:

Fases de amenorrea de dos a cuatro meses, seguidas de salida de sangre abundante


durante tres o cuatro semanas; en oportunidades existe irregularidad completa en el
sangrado;
sangrado de ms de seis compresas (bien empapadas) al da;
presencia de cogulos;
suele ser indolora;
menstruaciones de ms de siete das de duracin;
ciclos de menos de veintin das.
La gravedad de esta hemorragia se clasifica, de acuerdo con el grado de anemia que
produzca, en metrorragia leve, moderada o grave:

Leve:
metrorragia leve y prolongada,
ciclo menstrual acortado,
hemoglobina y hematocrito normales.
Moderada:
metrorragia copiosa prolongada,
ciclo menstrual acortado,
anemia leve (cifras de hemoglobina inferior a diez gramos por litro).
Grave:
metrorragia copiosa prolongada,
ciclo acortado e irregular,
anemia grave (cifras de hemoglobina de ocho gramos por litro o menos).

Ante un sangramiento uterino en estas edades se debe realizar el diagnstico diferencial


con:
a) gestacin y problemas relacionados con sta, tales como abortos y gravidez ectpica;
b) coagulopata: 20 % de las adolescentes con hemorragia uterina tienen un defecto de la
coagulacin. La manifestacin ms precoz de alteraciones de la coagulacin sangunea puede
ser evidenciada por un sangramiento genital anormal, lo cual puede estar relacionado con
deficiencias de plaquetas, leucemias, prpuras, enfermedad de Von Willebrand, deficiencia
de protrombina u otros factores de la coagulacin;
c) malformaciones del aparato genital, traumatismosgenitales, presencia de cuerpos
extraos;
d) dispositivos intrauterinos;
e) tumores uterinos, sarcoma botroides o tumores anexiales;
f) hipo o hipertiroidismo;
g) insuficiencia renal o heptica.
Como el diagnstico de HUD es de exclusin, hay que hacer una historia clnica minuciosa,
exmenes complementarios y sin falta descartar las otras causas de sangramiento
transvaginal.
Se debe precisar con detalle el nivel de desarrollo puberal, la actividad sexual y la
presencia de situaciones concomitantes como: a) contacto sexual sin proteccin
contraceptiva;
b) uso irregular de anticonceptivos orales o antecedentes de insercin de dispositivos
intrauterinos;
c) ejercicios fsicos extenuantes;
d) historia previa de sangrado excesivo, asociado a extracciones dentarias, pequeas
heridas, epistaxis y otras;
e) dolencias renales u hepticas preexistentes.
Ante cualquier demanda de atencin por adolescentes con sangramiento genital con las
caractersticas descritas, se requiere de un examen fsico general que incluya exploracin
general completa, toma de tensin arterial y pulso, bsqueda de exoftalmia, fascie Cushing,
visceromegalias o presencia de masas abdominales palpables, edemas parpebrales y de
miembros superiores, as como puntos hemorrgicos en epidermis y otros signos de
coagulopata.
El examen ginecolgico debe realizarse en todas las adolescentes, con excepcin de las que
no han tenido actividad sexual y presentan sangramiento leve. Adems del examen de sus
genitales, hay que efectuar una valoracin citolgica y microbiolgica en particular en
quienes presenten manifestaciones clnicas.
En la inspeccin de los genitales durante el examen de la paciente, es importante evaluar
que el sangramiento se origine en lesiones ubicadas en los genitales externos, uretra u
hemorroides, as como indagar acerca de la posibilidad de abuso sexual. Si despus del
tratamiento de la HUD leve contina el sangrado, se recomienda la realizacin de ecografa
abdominal para precisar el diagnstico.
En pacientes que ya han tenido relaciones sexuales se debe buscar si el tero tiene
caractersticas gravdicas, la posibilidad de un aborto en curso, as como la presencia de
una masa anexial que permita corroborar la existencia de embarazo o alguna neoplasia

benigna o maligna. El examen con espculo podr demostrar un cuello hipermico, sangrante
o gravdico; de encontrarse estos hallazgos, se descartara la etiologa disfuncional del
sangrado.
Exmenes complementarios bsicos que no pueden faltar:
hemograma completo,
coagulograma completo,
orina,
ultrasonido ginecolgico abdominal, transvaginal o transrectal segn proceda,
ecografa abdominal. Si fuese necesario por los signos identificados en el examen de la
paciente, se deben realizar:
dosificacin de FSH, LH, T3, T4, TSH y prolactina si hay sospechas clnicas de otras
enfermedades endocrinas concomitantes,
laparoscopia en casos seleccionados por patologa de base,
otros, segn hallazgos de la historia clnica y la exploracin.

10.- Recibe usted los resultados histopatolgicos de una paciente de 24 aos de edad que
acudi a revisin rutinaria, los resultados reportan imagen histolgica de coilocitos lo cual
sugiere infeccin por:

a)
b)
c)
d)

Herpes virus tipo 2


Citomegalovirus
Virus del papiloma humano
Vaginosis bacteriana.

El coilocito es un tipo de clula hallada en lesiones precancerosas cervicales. Tambin es


comn apreciarla microscpicamente en lesiones reaccionales en la mucosa oral, debido a su
similitud con la mucosa vaginal, en enfermedades como Papilomas, o en Condiloma
acuminado.
El coilocito es la manifestacin clsica de la infeccin por VPH en la clula. Fue descrito por
primera vez por Koss y Durfee en 1956. Esta clula tambin ha sido llamada clula en
baln.
El coilocito es una clula epitelial escamosa, ms comnmente superficial e intermedia,
aunque tambin puede verse en clulas parabasales y metaplsicas. Esta clula presenta
cambios tpicos tanto en su ncleo como en su citoplasma, pierde los bordes angulados
usuales de la clula escamosa superficial y su forma tiende a ser redondeada y ovoide. El
citoplasma muestra una condensacin perifrica que le da un aspecto en asa de alambre,
es opaco, denso y de aspecto creo, anfoflico, acidoflico o de color rojo/naranja brillante.
Adems se observa una gran cavidad o halo con un margen muy bien definido, de forma oval
o ligeramente festoneado. El ncleo de la clula se localiza de manera excntrica, lo que lo

convierte en un halo paranuclear, no perinuclear. Ocasionalmente puede encontrarse


material fagocitado dentro del espacio coiloctico.

Lesin Intraepitelial de Bajo Grado. Alteraciones Celulares compatibles con Infeccin


(Coilocitos).
Schlecht, N.F., Kulaga, S., Robitaille, J., Ferreira, S., Santos, M., Miyamura, R.A.,
Duarte-Franco, E., Rohan, T.E., Ferenczy, A., Villa, L.L., & Franco, E.L. (2002) Persistent
Human Papillomavirus Infection as a Predictor of Cervical Intraepithelial Neoplasia.
JAMA, 286, 3106-3114

11.- Se trata de paciente de 34 aos que cursa con 39 SDG; a la exploracin fsica
reflejos patelares hiperactivos, inquieta, se reportan cifras de TA 145/95, se realiza
laboratorio que reporta proteinuria 2+,. El diagnstico ms probable es:

a) Glomerulonefritis aguda
b) Hipertensin esencial
c) feocromocitoma
d) Preeclampsia

La hipertensin es la complicacin mdica ms comn del embarazo , aunque para algunos


autores es la segunda complicacin mdica del embarazo slo despus de la anemia; es ms
frecuente en jvenes durante el primer embarazo y en nulparas de mayor edad,
hipertensas previas y diabticas.
En Mxico, tambin es la complicacin ms frecuente del embarazo, la incidencia es de 47.3
por cada 1 000 nacimientos y es adems, la primera causa de ingreso de pacientes
embarazadas a las unidades de terapia intensiva (debido a hemorragia masiva, para recibir
soporte hemodinmico), segn la secretara de salud (2001) la mortalidad por
complicaciones del embarazo ocupa el 15 lugar en la mortalidad hospitalaria en general.

Adems, la tasa de preeclampsia se ha incrementado 40% en el periodo entre 1990 y 1999 y


constituye hasta 40% de los partos prematuros iatrognicos.
.

Preeclampsia
La preeclampsia es un sndrome clnico caracterizado por hipertensin con disfuncin
orgnica mltiple, proteinuria, edemas.
Es definida como un incremento de al menos 140/90 mmHg despus de la semana 20 de
gestacin, un incremento en la presin sangunea diastlica de al menos 15 mmHg respecto a
un nivel previo a la semana 20 combinado con proteinuria (> 300 mg en 24 horas). Las
mediciones de la presin arterial citadas deben ser medidas al menos 2 ocasiones con por lo
menos 6 horas de separacin. La proteinuria puede ser una toma simple de orina al azar que
indique al menos 30 mg/dL 3 ++ en dos muestras de orina1 segn el tipo de prueba. El
criterio del incremento de 30 mmHg en la presin sistlica y/o 15 mmHg en la presin
diastlica respecto a valores previos a la semana 20 de gestacin ha sido eliminado por ser
poco especfico15

1.-Myers JE, Baker PN. Hupertensive diseases and eclampsia. Curr Opin Obstet Gynecol
2002; 14: 119-125.
2. Tierney, McPhee, Papadakis. Diagnstico clnico y tratamiento 2003. 38 ed, Mxico,
Manual Moderno, 2003: 770-773.

3. Wilson MI, Goodwin TM, Pan VI, Ingles SA. Molecular epidemiology of preeclampsia.
Obstet and Gynecol Survey 2003; 58(1):39-66.
4. Burrow GM. Complicaciones mdicas durante el embarazo. 4 ed, Mxico, McGraw-Hill
panamericana: 1996: 1-25.
5. Guyton AC, Hall JE. Embarazo y lactancia en: Tratado de fisiologa mdica, 10 ed,
Mxico, McGraw-Hill Interamericana 2001: 1135-45.
6. Vaticon D. Fisiologa de la fecundacin, embarazo, parto y lactancia, en: Tresguerres
JAF. Fisiologa Humana. Mxico, Interamericana McGraw-Hill, 1992: 1086-1109.
7. Pridjian G, Puschett JB. Preeclampisa. Part 1: Clinical and
Considerations. Obstet and Gynecol Survey 2002; 57 (9): 598-618.

Pathophysiologic

8. Pridjian G, Puschett JB. Preeclampisa. Part I1: Experimental and Genetic Considerations.
Obstet and Gynecol Survey 2002; 57 (9): 619-40.
9. IMSS. Embarazo de alto riesgo. Gua diagnstica teraputica. Rev Med IMSS 1998;
36(1):45-60

12.- Mujer con diagnstico de amenorrea que acude con resultados de laboratorio los que
reportan los siguientes niveles hormonales: GnRH elevada, FSH y LH elevadas, hormonas
ovricas (estrgenos y progesterona) bajas. El defecto est a nivel de:
a)
b)
c)
d)

Hipotlamo.
Hipfisis.
Ovario.
Endometrio.

Evaluacin de la paciente con amenorrea secundaria


La mayora de las pacientes con AS que no estn embarazadas o no entraron en el
climaterio tienen una alteracin en algn nivel de la cascada reguladora del ciclo menstrual
femenino. A continuacin, esquematizamos la evaluacin de la AS en pasos. Cada mdico
deber adaptar los tiempos de la evaluacin a la situacin particular de cada paciente. No
obstante, recomendamos no saltear ninguno de estos pasos ya que un diagnstico preciso
permitir realizar un tratamiento racional y dar un pronstico respecto de la futura
funcin menstrual y de la fertilidad.
Primer paso (descartar el embarazo)

Su prueba se basa en el dosaje cuali o cuantitativo de la subunidad beta de la


gonadotrofina corinica humana. Si se certifica la ausencia de embarazo podr avanzarse al
siguiente paso, pero teniendo en cuenta siempre que si la probabilidad de embarazo es muy

alta se deber repetir la prueba nuevamente. Se recomienda no obviar esta prueba aunque
la paciente asegure que no ha mantenido relaciones sexuales.
Segundo paso (prueba de progesterona)

Debe realizarse slo si el test de embarazo es negativo y consiste en dar


medroxiprogesterona entre 30 a 50mg por va oral (un comprimido de 10mg durante 5 das)
o progesterona oleosa 100 a 200mg por va intramuscular en una sola dosis. El resultado de
esta prueba provee informacin acerca de si se produjeron estrgenos. La prueba se
considera positiva si se produce un sangrado luego de 2 a 14 das de la suspensin de la
progesterona. La respuesta positiva indica que existe integridad anatmica del aparato
genital femenino y que el ovario produce estrgenos. En estos casos, la AS se debe a que no
ha habido ovulacin. La causa ms frecuente de AS con prueba de progesterona positiva es
la disfuncin hipotalmica leve. En este caso, no se dispara el pico de LH necesario para
producir la ovulacin, no se ovula (ciclo anovulatorio), no hay cuerpo lteo y no hay
produccin de progesterona. El sistema reproductor queda en un estado folicular, con
grados variables de desarrollo folicular, acompaados de concentraciones cambiantes de
estradiol. Puede producirse hemorragia en ausencia de ovulacin dado que el endometrio
proliferativo sufrir ocasionalmente un desprendimiento parcial en respuesta a estos
niveles cambiantes de estradiol. Estos sangrados luego de un ciclo anovulatorio se
denominan sangrados uterinos disfuncionales (SUD). Como dijimos, ejemplos frecuentes de
situaciones de anovulacin son el estrs, la prdida brusca de peso y el ejercicio intenso.
Otras causas menos frecuentes son la hiperprolactinemia, el hipotiroidismo y el sndrome
del ovario poliqustico. Otras veces no hay una causa clara (idioptica) de este trastorno.
La prueba se considera negativa si NO se produce un sangrado luego de 2 a 14 das de la
suspensin de la progesterona, lo que ocurre cuando los niveles de estrgenos son bajos
(menores a 40 pg/ml). La causa ms frecuente de prueba negativa es la insuficiencia
gonadal de la menopausia.
Tercer paso

Para avanzar al tercer paso es preciso distinguir si la prueba de progesterona fue positiva
o negativa.

Pacientes con prueba de progesterona positiva

Una prueba de progesterona positiva hace diagnstico de ciclos anovulatorios. Segn los
antecedentes, el examen clnico y la probabilidad previa el mdico podr orientarse hacia
cul es el diagnstico. Hay situaciones de estrs, crisis vitales, cambios de peso brusco o
ejercicio extremo que ocasionan un trastorno del ciclo aislado. En estos casos, la prueba de
progesterona es diagnstica y teraputica. Es decir, la paciente no requiere estudios
posteriores. Es conveniente reasegurarla y explicarle que su periodo se normalizar una vez
solucionado el problema. En una paciente con trastornos anovulatorios frecuentes, que
presenta obesidad, acn e hirsutismo, existe alta sospecha de sndrome de ovario
poliqustico (SOP). En este caso, debemos solicitar una ecografa pelviana y un dosaje de

LH/FSH o de hormonas masculinas: testosterona libre (To), dehidroepiandrosterona


sulfato (DHEA-S) y 17OH progesterona.
Si la paciente tiene trastornos del ciclo y galactorrea, se debe solicitar un dosaje de
prolactina (PRL). El valor normal de PRL para la mujer no embarazada es de 20 a 25ng/ml.
Cuando el valor de PRL es menor de 100ng/ml, generalmente es de causa idioptica o
farmacolgica; si est entre 100 y 200ng/ml, puede ser por causa farmacolgica o por
enfermedad hipotalmica; si es mayor de 200ng/ml, la causa ms probable es el adenoma
hipofisario. Cuando el valor de la PRL es mayor de 100ng/ml, se debe solicitar un estudio
por imgenes que puede ser una tomografa computada de cerebro con contraste o una
resonancia magntica nuclear con gadolinio. Si se observa una imagen menor de 10mm, se
trata de un microadenoma de hipfisis y si es mayor de 10mm, se hace diagnstico de
macroadenoma. Si la paciente consulta por trastornos del ciclo frecuentes y no presenta
galactorrea o signos o sntomas de hipotiroidismo, igualmente deber solicitarse un dosaje
de tirotrofina (TSH) y de prolactina (PRL) sricas.

Pacientes con prueba de progesterona negativa

Deber dosarse el nivel srico de FSH cuyo valor refleja los niveles de estrgenos
circulantes (por el fenmeno de retroalimentacin negativa). Si la FSH es mayor de 40
UI/ml, los niveles de estrgenos son bajos. En este caso, se asume que existe una
insuficiencia ovrica. El valor de la FSH puede hacer el diagnstico diferencial entre la
falla ovrica (castracin temprana, menopausia precoz o menopausia normal) y la disfuncin
hipotlamo-hipofisaria severa. En la primera, la ausencia de retroalimentacin negativa
estimula la liberacin de las gonadotrofinas y, en consecuencia, stas se encuentran
elevadas en la sangre. En la segunda, los valores de gonadotrofinas son bajos debido a un
defecto en su produccin. El dosaje de FSH es ms sensible que el de LH para este
propsito, por lo tanto, si la prueba de progesterona es negativa y la FSH es baja, lo ms
probable es que la causa sea una disfuncin hipotlamo-hipofisaria severa. En este caso
debera solicitarse un estudio de diagnstico por imagen del cerebro (RMN con gadolinio o
TAC con contraste). En el cuadro 4 se muestran los valores normales de las hormonas
mencionadas.
Cuadro 4: valores hormonales normales
PRL (prolactina): 20 a 25ng/ml en mujeres no embarazadas.
TSH: de 0.5 a 5mUI/ml.
FSH: 5 a 30mUI/ml.
LH: 5 a 20mUI/ml (en el pico ovulatorio este valor se debe multiplicar por dos o tres).
Falla ovrica: FSH mayor de 40mUI/ml.
Disfuncin hipotlamo hipofisaria: FSH menor a 5mUI/ml y LH menor a 5mUI/ml.
En general, una paciente con disfuncin hipotalmica con gonadotrofinas bajas est
expresando una gravedad mayor que la disfuncin hipotalmica leve con prueba de
progesterona
positiva.
En los casos en los que la prueba de progesterona es negativa, la FSH es normal, la paciente
no est en una edad cercana a la menopausia y existe alta sospecha de que exista una
enfermedad uterina, se recomienda realizar una prueba de estrgenos y progesterona.
Esta prueba sirve para conocer si el efector (el tero) responde a los estmulos
hormonales. Consiste en administrar estrgenos (1.25mg por da durante 20 das) ms

progesterona (10mg por da durante los ltimos 5 das en que se administran los
estrgenos). Si no hay sangrado, se debe repetir la prueba y, si nuevamente no hay
sangrado, la prueba se considera negativa. Esto indica que hay una falla en el efector, o
sea, en el tero.

En este caso la paciente debe ser derivada al gineclogo. Las causas ms probables de falla
uterina son la endometritis por abortos o partos spticos y las sinequias por curetajes
vigorosos. Cuando falla el efector, la ecografa transvaginal puede ser normal. Por eso debe
realizarse una prueba de estrgenos y progesterona para evaluar directamente la
funcionalidad del tero. La prueba se considera positiva si hay sangrado (aunque sean slo
gotitas). Esto significa que el tero est indemne para responder a estmulos hormonales y
el problema es de origen hipotlamo- hipofisario.
1.

Jonathan R, Pletcher, Gail B. Slap. Menstrual Disorders Amenorrhea. Pediatric


Clinics of North America. June 1999; 46: Issue 3.
2. Bryan McIver, Susan A Romanski; Todd B Nippoldt. Evaluation and Management of
Amenorrhea. Mayo Clinic Proceedings Dec 1997; 72: 1161-1169.
3. Tarannun Master Hunter, Diana L.Heiman. Amenorrhea: Evaluation and Treatment.
Am Fam Physician 2006; 73: 1374-1382.
The Practice Committee of the American Society for Reproductive Medicine. Current
Evaluation of Amenorrhea. Fertility and Sterility Nov 2006: 86 Supl 4: 148-155.

13.- Paciente de 25 aos, Gesta 1, Para 1. Con dos citologas lesin de alto grado, prueba
de Schiller positiva y biopsia de crvix que demuestra carcinoma In Situ. La conducta es:
a)
b)
c)
d)

Conizacin.
Histerectoma total abdominal.
Histerectoma y salpingooforectoma bilateral.
Electrocauterizacin del crvix.

La conizacin cervical es el tratamiento de eleccin en pacientes con cncer cervicouterino


microinvasor y ms si existe deseo de fertilidad. Asimismo, la histerectoma extrafasciales
un mtodo adecuado en lesiones de 0.5 a 3 mm de invasin. Adems se propone que, para
pacientes con lesiones de 3.1 a 5 mm de invasin, a partir de la membrana inicial sin
factores de mal pronstico como invasin vascular y linftica, sean tratadas con
histerectoma extrafacial, ya que en aquellas a las que se realiz linfadenectoma plvica,
con este tipo de lesin, no se encontr metstasis a ganglios linfticos.

Resultados del tratamiento en cncer cervicouterino microinvasor en el Instituto Nacional


de Cancerologa de Mxico (1980-1999)
1.- Mestwerdt G. Fruhdiagnose des Kollumkarzinoms. Zentralb Gynaekol, 1947 ;69 :326.

2. - Morrow CP, Curtin JP. Surgery for cervical neoplasia. In Gynecologic Cancer Surgery.
New York, Churchill Livingstone, 1996, p 472.3.
3. - Burghardt E, Holzer E. Diagnosis and treatment of microinvasive carcinoma of the
cervix uteri. J Obstet and Gynecol 1977; 49:641-653.
4.- Sedlis A, Sall S, Tsukada Y, et al. Microinvasive carcinoma of the uterine cervix: a
clinical-pathologic study. Am J. Obstet Gynecol. 1979;133:64.

14.- Femenino de 33 aos de edad, que cursa con 8 SDG, portadora de Diabetes MellitusInsulino Dependiente de 10 aos de evolucin, entre las siguientes, la afirmacin correcta
es:
a) El riesgo de aborto espontneo es elevado, independientemente del grado de control de
la glucemia.
b) Se debe hacer una determinacin de Hb glucosilada en la primera visita obsttrica
(Hb A1).
c) Se debe hacer amniocentesis en el segundo trimestre para determinar cariotipo y
alfafetoprotena.
d) En el segundo trimestre habr que evaluar conjuntamente la alfafetoprotena en suero
materno, beta-HCG y el estriol no conjugado.

En los ltimos decenios se ha visto un cambio notable en el pronstico del embarazo cuando
ste se asocia a diabetes.
Con la introduccin de la insulina en la prctica mdica, hace poco ms de 60 aos, la
mortalidad materna se redujo en forma rpida y espectacular, en cambio la
morbimortalidad neonatal ha tenido una disminucin lenta y gradual, logrando ser reducida
a cifras inferiores al 5%.
Gran parte de la mortalidad actual del hijo de madre diabtica, se relaciona con
malformaciones congnitas en cuya prevencin se concentra la investigacin actual.
El embarazo supone una pesada carga para las diabticas, debido a los efectos
diabetognicos que posee en s ste estado.
stas pacientes tienen una tendencia a la inestabilidad metablica y precisan
monitorizacin frecuente, tratamiento estricto y un ritmo de vida muy controlado. (3)
En las pacientes diabticas que ya tienen lesin orgnica, el embarazo puede acelerar el
proceso, por lo que precisan un control y un tratamiento intensivo.(3)
Debido a lo anterior es que lo ideal es que reciba asesoramiento antes de que quede
embarazada.
Importancia de controlar la Glicemia: El control inadecuado de la glicemia suele aumentar la
incidencia de abortos en el primer trimestre, anomalas congnitas fetales, macrosoma
fetal, polihidroamnios y fetos nacidos muertos. (1)

El perodo de mayor riesgo de malformaciones fetales es el perodo periconcepcional y el


de la organognesis, ste, riesgo se puede reducir mediante un adecuado control
metablico.(3)
A su vez, la administracin de cido Flico antes del embarazo y durante el primer
trimestre, en dosis de 400 ug/da, previene la aparicin de defectos del Tubo /Neural
Neur
Importancia del control fetal durante el embarazo : Debido a la posibilidad de Embriopata
Diabtica, es necesario efectuar una determinacin de Hemoglobina Glicosilada en etapas
precoces del embarazo, Ecografa Obsttrica a la semana 20 y un Ecocardiograma Fetal a
la semana 24 para descartar malformaciones fetales ms habituales.(3)

La HbA1c, tambin conocida como hemoglobina glicosilada o glicada, glucohemoglobina


o HbA1, es un trmino utilizado para describir una serie de componentes estables
minoritarios de la hemoglobina que se forman lentamente y sin intervencin enzimtica,
a partir de la hemoglobina y la glucosa. La velocidad de formacin de la HbA1c es
directamente proporcional a la concentracin ambiente de glucosa. Como los eritrocitos
son fcilmente permeables a la glucosa, el nivel de la HbA1c en una muestra de
sangre facilita la historia glucmica de los 120 das anteriores, duracin media de la
vida de estas clulas. En particular, la HbA1c refleja de una forma bastante exacta
la glucemia en los 2-3 meses anteriores al anlisis.

Bibliografa:
1. Cunningham F, MacDonald P, Gant tN. N. Diabetes. Williams Obstetricia, Editorial
Mdica Panamericana,20 0a ed ed.1998;52:1119 .1998;1119-1136.
2. Oyarzn E.Patologa Mdica del Embarazo. Alto Riesgo Obsttrico.Impresos
Universitarios S.A, 2 ed ed.1997;177 .190.
3. Arias F.Gua Prctica para el Embarazo y el Parto de Alto Riesgo. Mosby Doyma
Libros.Segunda Edicin.1995;15:284 -303.
4. Catalano P, Feudtner C, Rosenn B.Diabetes y Embarazo. Clinicas Obstetricas y
Ginecologicas , McGraw Hill Interamericana,Primera Edicin.Volumen 1 /2000;1 1-14.-Guas
Perinatales ao 2002.Diabetes y Embarazo.Ministerio de Salud Chile; Pginas:22 39.
6.Garner P. Type I diabetes mellitus and pregnancy .The Lancet 1995;346:157 157-161.
7. Barrett J., Salyer S., Boehm F. The nonsstress test: An evaluation of 1000 patients Am
J Obstet Gynecol 1981;141:153 -157.
8. Dicker D., Feldberg D., Yeshaya A., Peleg D., Karp M., Goldman J. Fetal surveillance in
insulin dependent diabetic pregnancy :Predictive value of the biophysical profile .Am
Obstet Gynecol 1988;159:800 8804.

15.- Se trata de femenino de 33 aos con antecedentes patolgicos de hipertensin


arterial crnica bien controlada tratada con IECAS, actualmente cursa con 7 semanas de
gestacin , signos vitales dentro del parmetro normal y exmenes de laboratorio sin
alteraciones, se refiere asintomtica , La conducta a seguir es:

a) Suspender los IECAs dado el riesgo que presentan para el feto.


b) Mantener el tratamiento y asociar alfametildopa para disminuir los riesgos fetales de
los IECAs
c) Mantener el tratamiento y asociar hidralacina para disminuir los riesgos maternos de los
IECAs.
d) Mantener el tratamiento dado el buen control tensional.

El uso de IECA y ARAII durante el segundo y tercer trimestre de embarazo est


contraindicado, debido a que estos medicamentos inducen toxicidad fetal (descenso de la
funcin renal, oligohidramnios, retraso en la osificacin del crneo) y toxicidad neonatal
(insufi ciencia renal, hipotensin, hiperpotasemia).
En cuanto a su uso durante el primer trimestre de embarazo, un estudio publicado en el
ao 20061 mostraba un incremento de la incidencia de malformaciones congnitas, en
particular malformaciones cardiacas, en nios nacidos de madres expuestas a IECA
durante el primer trimestre de embarazo en comparacin con las mujeres que no
recibieron tratamiento antihipertensivo o que recibieron tratamiento con otros
medicamentos antihipertensivos. Estudios posteriores realizados no han confirmado a da
de hoy los resultados de este estudio. En lo referente a los ARAII, no se dispone de
estudios epidemiolgicos analticos apropiados, por lo que no se puede descartar que exista
el mismo riesgo que para los IECA.
A pesar de estas incertidumbres, el Comit de Medicamentos de Uso Humano (CHMP) de la
Agencia
Europea de Medicamentos (EMEA) ha recomendado prudencialmente evitar el uso de IECA

y ARAII durante el primer trimestre del embarazo.


Cooper WO et al. Major congenital malformations after fi rst-trimester exposure to ACE
inhibitors. N Engl J Med 2006; 354 (23): 243- 51.
(ref.: 2008/10, junio.

16.- Which of the following drugs combination would be most appropriate in the patient
treatment of acute pelvic inflammatory disease?

a)
b)
c)
d)

Ampicillin / cefoxitin
Tetracycline / gentamicin
Cefoxitin / Doxycycline
Ampicillin / Amikacin

E.P.I.
Regmenes de tratamiento CDC 2002

Parenteral.
Cefotetan 2gr iv/12h Cefoxitina 2gr iv/6h + Doxiciclina 100 mgs iv/8h.
Clindamicina 900 mg iv/8h + Gentamicina iv/im (2mg/kg de carga, luego 1.5
mg/kg/8h.
Alternativas.
Ofloxacina 400 mg iv/12h Levofloxacino 500 mg iv diario con o sin
Metronidazol 500 mg iv/12 h.
Oral.

Ofloxacina 400 mg vo diario x 14 das Levofloxacina 500 mg vo diario x 14


das con o sin Metronidazol 500 mg vo diario por 14 das.
Ceftriaxona 250 mg im x 1 dosis Cefoxitina 2 gr im x 1 dosis y Probenecid
1 gr vo x 1 dosis u otra cefalosporina de 3 gen im + Doxiciclina 100 mgs
vo diario or 14 das con o sin Metronidazol 500 mgs vo diario x 14 das.

17.- An 18-year-old woman complains of myalgias, a sore throat, and painful mouth sores
for 3 daysduration. Her temperature is 38.2 C (100.8 F), blood pressure is 110/80 mm Hg,
pulse is 84/min, respirations are 15/min. Her gingival are edematous and erythematous,
and there are vesicles on her right upper and lower lips. Her pharynx is mildly
erythematous but without exudates, and there is tender mobile cervical lymphadenopathy.
Her breath is not fetid, and the dentition is normal. Which of the following is the most
likely causal agent?

a) Actinomyces israelii
b) Herpes simplex virus 1
c) Nocardia asteroids
d) Streptococcus pyogenes

Las infecciones por herpes simplex son comunes en la prctica diaria, y con frecuencia el
paciente acude a los servicios de urgencia. Estas infecciones son ocasionalmente
recurrentes, generalmente dolorosas y asociadas con sntomas sistmicos, por lo cual el
mdico de urgencias debe estar familiarizado con el cuadro clnico y su manejo.
Existen dos variedades de virus del Herpes simplex (VHS) capaces de causar infeccin en
el hombre: el tipo 1 (VHS-1) y el tipo 2 (VHS-2) que se distinguen entre s por varias
caractersticas, incluyendo sus comportamientos clnico y epidemiolgico, antigenicidad,
composicin del ADN y la sensibilidad a diferentes agentes fsicos y qumicos (Cuadro No.1)
Cuadro No. 1
DIFERENCIAS ENTRE LOS VIRUS HERPES SIMPLEX TIPOS 1 Y 2
Caractersticas clnicas

VHS-1

Va
de
transmisin Oral
Sndromes
Oral-facial
caractersticos
Ocular
Encefalitis
Paroniquia
Reactividad a antgenos VHS-1 especfico
monoclonales especficos

VHS-2
Genital
Genita
PerianaL
Neonata
Paroniquia
l VHS-2 especfico

El VHS-1 es de localizacin primordialmente extragenital, con predileccin por los tejidos


de origen ectodrmico, mientras que el VHS-2 corresponde al "Herpes progenitalis"
descrito por separado, dentro de las infecciones de transmisin sexual.
La regin oral es la localizacin habitual del Herpes simplex 1, el cual es causa frecuente de
lesiones orofaciales recurrentes y de otro tipo de enfermedades (encefalitis).
EPIDEMIOLOGIA
El ser humano es el nico reservorio natural conocido del virus herpes simplex, aunque
algunos animales de experimentacin pueden infectarse con facilidad.
La infeccin primaria del VHS-1 ocurre sobre todo durante la infancia, mientras que el tipo
2 se presenta en la adolescencia y adultos jvenes activos sexualmente. Las tasas de
infeccin son inversamente proporcionales al estrato socioeconmico.
El principal mecanismo de transmisin es el contacto directo con las secreciones
infectadas. El VHS-1 se trasmite por saliva y el VHS-2 por va genital. Aunque los ttulos
virales son ms altos cuando existen lesiones activas, tambin es frecuente la liberacin
viral en infectados asintomticos. Por lo tanto, la transmisin viral puede efectuarse an en
ausencia de lesiones activas.
La persistencia de la infeccin y la recurrencia de las lesiones son un fenmeno frecuente
tanto para el VHS-1 como para el VHS-2 y por lo comn se producen por reactivacin

endgena. Los factores que la precipitan van desde la luz solar, el viento, traumatismos
locales, fiebre, menstruaciones y hasta estrs emocional.

DIAGNOSTICO
Cuadro Clnico. Los cuadros clnicos causados por este virus se suelen dividir en dos
grupos: el debido a la infeccin primaria y el correspondiente a la infeccin recurrente. En
el primer grupo se incluyen la gingivoestomatitis aguda, la vulvovaginitis aguda y la infeccin
herptica del ojo, que puede llegar a queratitis. Las recurrentes se circunscriben al
"Herpes labialis", queratitis, blefaritis y queratoconjuntivitis. Todos los cuadros son
autolimitados, pero tanto las formas primarias como las recurrentes, se pueden complicar.
Una de estas complicaciones es la Encefalitis herptica y el Eczema herpeticum.
Infeccin primaria. El primer contacto clnico de infeccin por virus del herpes simple
suele ser el ms grave. Los enfermos aquejan fiebre, malestar general, artralgias y por
ltimo la presencia de un grupo de vesculas sobre una base eritematosa, dolorosa,
inflamada y sensible. La gingivoestomatitis es la manifestacin ms comn, cuya gravedad
vara desde la erosin de pequeas reas a la ulceracin extensa de la boca, lengua y encas.
La infeccin puede ser bastante grave como para dificultar la ingesta de alimentos y
lquidos (odinofagia). La curacin tiene lugar en 7 a 14 das, a menos que las lesiones se
sobreinfecten con estafilocos o estreptococos.
Infeccin recurrente. Generalmente existe prurito, dolor o molestias focales que
preceden la aparicin de las vesculas. Las vesculas se rompen espontneamente despus
de unos cuantos das y sanan en una semana sin dejar secuelas.
LECTURAS RECOMENDADAS
1. Callen JP, Cooper Ma. Dermatologic emergences. Emerg Med. Clin North Am
3:641, 1985
2. Guzmn M. Herpes simple, varicela zoster. En: Medicina Interna. Segunda
Edicin. Editado por F Chalem, JE Escandn, J Campos, R Esguerra.
Fundacin Instituto de Reumatologa e Inmunologa. Editorial Presencia
Ltda. Santaf de Bogot, 1992
3. Guerra Flecha J, Lizarraga Bonelli S. Enfermedades de transmisin sexual:
herpes genital Trib Med 79:29, 1989
Jaramillo AC. Infecciones virales de la piel y sus anexos. En: Fundamentos de Medicina.
Enfermedades Infecciosas. Cuarta edicin. Corporacin para Investigaciones Biolgicas.
CIB. Medelln, 1989

18.- Al encontrarse reparando una de la episiotoma media realizada posterior a la


atencin de parto eutcico, en una mujer de 24 aos de edad aprecia un marcado
incremento en el sangrado transvaginal. La causa ms probable de este fenmeno es:
a) Retencin de restos placentarios
b) Laceracin vaginal
c) Laceracin cervical
d) Atona uterina

La mortalidad materna es un indicador de disparidad social y econmica. Cada ao en todo


el mundo mueren cerca de 600,000 mujeres, entre 15 y 49 aos. Como resultado de
complicaciones relacionadas con el embarazo, el parto y el puerperio. Entre las causas
principales destacan: hemorragia postparto (25 %), se psis (15%), eclampsia (12 %) y labor
prolongada o detenida (8 %).1
1
DEFINICIN
Se define la hemorragia posparto (HPP) como la prdida sangunea de 500 mL. o ms en las
primeras 24 horas despus del parto o el descenso del hematocrito en un 10 % o ms.
CLASIFICACIN
HPP Inmediata.- Prdida sangunea de 500 mL. o ms originada en el canal del parto
dentro de las 24 horas posteriores al parto.
HPP Tarda.- Sangrado anormal o excesivo originado en el canal del parto que se presenta
entre las 24 horas posteriores al parto y el final del puerperio (42 das).
FACTORES DE RIESGO
Se han descrito los siguientes factores de riesgo para la HPP:
Embarazo mltiple
Polihidramnios
Macrosoma
Trabajo de parto disfuncional
Gran multiparidad
Corioamnionitis
Uso inadecuado de oxitcicos
Endometritis en el puerperio
Prpura trombocitopnica
Anestesia general
Administracin previa y reciente de inhibidores uterinos
Placenta previa
Enfermedad de von Willebrand
Desprendimiento prematuro de placenta
Acretismo placentario
CAUSAS DE HEMORRAGIA POSPARTO
A continuacin se lisian las causas ms frecuentes de HPP. Segn su origen, se dividen en
uterinas y no uterinas:
2

Uterinas - No Uterinas
Hipotona o atona uterina
Retencin de placenta o restosplacentarios o membranas
Placentacin anormal (acretismo)
Inversin uterina
Traumatismo uterino (rotura uterina, desgarro cervical)
Laceraciones del canal del parto, incluyendo la episiotoma
Coagulopatas
Hematomas
CUADRO CLNICO
La HPP se caracteriza por los siguientes signos y sntomas:
Sangrado transvaginal de moderado a grave.
tero flcido (no contrado) o desgarros.
Alteraciones hemodinmicas que se manifiestan como: mareos, sudoracin, nuseas,
taquicardia y/o hipotensin arterial.
Oliguria.
tero nacido (no contrado).
Al considerar la HPP, deben diferenciarse dos tipos de situaciones: la prevencin orientada
a minimizar la probabilidad de que una mujer presente hemorragia tras el parto y el
manejo o tratamiento de la hemorragia, cuando sta ya se haya producido

Referencias Bibliogrficas
1.

AbdaRabbo SA: Stepwise uterine devascularization: A novel technique for


management of uncontrollable postpartum hemorrhage with preservation of the
uterus. Am J Obstet Gynecol 1994;171:694-700.

2.

Bakri YN, Linjawi T: Angiographic embolization for control of pelvic genital tract
hemorrhage. Report of 14 cases. Acta Obstet Gynecol Scand 1992;71:17-21.

3.

Bick RL: Disseminated intravascular coagulation. Objetive criteria for diagnosis


and manegement.- Med Clin N Am Vol 1994;78(3):511-43.

4.

B-Lynch C, Coker A, Lawal A II, Abu J, Cowen MC: The B-Lynch surgical technique
for the control of massive postpartum haemorrhage: an alternative to? Five cases
reported. Br J Obstet Gynaecol 1997;104:372-5.

5.

Brahaems D: Unwanted hysterectomies. Lancet 1993;342-61.

6. Braithwaite JL: Variations in origin of the parietal branches of the internal iliac
artery. J Anatomy 1952;1:423-30

19. Se presenta a consulta paciente de 25 aos de edad refiere que desde hace varios das
ha presentado flujo vaginal cuyas caractersticas son: blanquecino, grumoso, sin mal olor,
muy irritante El agente causal ms probable en esta patologa es?
a) Gardnerella
b) Tricomonas
c) Cndida albicans
d) Gonococo

Leucorreas micticas: los hongos dan un flujo abundante, blanco, con grumos (aspecto como
de quesillo cortado) sin mal olor y que es muy irritante de la piel de la regin genital y por
lo tanto genera gran ardor vaginal y prurito (picazn). Son muy frecuentes y por lo general
son producidas por el hongo Candida albicans.
.

Tabla I. Caractersticas del flujo vaginal segn la causa


Cantidad

Color

Consistencia

Olor

Escasamoderada

Blancoamarillento

Grumosa

Indiferente

Tricomonas Aumentada

Amarilloverdoso

Espumosa

Maloliente

Vaginosis

Blancogrisceo

Homogneoadherente

Maloliente

Candidiasis

Moderada

Speroff Leon and Fritz Marc A. Clinical Gynecologic endocrinology and infertility. 7 ed.
Philadelphia: Lippincott Williams and Wilkins, 2005. p. 25 44.

20.- Femenino de 23 aos, acude a su consultorio refiriendo secrecin vaginal ftida.


Exploracin Fsica: especuloscopa se observa secrecin gris que rodea la cpula vaginal. En
la preparacin en fresco se observan cocobacilos que rodean a las clulas epiteliales.
La medida teraputica ms apropiada en esta paciente es?

a)

Clindamicina

b)

Clotrimazol

c)

Metronidazol

d)

Tratamiento para ella y su pareja.

Metronidazol: Tratamiento especfico en vaginosis, la evidencia apoya en primer lugar el


uso de:
metronidazol por va oral, en dosis de 500mg cada 12 horas durante siete das, con una tasa
de curacin de 80 a 90 %. Para el tratamiento local no se ha demostrado una diferencia
significativa en la efectividad para curarla vaginosis, al comparar clindamicina en crema a 2
% y metronidazol vaginal, ni entre stos y el tratamiento con metronidazol por va bucal (Ia
y III).4,8 En vaginosis tampoco se ha demostrado la utilidad de tratar a la pareja sexual
(Ia).

Trejo y PJA, Hernndez LB , Carrasco RJR , Ducoing DDLR


Gua clnica para el diagnstico, tratamiento y prevencin de cervicovaginitis por bacterias,
Trichomonas y Candida
Rev Med IMSS 2003; 41 (Supl 1):71-76

21.- Femenino de 36 aos nulpara tras 2 aos de relaciones sexuales sin contracepcin,
que desde hace 1 ao presenta dismenorrea, dispareunia y sangrado vaginal intermenstrual.
El diagnstico ms probable en sta paciente es:
a) Insuficiencia lutenica.
b) Enfermedad inflamatoria plvica.
c) Dismenorrea funcional.
d) Endometriosis.

La endometriosis consiste en la aparicin y crecimiento de tejido endometrial fuera del


tero, sobre todo en la cavidad plvica como en los ovarios, detrs del tero, en los
ligamentos uterinos, en la vejiga urinaria o en el intestino. Es menos frecuente que la
endometriosis aparezca fuera del abdomen como en los pulmones o en otras partes del
cuerpo.
La endometriosis es una enfermedad relativamente frecuente, que puede afectar a
cualquier mujer en edad frtil, desde la menarquia hasta la menopausia, aunque algunas
veces, la endometriosis puede durar hasta despus de la menopausia. La endometriosis
altera la calidad de vida de las mujeres que la padecen, afectando a sus relaciones de
pareja, familiares, laborales y de reproduccin.

Sntomas
Los sntomas clsicos son la dismenorrea, dolor

plvico, dispareunia,

sangrados

intermestruales y en muchos casos, esterilidad.


El dolor no tiene que ver con el tamao y la severidad de la lesin; generalmente cuanto
menor es la lesin mayor dolor produce. El dolor se agrava con las menstruaciones y en los
casos en que la lesin ocupa el fondo de saco de Douglas, puede dar dispareunia. Existe un
aumento de la PGF2 alfa y PGE2 y un aumento de las contracciones uterinas que podra
deberse a un depsito de endometrio en la cavidad peritoneal.

La esterilidad debido a la endometriosis podra deberse a distintas causas de acuerdo a la


severidad de la patologa. En los casos de endometriosis severa puede haber un factor
tuboperitoneal con adherencias y alteracin en la anatoma de la pelvis que interfiera con el
transporte del esperma y el vulo. En los casos de endometriosis leve hay varios
mecanismos propuestos que justifican su relacin con la infertilidad: foliculognesis
alterada, fase ltea inadecuada, fagocitosis espermtica, mala calidad ovocitaria,
embriotoxicidad y alteracin a nivel de la implantacin.. La produccin de prostaglandinas
por el endometrio ectpico puede afectar la motilidad tubaria, la foliculognesis y la
funcin del cuerpo lteo. Puede haber un aumento de la activacin de los macrfagos
peritoneales en la endometriosis que cause la fagocitosis de los espermas o la secrecin de
citoquinas que pueden ser txicas para el embrin. Segn algunos investigadores habra un
60% de las mujeres con endometriosis que presentan un sndrome de Folculo Luteinizado
no roto (LUF) en el cual el folculo no se rompe en la ovulacin y el vulo queda atrapado.

Referencias bibliogrficas

1. Ruiz V. Endometriosis y fertilidad. Ed. Acosta y Warman, pp. 99


2. Lpes,VH. Palomo E. Incidencias de endometriosis en una poblacin infrtil. XXI
Congreso nacional de Ginecologa y Obtetricia. Guatemala, 1993.
3. El-Eoley, et al. Danazol but not ginadotropin releasing hormone agonists suppresses
autoantibodies in endomeriosis. Fertil Steril 1990; 54:725
4. Acosta AA. Buttram VC Jr. Besch PK, Malinak LR, Van Der Heyden J. A.proposed
classfication of pelvic endometriosis. Obstet Gynecol 1973;42:19.
5. Buttran VC Jr. Evolution of the revised American Fertility classification of
endometriosis. Fert. Steril 1985; 43: 347
6. Lpez VH. Tratamiento mdico-quirrgico de la endometriosis. Simposio El rostro
cambiante de la endometriosis panam 3. 12. 1993.
7. Steinleitner A. Heterolous transplation of activated murine peritonel macrophages
inhibitis gamete interaction in vivo; A paradigm fo endometriosis associted subfertility.
Fertil Steril 1990; 54:725.
8. Damewood M. Effect of serum from patients with minimal to mild endometriosis on
mouse embryo growth. Fertil Steril 1990; 54: 917
9. Proug S. Peritoneal fluid fracctions from patients with endometriosis do not promote
two-cell mouse embryo growth. Fertil Steril 1990; 54: 927.

22.- Se presenta paciente de 40 aos de edad a su consultorio refiriendo mastalgia que


es ms severa antes de la menstruacin. A la palpacin hay nodularidad excesiva,
hiperestesia y reas qusticas que la paciente refiere disminuyen en tamao despus de la
menstruacin. El diagnstico ms probable es:
a) Mastopata fibroqustica
b) Fibroadenomas
c) Papiloma intraductal
d) Cncer de mama

Es raro encontrar una mujer mayor de 35 aos a quien no le hayan dicho, en un examen
fsico mamario, ecogrfico o mamogrfico, que tiene quistes en la mama o que su mama es
mastoptica.
Es el trastorno benigno de la mama ms frecuente y consiste en un aumento del tejido
mamario, especialmente en las zonas superiores y externas de las mamas, hacia las axilas,
que las hace ms densas.
La mastopata fibroqustica suele presentarse en ambas mamas, aunque puede ser de
diferente intensidad en una que en otra.

Puede presentarse a cualquier edad despus del inicio de la menstruacin, pero es ms


probable que aparezca entre los 30 aos y la menopausia. Raramente se presenta ms tarde
de esa edad.

El origen de este trastorno es funcional y responde a desequilibrios de las hormonas


sexuales femeninas y puede condicionar la aparicin de quistes mamarios.

Los sntomas pueden fluctuar de leves a severos en una mastopata fibroquistica mamaria,
se acentan tpicamente antes de cada perodo menstrual y desaparecen inmediatamente
despus.
Los sntomas abarcan:

Consistencia de protuberancias (como de "guijarros"), irregular y densa del tejido


mamario
o generalmente ms notoria en la parte superior externa de la mama
Molestia en las mamas
o generalmente en ambas mamas

o puede ser persistente o puede aparecer y desaparecer


Sensacin de llenura en las mamas
Sensibilidad y dolor sordo e intenso
Sensibilidad y edema premenstrual
Secrecin ocasional del pezn

Bibliografa:

Jones III HW, Wentz AC. Tratado de Ginecologa de Novak. Editorial


Interamericana-McGraw Hill. Undcima Edicin 1994.
DiSaia-Creasman. Oncologa Ginecolgica Clnica. Editorial Mosby. Cuarta Edicin
1994.
Van Dinh T. Sumario de Patologa Ginecolgica. Editorial La Prensa Mdica
Mexicana. 1992.
Pernoll ML. Diagnstico y Tratamiento Ginecoobsttricos. El Manual Moderno.
Mxico. Sexta Edicin 1991.
Alvarez-Bravo A. Diagnstico de los trastornos menstruales y hemorrgicos. En:
Alfonso Alvarez Bravo y su obra. Editorial Marketing y Publicidad SA. Tomo I.
1993.
Vzquez E. Aspectos histoqumicos del endometrio humano despus del tratamiento
con progestgenos sintticos. Gac Md Mx 1966; 96: 1277-93.
Huerta MR, Malacara JM, Rivera-Cisneros A, Daz Cisneros FJ. Sntomas en
adolescentes de dos ciudades de Mxico y su asociacin con el ciclo menstual. Ginec
Obstet Mx 1994; 62: 146-50.

23.- Se trata de mujer de 32 aos que cursa en ste momento con diagnstico de
preclampsia leve, el frmaco de eleccin que se administra en esta patologa es:

a)
b)
c)
d)

Nifedipina.
Inhibidores de la enzima convertidora de angiotensina.
Clonidinas.
Alfametildopa.

Prevenir complicaciones a corto plazo de las mujeres con PA elevada que


comprometa el bienestar fetal
Cuando la PAS es mayor o igual a 150 mmHg y la PAD mayor o igual a 100 mmHg.




El propsito es alcanzar cifras de TA alrededor de 140/90.


La medicacin antihipertensiva se reserva para los casos en que la PAD 100 mmHg.




Se recomienda continuar el tratamiento antihipertensivo previo al embarazo,


exceptuando el uso de IECA.
La alfametildopa y la hidralazina va oral son los frmacos de eleccin dado su uso
extensivo con seguridad y eficacia y sin efectos colaterales para el feto (excepto
hidralazina en lupus).

ALFA METILDOPA
500-2000 MG/DA

HIDRALAZINA
50-200 MG/DA
LABETALOL
100-400 MG/DIA
ATENOLOL
50-200 MG/DA
NIFEDIPINA
10-30 MG/DA
1. Aagard K, Belfort M. Eclampsia: Morbility, mortality, and management. Clin
Obstet Gynecolol. 2005; 48: 12-23.
2. Oyarzn E. Sndrome hipertensivo del embarazo en Oyarzn E. Ed. Embarazo de alto
riesgo. Ediciones Universidad Catlica de Chile. Santiago. 1997: 157175.
3. Roberts J, Redman C. Pre-eclamsia: More than pregnancy induced hypertens

24.- Paciente de 17 aos que acude por sangrado vaginal y que en la ecografa se observa un
saco gestacional de aproximadas 7 semanas, sin presencia de
actividad cardaca
embrionaria, localizado a nivel infundibular. La paciente est hemodinamicamente estable.
El tratamiento ms indicado es :

a)
b)
c)
d)

Metrotexate a altas dosis durante un mes.


Expresin tubrica o salpingostoma por laparoscopia.
Actitud expectante.
Alta y revisin en una semana.

Con estabilidad hemodinmica:


a). El dolor ser ms intenso y puede irradiarse a otros sitios, a todo el abdomen o al
hombro (signo de Laffon).
b). La hemorragia, generalmente, no variar.
c). la palpacin de la tumoracin anexial es muy dolorosa.
d). Al examen, el fondo del saco de Douglas puede estar abombado o ser muy doloroso.
e). Cuando se sospeche la rotura, no debe diferirse s confirmacin por puncin del fondo
del saco de Douglas o del abdomen. La puncin tambin puede realizarse en pacientes sin
evidencias de complicacin para poder asegurar que no existe un hemoperitoneo pequeo o
de poco tiempo de evolucin.

f). En dependencia del tiempo de evolucin del hemoperitoneo, se encontrarn signos de


irritacin peritoneal e intestinal por la sangre libre.
g). Ante la sospecha o el diagnstico presuntivo de embarazo ectpico, en una
paciente estable, debe
realizarse una laparoscopia confirmativa que puede ser
seguida de ciruga laparoscpica o de una laparotoma para la operacin convencional

EMBARAZO ECTPICO
SITIOS DE
IMPLANTACIN
Ampular
Itsmica
Intersticial
Infundibular

EMBARAZO ECTPICO
TRATAMIENTO CON METOTREXATO
 Mayor xito:




Embarazo menor a 6 SDG


Masa tubaria menor de 3.5cm
Feto sin latido cardaco

Teran JMetotrexate y conducta expectante en el embarazo ectpico no roto


Rev Obstet Ginecol Venez.2002:62:3

EMBARAZO ECTPICO
TRATAMIENTO QUIRRGICO






Salpingocentesis
Expresin de las
fimbrias
Salpingectoma
Salpingostomia
lineal
Reseccin
segmentaria
Hajenius PJ, Mol BWJ, Bossujt PMM, Ankum WN, Vander Veen F. Intervenciones para el embarazo ectpico tubarico
En: The Cochrane Library Isue 1; Oxford Update Software.

25.- Ante una paciente de 33 aos que acude a consulta con antecedentes de G4 C2 A1,
refiere que ha presentado durante el primero y segundo trimestres de su embarazo
manchado con frecuencia intermitente ,a las 34 SDG inicia con hemorragia abundante,
repentina e indolora, su principal sospecha es:

a) Coriocarcinoma
b) Ruptura uterina
c) Placenta previa
d) Desprendimiento grave de placenta normoinserta

PLACENTA PREVIA
DEFINICIN:
Es cuando la placenta se implanta sobre o muy cerca del orificio cervical interno y una
parte de la placenta precede a la parte fetal que se presenta.
INCIDENCIA:
Esta es difcil determinar ya que muchos casos pasan desapercibidos, sobre todo cuando
ocurren los abortos en embarazos tempranos.
La prevalencia vara de 1 en 100 a 1 en 850 nacidos vivos, pero solo el 20% total.
Etiologa:
Edad avanzada, multparas, paciente con cesreas previas, paciente con aborto de
repeticin, esto debido a las gestaciones previas.
TIPOS:
Insercin baja.- Es cuando el borde placentario se encuentra en el segmento inferior a
menos de 6 cm del orifico cervical interno.
Marginal.- Es cuando el borde placentario alcanza los mrgenes del orificio cervical interno.
Parcial.- Es esta la placenta cubre parcialmente el orificio cervical interno.
Total.- La placenta cubre la totalidad del orificio cervical interno an con dilatacin
cervical avanzada.
DIAGNOSTICO:
La caracterstica es el STV de aparicin brusca en forma indolora en el segundo o tercer
trimestre. Frecuentemente hay ausencia de dolor a actividad uterina que son parmetros
para hacer el diagnstico.

La mayor incidencia de sangrado aparece a las 33-34 sdg.


ESTUDIOS DE GABINETE:
El estudio ms utilizado es la ultrasonografa obsttrica.
TRATAMIENTO:
El manejo va a depender de factores como son:
Edad gestacional, magnitud del sangrado, si hay trabajo de parto, variedad de placenta
previa y complicaciones materna.

BIBLIOGRAFA:
1.- Waxler P, Gottesfeld KR. Early diagnosis of placenta previa. Obstet Gynecol
1979;54:231-32.
2.- Cabrero-Roura L. Riesgo elevado obsttrico. Ed. Masson 1996; pp; 109-118.
3.- Patrick J, Placenta Previa, Clinical Obst and Gynecology 1990;33(3): 414-421.
4.- Chapman M, Furtenes ET, Significance of ultrasound in location of placenta in early
pregnancy Br J Obst Gynecol 197;86: 846.
57

26.- Se trata de paciente femenino de 26 aos de edad con deseo de un embarazo,


antecedentes gineco-obsttricos: G3 A2 - P1, se le realiza una histerosalpingografa, se
constata que existe un sndrome de Asherman. Ello significa que se trata de:

a)
b)
c)
d)

Sinequias uterinas
tero bicorne
Endometriosis en la trompa
Insuficiencia istmico cervical

El sndrome de Asherman es una enfermedad ginecolgica rara que se caracteriza por la


presencia de sinequias (adherencias) intrauterinas que pueden ocasionar amenorrea
(ausencia de perodos menstruales regulares) e infertilidad.

En 1894 Heinrich Fritsch describe por primera vez la presencia de sinequias intrauterinas
de tipo postraumtico, en una paciente que desarroll una amenorrea secundaria a un
curetaje. Posteriormente en 1927 Bass inform de veinte casos de atresia (oclusin de una
abertura natural) cervical tras abortos inducidos, pero no fue hasta 1948, cuando Joseph
G. Asherman recopil la informacin hasta entonces existente y acu el nombre con el que
se conoce actualmente a la enfermedad.

Asherman describi originalmente dos tipos diferentes de amenorrea secundaria, en


funcin de su etiologa (estudio de las causas de las enfermedades): la amenorrea
traumtica atrtica, debida a estenosis del orificio cervical interno y la amenorrea debida
a adherencias intrauterinas. Posteriormente ambas entidades se agruparon en una nica
entidad bajo el nombre de sndrome de Asherman.

Suele presentarse en mayor proporcin tras dilataciones y curetajes uterinos de repeticin


y sobre todo si se realizan durante el embarazo o si existe infeccin uterina en el momento
en el que se realizan estas intervenciones.

Las adherencias intrauterinas pueden producirse debido a cualquier factor que lleve a una
destruccin de las paredes del miometrio (capa muscular de la pared del tero). Sin
embargo, hay que distinguir entre factores predisponentes, siendo el principal de ellos el
embarazo y factores causales, entre los que se encuentran: traumatismos uterinos,
intervenciones quirrgicas que afecten al tero, agentes fsicos o qumicos e infecciones
uterinas por tuberculosis o esquistosomiasis. En cualquier caso, el factor ms importante es
el trauma uterino en el momento del parto o el puerperio.

El cuadro clnico es muy variable y las manifestaciones clnicas varan con el grado de
oclusin de la cavidad uterina y la severidad de las adherencias, pudiendo presentarse:
esterilidad cuando la oclusin de la cavidad uterina incluye porciones proximales (ms cerca
de un centro, tronco o lnea media) de las trompas de Falopio o cuando las adherencias
impiden la nidacin del huevo; las pacientes presentan con frecuencia amenorrea,
oligomenorrea (disminucin de la frecuencia de las menstruaciones), dismenorrea
(menstruacin dolorosa) y abortos repetidos.
1.

Hysteroscopic treatment of severe Asherman's syndrome and subsequent fertility.


Capella-Allouc S; Hum Reprod, 1999 May.

27.- Femenino de 36 aos, es atendida en consulta externa con reporte de papanicolaou


que reporta un NIC I, la especuloscopa se observa crvix con ectropin periorificiario.
El mtodo ms sensible para corroborar el diagnstico en esta paciente es:
a)
b)
c)
d)

Papanicolaou.
Colposcopa
Exudado vaginal.
Prueba de koh.

9.5.2 Las pacientes a quienes se les realiz citologa cervical, cuyo resultado es LEIBG
(infeccin por VPH, displasia leve o NIC 1); LEIAG (displasia moderada y grave o NIC 2 y 3)
o cncer deben enviarse a una clnica de colposcopa, para realizar estudio colposcpico.
9.5.3 Si el resultado de la citologa es LEIBG, la colposcopa es satisfactoria y sin
evidencia de LEIBG, se realizar control citolgico en un ao (Apndice Normativo A)

9.5.4 Si la citologa es de LEIBG, la colposcopa es satisfactoria y existe evidencia de


lesin, se debe tomar una biopsia dirigida.
9.5.4.1 Si la biopsia dirigida es negativa, se realizar nueva colposcopa para verificar el
diagnstico y en caso necesario, tomar nueva biopsia dirigida y revalorar.
9.5.4.2 Si la biopsia dirigida es reportada como LEIBG se podr dar tratamiento
conservador: criociruga, electrociruga o laserterapia (slo si cumple con las condiciones
referidas en el Apndice 1) o se podr mantener a la paciente en vigilancia en la clnica de
colposcopa, con colposcopa y estudio citolgico cada seis meses, durante 24 meses.
Jueves 31 de mayo de 2007 DIARIO OFICIAL (Primera Seccin)
9.5.4.3 Si la biopsia dirigida es reportada como LEIAG (Lesin Intraepitelial Escamosa de
Alto Grado) se realizar tratamiento conservador (electrociruga o laserterapia). En las
mujeres posmenopusicas, dependiendo de las condiciones anatmicas del crvix, se
realizar tratamiento conservador en la clnica de colposcopa o tratamiento quirrgico
(histerectoma extrafascial) en el servicio que corresponda.
9.5.4.4 Si la biopsia dirigida reporta cncer microinvasor o invasor, la paciente se
transferir a un Servicio o Centro Oncolgico para su tratamiento correspondiente.
9.5.4.5 Si la citologa reporta LEIBG y la colposcopa es no satisfactoria, se tomar
cepillado endocervical (Apndice Normativo A)
9.6 En caso de colposcopa no satisfactoria, negativa a LEIBG y con cepillado endocervical
negativo, se continuar su control en la clnica de colposcopa en seis meses, con colposcopa
y citologa.
9.6.1.1 Si el cepillado endocervical reporta LEIBG se tratar a la paciente como LEIAG,
con mtodos conservadores escisionales.

Jueves 31 de mayo de 2007 DIARIO OFICIAL (Primera Seccin)


Modificacin a la Norma Oficial Mexicana NOM-014-SSA2-1994, Para la prevencin,
deteccin, diagnstico, tratamiento, control y vigilancia epidemiolgica del cncer
crvico uterino.
Al margen un sello con el Escudo Nacional, que dice: Estados Unidos Mexicanos.- Secretara
de Salud.
MODIFICACION A LA NORMA OFICIAL MEXICANA NOM-014-SSA2-1994, PARA LA
PREVENCION,
DETECCION,
DIAGNOSTICO,
TRATAMIENTO,
CONTROL
Y
VIGILANCIA
EPIDEMIOLOGICA DEL CANCER CERVICO UTERINO.

28.- Femenino

de 29 aos acude al servicio de consulta externa refiriendo presentar


baches amenorreicos, acn, hirsutismo y esterilidad de aproximados 2 aos de evolucin
E.F. con ndice de masa corporal 31 KG/M2, es diagnstico ms probable es:

a)
b)
c)
d)

Hipotiroidismo.
Fallo ovrico precoz.
Amenorrea de causa uterina.
Sndrome del ovario poliqustico.

El sndrome de ovarios poliqusticos (SOPQ) afecta aproximadamente a un 4% de mujeres


en edad reproductiva y se caracteriza por anovulacin crnica e hiperandrogenismo. Es la
causa ms comn de infertilidad en mujeres.
Se caracteriza clnicamente por acn, alopecia, hirsutismo, irregularidades menstruales
e infertilidad.
Los hallazgos de laboratorio ms frecuentes son: aumento de la hormona luteinizante
(LH), aumento de la relacin LH/FSH (hormona folculoestimulante), aumento de
andrgenos (tanto ovricos como adrenales) y de estrgenos circulantes. Otros hallazgos
de laboratorio habituales son una prueba tolerancia oral a la glucosa anormal y alteraciones
en el perfil lipdico.
Todo esto junto con las imgenes ecocardiogrficas caractersticas definen al sndrome.
La teraputica permite dos grandes enfoques que pueden superponerse: la correccin de
las manifestaciones de hiperandrogenismo y el tratamiento de las alteraciones del eje
reproductivo (anovulacin, esterilidad).
Los antiandrgenos estn fundamentalmente indicados para tratar los sntomas
virilizantes.
Las alternativas para inducir la ovulacin son numerosas: al citrato de clomifeno y a la
antigua reseccin en cua se agregan las gonadotrofinas humanas, pulsos de GnRH (hormona
liberadora de gonadotrofinas), medidas o frmacos para modificar los niveles de insulina, y
finalmente tcnicas quirrgicas endoscpicas para reducir la masa ovrica.
Revista de Posgrado de la VIa Ctedra de Medicina - N 125 Marzo 2003
Pg. 37-40
SINDROME DE OVARIOS POLIQUISTICOS
Dra. Sandra Beneyto, Dra. Mara Andrea Ferreyra, Dr. Andrs Galfrascoli,
Dr. Andrs Gonzlez, Dra. Susana Sosa

29.- Femenino de 43 aos, gesta- 4, partos-3, abortos-1, con diagnstico de anemia


ferropnica, de 9.5 g/dl, refiere ciclos menstruales de 31,32 x 8,9 das de duracin,
acompaados de cogulos, los cuales aparecieron despus del nacimiento de su segundo hijo
hace 13 aos. E.F.: Buen estado general, TA 130/80, genitales con evidencia de sangrado
activo, al tacto vaginal se detecta tero de consistencia firme voluminoso, irregular,
aproximadamente de 12 cm. anexos libres.
El diagnstico ms probable es:
a) Adenomiosis uterina.
b) Cncer cervicouterino.
c) Miomatosis uterina.
d) Hiperpalsia adenomatosa de endometrio.

MIOMATOSIS UTERINA
Definicin:
Tumor benigno que se origina en el miometrio, por lo que su componente histolgico
predominante es el tejido muscular y, en menor medida, el conectivo y fibroso. El nico

tratamiento efectivo es el quirrgico; sin embargo, slo requieren ser tratados aquellos que
producen sntomas.
Evaluacin y Diagnstico:
Historia:
1.
2.
3.
4.

El sntoma ms frecuente suele ser la hemorragia uterina.


Los sntomas principales estn relacionados con el crecimiento del tumor.
La paciente puede notar una masa en hipogastrio o abdomen inferior.
La masa se puede asociar a dolor plvico, o manifestaciones por compresin de
rganos o estructuras vecinas.
5. Puede haber alteracin de la fertilidad.

Examen Fsico:
1. Se debe realizar con la vejiga y el recto vacuo.
2. El hallazgo primordial es el aumento de volumen y consistencia del tero, el cual
puede ser simtrico (ndulos submucosos) o irregular (ndulos intramurales o
subserosos).
Exmenes Auxiliares:
1.
2.
3.
4.

5.
6.
7.

Papanicolau crvicovaginal: Indicado siempre; permite descartar neoplasia epitelial


cervical o cncer infiltrante de crvix.
Ultrasonido transabdominal y transvaginal: Indicado siempre; permite evaluar la
localizacin, tamao y nmero aproximado de miomas.
Hemoglobina, hematocrito: Indicado cuando hay historia de sangrado; orienta en la
severidad del sangrado y anemia.
Hemograma y VSG: Indicado cuando hay historia de fiebre; si es anormal sugiere
infeccin o necrosis del mioma (puede ser apropiado descartar infeccin de otro
rgano o sistema).
Grupo sanguneo y factor Rh: Si hay anemia severa o en el preoperatorio.
Perfil de coagulacin (tiempo de protrombina y de tromboplastina parcial, recuento
de plaquetas): Si hay historia de sangrado exagerado.
Gonadotrofina corinica (subunidad e srica): Permite descartar posible embarazo
en casos de ciclos irregulares, retraso menstrual o tero de consistencia blanda.

Diagnstico Diferencial:
1.
2.
3.
4.
5.
6.

Embarazo.
Tumor de ovario.
Enfermedad inflamatoria plvica, complejo inflamatorio anexial plvico.
Endometriosis.
Adenomiosis.
Tumor extragenital: colon, retroperitoneo.

Referencias Bibliogrficas:
1.

Hillard PA. Benign Diseases of the Female Reproductive Tract: Symptoms and
Signs. En: Berek JS, Adashi EY, Hillard PA, eds. Novak's Gynecology. Baltimore:
Williams and Wilkins, 1996:331-97.

1.

Hutchins FL, Greenber MD. Miomas Uterinos: Diagnstico e Indicaciones de


Tratamiento. Clinicas de Ginecologa y Obstetricia. Temas Actuales. 1995;5:609-14.
2. Davis KM, Sclass WD. Tratamiento Mdico para Miomatosis Uterina. Clinicas de
Ginecologa y Obstetricia. Temas Actuales. 1995;5:671-81.

1.

Selwyn P, Oskowitz MB. Leiomyomata Uteri. En: Friedman EA, ed. Gynecological
Decision Making. St. Louis: Mosby, 1983:148-9.

1.

Diaz Huamn V. Tumores Benignos del Aparato Reproductor Femenino. En: Ludmir
A, Cervantes R, Castellano C, eds. Ginecologa y Obstetricia, Prevencin Diagnstico - Tratamiento. Lima: Concytec, 1996:907-25.

30.- Mujer de 26 aos, con tumor anexial de 6 cm, dolor abdominal, fiebre, leucorrea, con
historia de cervicovaginitis de repeticin y dispareunia crnica, ltima menstruacin hace
una semana. El diagnstico ms probable es:
a)
b)
c)
d)

Cistadenoma
Embarazo ectpico
Quiste de ovario
Enfermedad plvica inflamatoria

La EIP puede cursar con los siguientes sntomas:

Dolor abdominal bajo (incluyendo dolor anexial, dispareunia). Es el sntoma ms


frecuente (95%)
Aumento del flujo vaginal, flujo de caractersticas anormales (74%)
Sangrado anormal (intermestrual, poscoital) (45%)
Sntomas urinarios (35%)
Vmitos (14%)
Es posible la ausencia de sntomas

Y en ella podemos encontrar estos signos:

Dolor a la movilizacin del cuello, dolor anexial en la exploracin vaginal bimanual


(99%)
En el examen con espculo observamos cervicitis y descarga endocervical purulenta
(74%)
Fiebre (> 38 C) (menos del 47%).

Masa plvica: sugiere abceso tuboovrico (ATO)


Peritonitis

CRITERIOS CLINICOS PARA EL DIAGNOSTICO DE SALPINGITIS


a. Dolor abdominal con o sin rebote.
b. Sensibilidad a la movilizacin del crvix.
c. Sensibilidad anexial.
Los tres criterios anteriores son necesarios para establecer el diagnstico, con uno o ms
de los siguientes:
a. Extendido de Gram de endocrvix positivo, para diplococos gram negativos
intracelulares
b. Temperatura mayor de 38C
c. Leucocitosis (mayor de 10.000 por c.c.)
d. Material purulento (positivo para leucocitos) en la cavidad peritoneal
obtenido por culdocentesis o laparoscopia.
Establecido el diagnstico clnico de EPI, se debe hacer la definicin del estado clnico y
anatmico de la patologa plvica:
a) No complicada (limitada a trompas u ovarios)
1) Sin peritonitis plvica
2) Con peritonitis plvica
b) Complicada (masa inflamatoria o absceso que compromete trompa (s) u ovario (s)
1) Sin peritonitis plvica
2) Con peritonitis plvica

CLASIFICACION LAPAROSCOPICA DE LA ENFERMEDAD PELVICA INFLAMATORIA


El stndar de oro para el diagnstico de EPI es la laparoscopia, ya que adems de visualizar
directamente los rganos plvicos, permite la toma de muestras para estudios
bacteriolgicos.
Los criterios laparoscpicos para el diagnstico de EPI, se describen en la tabla No.3.

Tabla No. 3
CLASIFICACION LAPAROSCOPICA DE LA EPI
Leve

Eritema, edema, las trompas se mueven


libremente. No hay exudado purulento.

Moderada

Eritema, edema ms marcado, material


purulento evidente. No hay movimiento libre
de las trompas. La fimbria puede no ser
evidente.

Severa

Presencia de pioslpinx y/o absceso

EPI
Diagnstico
Dolor abdominal bajo y/o dolor a la movilizacin cervical
y anexial y:
Fiebre de 38 C o mayor
leucocitosis > 10, 500 mm3, eritrosedimentacin y PCR elevada
Bacterias o leucocitos en liquido peritoneal
Masa anexial palpable o por USG
Laparoscopia
Sndrome de Fitz- Hugh-Curtis

Bibliografa:
Beigi RH, Wiesenfeld HC. Pelvic inflammatory disease: new diagnostic criteria and
treatment. Obstet Gynecol Clin Norh Am. 2003; 30 (4): 777 93
Center for Disease Control. Guidelines for treatment of sexually transmited diseases.
MMWR Recomm Rep. 2002 May 10;51(RR-6):1-78
Center for Disease Control. Guidelines for prevention and management (MMWR. 40: 1 - 25
1991) Pelvic inflammatory disease: guidelines for prevention and management.
MMWR Recomm Rep. 1991 Apr 26;40(RR-5):1-25.
Hager WD, Eschenbach DA, Spence MR, Sweet RL. Criteria for diagnosis and grading of
salpingitis. Obstet Gynecol. 1983 Jan;61(1):113-4.
Prodigy Guidance. Pelvic inflammatory disease. [Internet]. UK : NHS, Department of
Health; 2003. [Acceso 18 de Junio de 2005]. Disponible en:
Ross J. Pelvic inflammatory disease. Clin Evid. 2004 Dec;(12):2259-65.
Royal College of Obstetricians and Gynaecologists. Pelvic Inflammatory Disease. Guideline
n 32. [Internet]. RCOG; Mayo 2003. [Acceso 18 de Junio de 2005].

31.- Femenino de 25 aos con antecedentes de G/2, P/1, C/1 acude al servicio de consulta
externa, refiere que presenta una secrecin transvaginal bastante lquida, de baja
viscosidad, maloliente de color amarillo y gris, espumoso.
El tratamiento de eleccin para esta entidad es:

a) Metronidazol 500 mg. VO c/12 por 7 a 10 das


b) Ampicilina 1g VO c/ 6 hrs.
c) Clotrimazol + Metronidazol 500 mg VO c/12 hrs. por 7 a das
d) Clindamicina 300 mg VO cada 12 hrs. x 5 das.

Referencias bibliogrficas:
1. Secretara de Salud. Norma Oficial Mexicana NOM -039-SSA2-2002, Para la prevencin
y control de las infecciones de transmisin sexual. D.O.F. 19 de Septiembre 2003.
2. Kettler H, White K, Hawkes S. Mapping the landscape for sexually transmitted
infections: key findings and recommendations. Geneva, TDR (TDR/STI/ IDE/04.1).
3. CDC. Trends in Reportable Sexually Transmitted Diseases in the United States. CDC,
National Report. 2004
4. Distribucin de los casos nuevos de enfermedades por mes Estados Unidos Mexicanos
2004. Sistema nico de Informacin para la Vigilancia Epidemiolgica/Direccin General
de Epidemiologa/SSA
5. Aral S O. Sexual risk behaviour and infection: epidemiological considerations. Sex.
Transm. Inf. 2004;80:8-12

32.- Femenino de 55 aos, se queja de presin plvica y una masa en la entrada vaginal.
Anetecedentes: G.3 P.3, el ltimo con peso al nacer de 4,500 grs. FUR hace tres aos. Sin
terapia de reemplazo. Historia de tabaquismo positivo a razn de 40 cajetillas ao.
Actualmente refiere dificultad para evacuar, tos crnica, presenta una orina de 60 cc. En
la exploracin plvica, en hallazgo ms probable es :

a)
b)
c)
d)
e)

Rectocele
Cistocele
Enterocele
Uretrocele

El Rectocele es una hernia de la pared anterior del Recto hacia la porcin posterior de la
vagina. La incidencia real de esta deficiencia anatmica es desconocida y en muchas
ocasiones es un resultado del paso del tiempo. Es un hallazgo muy frecuente del examen
perineal, siendo en mltiples ocasiones asintomtico.
El rectocele puede ser un hallazgo importante del sndrome de Obstruccin Defecatoria
(SOD). No debe tomarse como una deficiencia anatmica nica, sino como parte importante
de un problema anatomo-fisiolgico complejo.
Un principio importante es la etiologa y anatomo-patologa del rectocele. Existen varias
teoras sin consenso principal. La existencia, deficiencia o alteraciones del septo
rectovaginal son controversias importantes. No existe una fascia visceral que separe el
recto de la vagina o que forme un septo especfico. Existe frontera entre donde termina la
pared anterior del recto y donde comienza la pared de la vagina, pero mltiples estudios no
han encontrado un septo rectovaginal especfico. El septo puede estar formado de una
pelcula casi transparente hasta una pared de consistencia fibromuscular fuerte.
El rectocele es un hallazgo comn. Se presenta en el 80% de las pacientes femeninas y
13% de los masculinos en una defeco grafa (> a 1cm.) Entre ms grande es el rectocele,
mayor son los sntomas asociados, Dificultad en la evacuacin, constipacin crnica, dolor
rectal y perineal, sensacin de masa y en ocasiones sangrado. La necesidad de presin
manual para ayudar a la evacuacin o para vaciar el rectocele es comn en ms del 50% de
los pacientes.
El diagnostico se hace con un simple tacto rectal e inspeccin vaginal, pero debe recordarse
que rara vez es el rectocele un hallazgo aislado. La presencia cistocele, peritoneocele,
enterocele u otros prolapsos perineales y problemas funcionales deben ser descartados
antes de proponer la reparacin quirrgica del rectocele. Defeco grafa,
Pruebas de funcin fisiolgica del piso plvico, pruebas de funcin urinaria, evaluacin del
esfnter anorectal por ultrasonido y hasta la resonancia magntica han sido propuestos
antes de la ciruga.

Bibliografa:

Rectocele: Pathogenesis and surgical managment. Zbar AP, Linemann A, Fritsch H,


Beer-Gabel M, Pescatori M. Int J Colorectal Dis. (2003) 18:369-384.
Evaluation and Treatment of Women with rectocele. Cundiff GW, Fenner D, Obstetrics
and Ginecology 104(6): 1403-1416
Stapled transanal rectal resection to treat obstructed defecation caused by rectal
intussusseption and rectocele. Renzi A, Izzo D, Di Sanrno (26) 21:661-667
Rectocele repair using biomaterial augmentation. Altman D, Melgren A, Zetterstrom J.
Obstet Gynecol (2005) 60(11)753-760.

33.- Femenino de 25 aos, con embarazo de trmino, sin antecedentes de control prenatal.
G 3. C-1. Se ingresa al servicio de obstetricia por presentar actividad uterina regular y
dolorosa. Ef.: Deambulante, tranquila, adecuada coloracin de tegumentos, abdomen con
fondo uterino a 32 cm. con producto nico vivo en situacin transversa dorso inferior FCF
144, al tacto vaginal crvix dilatado a 3 cm. y membranas ntegras. Se realiza cesrea con
retencin de placenta e invasin a vejiga.
La alteracin placentaria que presenta esta paciente es:
a)
b)
c)
d)

Placenta increta
Placenta acreta
Placenta percreta
Placenta marginal

Es la penetracin y adherencia anormal de la placenta en la pared uterina.

Se divide en:
 Placenta acreta.
 Placenta increta.
 Placenta percreta.


ACRETA: Las vellosidades se adhieren al miometrio.

INCRETA: Penetran ms de la mitad del espesor del miometrio.

PERCRETA: Atraviesa todo el espesor del miometrio, llegando a la


serosa, incluso atravesndola y adhirindose a rganos vecinos.

Factores:








Endometrisis previa.
Tumores submucosos. (Miomas)
Cicatrz uterina previa. (Cesrea, miomectoma)
Implantacin baja. (Placenta previa)
Malformaciones placentarias. (Placenta extracorial)
Legrado enrgico previo.
Extraccin manual previa de una placenta.

Diagnstico transparto:






Placenta retenida por ms de 20 minutos.


Imposibilidad para encontrar un plano de separacin placentaria cuando se intenta
su extraccin manual.
Hemorragia incontrolable despus de la pseudoextraccin.
El diagnstico histopatolgico corrobora el diagnstico clnico.
Escenario menos deseable.

Tratamiento:


Histerectoma Obsttrica.
 Constituye una ciruga no planeada y secundaria al hallazgo del acretismo
placentario con sangrado incohercible.

Cesrea-Histerectoma. (Con diagnstico previo)


 Ciruga planificada ante un correcto diagnstico prenatal.

Recomendacin ACOG:
 Maduracin pulmonar intrauterina.
 Inyectar al cordn umbilical 50 mg de metrotexate.
 Ligar el cordn en el nacimiento placentario y dejar la placenta in-situ.
 Embolizacin inmediata de arterias uterinas bilaterales, as como de ramas
de la divisin anterior de la arteria iliaca interna con alcohol polivinlico.
 Continuar con 5 dosis I.M. de 50 mg de metrotexate y cuantificar niveles
de hCG.
 Programar Histerectoma Total Radical Abdominal y/o Cistectoma parcial
y/o reseccin pared anterior recto.

Lee et al. Conservative Management of Placenta Percreta. Obstet Gynecol, 112(2):421-424

34.- Femenino 44 aos, G-3 P-2 A-1, se detecta anemia ferropnica, de 9.5 g/dl, refiere
ciclos menstruales de 31,32 x 8,9 das de duracin, acompaados de cogulos, los cuales
aparecieron despus del nacimiento de su segundo hijo hace 14 aos. e.f.: con ligera palidez
de tegumentos, S/V dentro de los parmetros normales, genitales con evidencia de
sangrado activo, al tacto vaginal bimanual se detecta tero de consistencia firme
voluminoso, irregular, aproximadamente de 12 cm. anexos libres. El tratamiento para esta
paciente es:

a)
b)
c)
d)

Histerectoma total sin conservar anexos.


Histerectoma total conservando anexos.
Histerectoma vaginal.
Histerectoma radical.

Tratamiento: La ciruga es el tratamiento ms comn del mioma uterino:


Miomectomia: Extirpar slo el mioma, los tumores nicos y accesibles, la reseccin
histeroscpica de miomas submucosos tambin es posible realizarlo con electrocoagulacin
en pacientes con hemorragias (>90%).
Histerectoma:
La ciruga puede ser abdominal o laparoscpica, la decisin final de la tcnica depender de
la eleccin del cirujano, en base al caso individual y a su experiencia, el procedimiento
puede ser:
-histerectoma subtotal.
-histerectoma total (remocin cervical). Tcnicamente ms sencillo en su realizacin. La
incidencia de carcinoma con origen en el mun cervical es menor al 1% en nuestros das.

Bibliografa:
1. hanafi m. predictors of leiomyoma recurrence after myomectomy. am coll obstet ginecol
2005;
105: 877-880.
2. inclan j, mojarra j. miomectoma histeroscpica. abordaje actual para el manejo de los
miomas
submucosos. reporte de un caso y revisin de la literatura. bol clin hosp. infant edo son
2001;18: 29-34.
3. goldrath mh, husain m. the hysteroscopic management of endometrial leiomyomatosis. j
am

assoc gynecol laparosc 1997; 4: 263-267.


4. clement pb, scully re. mullerin adenofibroma of the uterus with invasion of myometrium
and pelvic veins. int j gynecol pathol 1990; 9: 363-371.
5. ravina jh, herbreteau d, ciraru-vigneron n, bouret jm, houdart e, aymard a, merland jj.
arterial embolization to treat uterine myomata. lancet 1995; 346: 671-672.
6. kunhardt-urquiza e, cruz si, Fernndez-Martnez rl, hernndez-ziga ve: miomatosis de
localizacin poco frecuente. ginecol obstet mex 1997; 65: 541-544.

35.- En el servicio de consulta externa recibe un frotis vaginal el cual reporta la presencia
al microscopio de clulas clave las cuales son propias de infeccin por:
a) Cndida albicans
b) Gardnerella
c) Tricomonas
d) Gonococos

La infeccin por gardnerella (bacteria) tiende a producir una secrecin blanca, gris o de
color amarillo turbio, con un olor ftido o a "pescado" que aumenta cuando la secrecin se
vuelve alcalina, como sucede despus del coito o de lavarse con jabn. Puede haber prurito
o irritacin vulvar, pero por lo general no son muy pronunciados. Al microscopio se aprecia
las famosas clulas clave. Realizando el diagnstico diferencial.

Clulas escamosas de capa intermedia alta, algunas de ellas con ncleos picnticos, y dos de
ellas con el citoplasma cubierto por formas cocceas que borran sus bordes, dndole el
aspecto de clula rebozada o "clula clave". Gardnerella. .

Pernoll M. Enfermedades de trasmisin sexual. En: Manual de Obstetricia y Ginecologa.


Benson/Pernoll Editores.Editorial Interamericana S A. Mxico DF, 1994

36.- A 19 year old man with acute non lymhocytic leukemia is admitted to he hospital 2
weeks after hi first round of chemotherapy. His temperature is 39.2 C, and physical
examination shows a no localized abnormalities. Chest radiograph shows a Hickman
catheter with its tip in the right atrium. The white blood cell is 300/uL with no
polymorphonuclear or band cells in the differential count. Blood cultures are obtained. The
next step is to
a) initiate antistaphylococcal treatment for the possibility of Hickman catheter
related bacteremia
b) administer broad spectrum antibiotics with excellent activity for enteric
gramnegative rods ad Pseudomonas aeruginosa
c) await results of blood cultures and other diagnostic tests because infection could
be caused by almost any microorganism
d) administer parenteral antifungal therapy

Pseudomonas aeruginosa es un patgeno oportunista que causa infecciones del aparato


urinario y respiratorio, de los tejidos blandos, endocarditis y una variedad de infecciones
sistmicas, particularmente en pacientes quemados, con cncer, fibrosis qustica o
compromiso del sistema inmune. Pseudomonas aeruginosa ha emergido como uno de los ms
importantes patgenos hospitalarios causantes de infecciones graves. Las infecciones
asociadas a catteres son una importante causa de morbi-mortalidad. Caractersticas en
pacientes con
infeccin relacionada a catteres: infeccion local; fiebre de origen
desconocido en paciente con cateter de ms de 3 dias; con hemocultivos positivos sin otro
foco probable; normalizacin de la temperatura luego de la retirada del dispositivo.
IDSA Guidelines for the management of intravascular catheter related infections. Clinical
Infectious Diseases 2001; 32:1249.
Centers for Disease Control/ Hospital Infection Control Practices Advisory Committee.
Guidelines for Prevention of Intravascular
Device-Related Infections American Journal of Infection Control 1996: 24: 262-293.

37.- A 6 year old girl who presented skin lesions such macules and papules that evolve into
blisters within hours. The mother states that introduced upper respiratory infection 2
weeks ago. The diagnosis you do it?
a) Herpes Zoster.
b) Rubella
c) Varicella
d) Scarlet fever

Varicela
Manifestaciones clnicas: la infeccin primaria produce fiebre moderada, y un exantema
vesicular genereralizado y pruriginoso. Las lesiones se observan en distintos estados
(mcula, ppula, vescula y costras) y duran alrededor de una semana. Es posible tambin
apreciar vesculas o lceras en la mucosa oral. La reactivacin del virus es responsable del
herpes zoster.
Agente etiolgico: virus varicella zoster (ADN) perteneciente a la familia Herpesviridae
Epidemiologa: los humanos son la nica fuente de contagio para este agente. La transmisin
es a travs del contacto persona a persona y por la ruta respiratoria. Los brotes aparecen
hacia fines del invierno y en primavera. El perodo de incubacin vara entre 10 a 21 das, y
usualmente es de dos semanas. El momento de mayor contagiosidad ocurre desde dos das
antes de que el exantema aparezca, hasta que las lesiones se encuentren en etapa de
costra. En nios inmunodeprimidos, los perodos de incubacin, de contagiosidad y de
erupcin pueden ser ms largos.
Diagnstico: el diagnstico es fundamentalmente clnico; sin embargo, existen situaciones
de diagnstico diferencial de lesiones vesiculares en el husped inmunodeprimido, en que la
inmunofluorescencia directa permite hacer el diagnstico rpido de infeccin por virus
varicela y as, tomar decisiones teraputicas precoces.
Aislamiento del paciente hospitalizado: Aislamiento respiratorio y de contacto por al
menos cinco das desde que comienza el exantema o hasta que todas las vesculas estn en
estado de costra.
Tratamiento: en el husped inmunocompetente el manejo del paciente con infeccin aguda
es sintomtico. Si se requiere uso de antipirticos se recomienda usar slo paracetamol,
por la posible riesgo de desarrollo del Sndrome de Reye con el uso de cido acetilsaliclico.
Debe prevenirse la sobreinfeccin bacteriana de las lesiones, evitando el prurito y el
grataje, manteniendo las uas cortas, bao diario y usando antihistamnicos.
El uso de aciclovir est indicado en sujetos con riesgo de desarrollar complicaciones
(inmunodeprimidos, adolescentes, adultos, pacientes con terapia crnica con saliclicos y
esteroides) y debe iniciarse en las primeras 24 hrs del exantema. El uso de aciclovir puede

tambin considerarse en el segundo caso intrafamiliar ya que en estos pacientes se ha


observado una evolucin ms severa de la enfermedad.

Bibliografa:
Report of the Committee on Infectious Diseases, 25h edition, Red Book 2000.
American Academy of Pediatrics.
Fifth (human parvovirus) and sixth (herpesvirus 6) diseases. Koch WC. Curr Opin
Infect Dis 2001, Jun; 14 (3): 343-356.
Primary human herpesvirus 8 in immunocompetent children. Andreoni M, Sarmati L,
Nicastri E, El Sawaf G, El Zalabani M, Uccella I, et al. JAMA 2002 Mar 13; 287 (10):
1295-300.
Varicella vaccine update. AAP. Pediatrics 2000, Jan 105: 136-141.

38.- Paciente de 40 aos con ndulo mamario indoloro, de bordes imprecisos. La


mamografa revela imagen nodular, con espculas en todos sus mrgenes, y 10
microcalcificaciones finas, agrupadas en el interior. El diagnstico ms probable, entre los
que se citan, es:
a) Fibroadenoma.
b) Carcinoma
c) Quiste
d) Mamografa normal para la edad de la paciente.

Tcnicas diagnsticas
Exploracin
Masa palpable o engrosamiento unilateral. La posibilidad de que una masa palpable en la
mama sea maligna est en relacin con mayor edad, postmenopausia y con las siguientes
caractersticas en el examen fsico: consistencia firme, aspecto slido, bordes irregulares,
escaso desplazamiento sobre la piel, la regin costal o los tejidos que le rodean, unilateral,
no dolorosa y la presencia de adenopatas axilares. Sin embargo, an en ausencia de estos
factores un 10% pueden ser malignas, algunas veces una zona de engrosamiento que no llega
a masa puede ser cncer. La coexistencia de masa y adenopata axilar palpable debe
considerarse cncer mientras no se demuestre lo contrario. El 90 % de las masas suelen
ser lesiones benignas. Las masas de superficie lisa y consistencia elstica estn asociadas
a fibroadenoma en mujeres entre 20-30 aos y a quistes en las mujeres de 30 a 40. La
exploracin a realizar ante esta situacin es una mamografa si hay antecedentes de cncer
de mama y una ecografa sobre todo si existe dolor (ICSI, 2005).
Secrecin por el pezn. Siempre se debe estudiar. Hay mayor riesgo de lesin maligna en el
caso de que la secrecin contenga restos hemticos y est asociado a masa. La citologa del
lquido expulsado slo puede ser tenida en cuenta si es positiva. Est indicado realizar
mamografa y galactografa en el caso de que el exudado se presente en un solo conducto.
La presencia de secrecin lechosa bilateral orienta a causa endocrinolgica se ha de
realizar el diagnstico diferencial de galactorrea (ICSI, 2005).
Dolor. Es uno de los motivos de consulta ms frecuente. En ausencia de masa otros
sntomas de sospecha suele ser debida a tensin premenstrual, dolor condrocostal y a otras
causas (ICSI, 2005). Est asociado con mayor frecuencia a cambios fibroqusticos en la
mama premenopusica.
Sntomas cutneos. La Enfermedad de Paget afecta al pezn y areola de forma unilateral,
clnicamente muy similar a la dermatitis crnica crnica eccematosa se asocia a un
carcinoma mamario intraductal subyacente. (Fitzpatrick, 2001)
La retraccin del pezn o de la piel de presentacin reciente se debe evaluar
cuidadosamente. Los fenmenos inflamatorios del tipo de eritema, induracin, aumento de
temperatura y dolor pueden ser indicativos de un tumor inflamatorio de mal pronstico. En
ocasiones un tumor evolucionado puede dar lugar a un cncer ulcerado.
Imgenes
Mamografa
Tiene una sensibilidad y especificidad del 90%, siendo el mtodo aislado de diagnstico ms
eficaz, aunque en mamas densas pierde sensibilidad. Utilizada para el screening puede
reducir la mortalidad del cncer de mama en un 33%. Nos puede dar el diagnostico, la
presencia de multicentricidad o de lesiones sincrnicas.
La mamografa nos va a valorar distintos tipos de imgenes:
1: Signos primarios:
A.
Masa dominante: Valorando tamao, densidad, forma nitidez y estabilidad en el
seguimiento. Es la lesin ms frecuentemente hallada.
Considerando la clnica y los datos mamogrficos, ecogrficos, etc. se establece la
probabilidad de malignidad del ndulo, pudiendo expresarla en las siguientes cuatro
categoras:
o
Benigno
o
Probablemente benigno

o
Probablemente maligno
o
Maligno
NDULO BENIGNO

NDULO PROBABLEMENTE BENIGNO

NDULO PROBABLEMENTE MALIGNO

NDULO MALIGNO

B.
Lesiones estrelladas (o de alteracin de la arquitectura): Representadas por reas
de distorsin de la arquitectura mamaria, de bordes irregulares y que adoptan una
morfologa radiada. Suele ser un signo temprano en el carcinoma de mama y es de difcil
interpretacin. Posee menos valor predictivo positivo que el ndulo o las
microcalcificaciones, por lo que se recomienda biopsia quirrgica en todos los casos
excepto en los que los antecedentes de traumatismo, ciruga previa o inflamacin permitan
optar por el seguimiento de la lesin. En estos casos siempre es conveniente haber
realizado una citologa con resultado negativo.
C.
Microcalcificaciones: Son hallazgos frecuentes y el anlisis de sus caractersticas
nos puede ayudar a diferenciar las benignas de las sospechosas y de las claramente
malignas.
Las benignas no requieren ms pruebas diagnsticas complementarias, las probablemente
benignas precisas de un seguimiento mamogrfico no inferior a los dos aos y en las
sugestivas de malignidad, la biopsia es preceptiva. Hay que analizar las siguientes
caractersticas:

Tamao: Las superiores a 2 mms. se clasifican de macrocalcificaciones y suelen ser


benignas. Por debajo de los 2 mms. se denominan microcalcificaciones y cuanto ms
pequeas y agrupadas ms sospechosas son de malignidad

Morfologa: Las calcificaciones malignas suelen ser heterogneas en forma y


tamao, puntiagudas, anguladas, irregulares, en "coma", ramificadas y con forma de punto y
raya. Las benignas suelen ser homogneas, redondas y en ocasiones anulares y de centro
claro.

Nmero: Se considera que cuando hay cinco o ms calcificaciones menores de 1 mm.


en un rea de 1x1 cm. de mamografa, existe sospecha de malignidad. Cuanto mayor es el n
de calcificaciones en esa rea, ms sospechosas son.

Distribucin: Las calcificaciones distribuidas de forma segmentaria, no al azar, son


sospechosas e indicativas de biopsia.

Variacin en el tiempo de las calcificaciones: Las calcificaciones malignas varan con


el tiempo. La estabilidad de las calcificaciones durante ao y medio - dos aos, se
consideran como benignas.

Calcificaciones asociadas a mama: Los carcinomas de mama calcifican en un 50 %.


Cuando hay calcificaciones internas en lesiones con signos de malignidad, aumentan las
posibilidades de malignidad. Se hallan en un 75% de los cnceres ocultos y suponen el 3047% de hallazgo aislado en los cnceres de mama. Son el primer marcador de cncer de
mama en las mujeres jvenes.
2: Signos secundarios:
1.
Engrosamiento de la piel.
2.
Permeabilidad linftica.
3.
Aumento de la vascularizacin.
4.
Afectacin linftica.
5.
Dilatacin ductal.

Bibliografa:
Apantaku LM. Breast cancer diagnosis and screening. [Internet]. American Family
Physician;
2000
[
acceso
28/6/2007].
Disponible
en:
http://www.aafp.org/afp/20000801/596.html
Barratt A, Howard K, Irwig L, Salkeld G and Houssami N. Model of outcomes of
screening mammography: information to support informed choices. BMJ 2005;330;936940.[Texto completo]
Brewer NT, Salz T, Lillie SE. Systematic review: the long-term effects of falsepositive mammograms. Ann Intern Med. 2007;146(7):502-10 [PubMed] [Texto completo]
Breast Imaging reporting and data system (BI-RADS). 2nd ed. Reston (VA):
American College of Radiology; 2007 [Resea]
Cantin J, Scart H, Levine M, Hugi M. Clinical practice guidelines for the care and
treatment of breast cancer: 13. Sentinel lymph node biopsy. Can. Med. Assoc. J. 2001; 165:
166 173 [Texto completo]
Dynamed. Breast cancer (female9) [Internet]. Ebsco industries ;2007 [acceso
3/7/22007]. Disponible en http://dynamed102.ebscohost.com/Detail.aspx?id=114433
Eberl MM, Fox ChH, Edge SB, Carter CA, Mahoney MC. BI-RADS Classification for
Management of abnormal Mammograms. J Am Board Fam Med 2006;19:161-4 [PubMed]
[Texto completo]
Fitzpatrick TB, Johnson RA, Wolff K, Suurmod D. Atlas color y sinopsis de
dermatologa clnica. Madrid: McGraw Hill Interamericana; 2001. p.494-5

39.- Se trata de femenino de 32 aos gestante, acude a ultrasonido de control. Se reporta


lo siguiente: Placenta marginal, cursando con embarazo de 37 SDG. Antecedentes: G=V P=
IV.
La complicacin esperada en esta paciente es:
a)

Hemorragia materna.

b)

Desprendimiento de placenta normoinserta.

c)

Parto pretrmino.

d)

Restriccin en el crecimiento intrauterino.

Placenta previa: La placenta en lugar de estar implantada en el cuerpo del tero por fuera
del orificio cervical interno, se localizaa por encima del orificio interno o cerca de l. En la
placenta marginal el borde la placenta est en el margen del orificio. La multiparidad y la
edad avanzada parecen favorecer la placenta previa. El hecho ms caracterstico de la
placenta previa es la hemorragia no dolorosa, la cual no suele aparecer hasta casi del final
de segundo trimestre o ms tarde. Complicaciones maternas: Hemorragia y muerte.

Pritchard, Williams Obstetricia, 3Edicin, Salvat editores, pg. 395-399

40.- Thrombocytopenia that is caused by increased platelet destruction is most closely


associated with which of the following conditions?

a)
b)
c)
d)

Combination chemotherapy
Acute leucemia
Systemic lupus erythematosus
Excessive ethanol intake

El diagnstico de LEG se hace mediante la identificacin de manifestaciones clnicas de


la enfermedad acompaadas de uno o ms autoanticuerpos tpicos (Tablas 2 y 3).

Tabla
Criterios para la Clasificacin del Lupus Eritematoso Generalizado

Eritema malar
Eritema discoide
Fotosensibilidad
Ulceras orales
Artritis
Serositis
a. pleuritis
b. pericarditis
Renal:
c. proteinuria persistente > 0,5 g/24 h o +++
d. cilin
Neurolgicos:
e. convulsiones
f. psicosis
Hematolgicos:
g. Anemia hemoltica con reticulocitosis
h. Leucocitopenia < 4.000 por dos veces
i. Linfocitopenia < 1.500 por dos veces
j. Trombocitopenia < 100.000
10. Inmunolgicos:
a) Anti-DNA elevado
b) Anti-Sm
c) Antifosfolpidos: Anticardiolipinas (IgG o IgM) o Anticoagulante
lpico o VDRL falso positivo por 6 meses.
11. Anticuerpos antinucleares (en ausencia de lupus por drogas)

41.- Femenino de 23 aos, G1 en trabajo de parto prematuro con embarazo de 30 semanas


de gestacin. A pesar del uso de agentes tocolticos, estos no han dado resultado. Se puede
inducir la maduracin pulmonar del producto por medio de:
a) Betametasona
b) Sulfato de magnesio
c) Hidroxiprogesterona
d) Clorprocana

La utilizacin de betametasona como inductor de madurez pulmonar fetal (IMPF) disminuye


la morbilidad neonatal relacionada con prematurez pero su efecto diabetgeno materno ha
sido poco estudiado.
La revisin Cochrane de un ciclo nico de corticosteroides se actualiz en 2006. En esta
actualizacin se incluyeron 21 estudios con un total de 3885 mujeres y 4269 lactantes.
En la revisin se descubri que la administracin de determinados corticosteroides a
mujeres con riesgo de tener un parto prematuro reduce considerable los riesgos de
complicaciones relacionadas con la prematurez como muerte fetal y neonatal combinada,
sndrome de dificultad respiratoria, hemorragia cerebroventricular, enterocolitis
necrotizante, infecciones sistmicas y retraso en el desarrollo durante la niez. Los
beneficios estaban presentes cuando el tratamiento se iniciaba entre las 26 y las 35
semanas de gestacin y en los nios que nacan entre 1 y 7 das despus de haber
comenzado el tratamiento; tambin se observaron beneficios en los subgrupos de mujeres
con rotura prematura de membranas y trastornos hipertensivos. La muerte fetal y neonatal
combinada se redujo incluso en neonatos que nacieron a menos de las 24 horas de haber
administrado la primera dosis.
No se demostraron beneficios cuando el tratamiento comenz antes de las 26 semanas de
gestacin, tampoco se observaron beneficios en los recin nacidos antes de las 26 semanas
de gestacin ni en los que nacieron despus de 7 das o ms de la administracin del
tratamiento.
En el caso de los neonatos que nacieron despus de las 36 semanas hubo una tendencia a
aumentar la muerte fetal y neonatal combinada.
Se observ una reduccin en el peso al nacer en los neonatos que nacieron entre los das 1 y
7, al igual que en los que nacieron ms de 7 das despus del primer tratamiento.

Un estudio que reclut mujeres con preeclampsia severa sugiri que las mujeres tratadas
tenan un mayor riesgo de sufrir diabetes gestacional.
La evidencia epidemiolgica y en animales sugiere que pueden haber efectos adversos a
largo plazo por la exposicin prenatal a los corticosteroides, entre ellos la alteracin de la
tolerancia a la glucosa y la hipertensin. Los estudios en animales tambin han sugerido que
afecta el crecimiento del cerebro.
1.
2.

3.

4.

5.
6.

7.

National Institute of Health (NIH). Consensus Conference; Effect of corticosteroide for fetal maturation on perinatal outcomes. JAMA 1994;(12):1-19.
White A, Marcucci G, Andrews E, Edwards K. Antenatal steroids and neonatal
outcomes in controlled clinical trials of surfactant replacement. Am J Obstet
Gynecol 1995; (173):286-90.
Klauss MH, Fanaroff AA, Martin RJ. Problemas respiratorios. En: Asistencia del
recin nacido de alto riesgo. 2 ed. La Habana: Editorial Cientfico-Tcnica,
1981:194.
Avery M, Frank N, Gribetz I. The inflationary force produced by pulmonary
vascular distention in excised lungs. The possible relation of this force to that
needed to inflatc the lungs at birth. J Clin Invest 1959;38:456.
Chu J, Clements J, Cotton E. Neonatal pulmonary ischemia. Pediatrics 1965;40:733.
Liggins GC, Howle RN. A controlled trial of antepartum glucocorticoid treatment
for prevention of respiratory distress syndrome in premature infants. Pediatrics
1972;50: 515-25.
Wright LL, Verter J, Younes N. Antenatal corticosteroids administration and
neonatal outcome in infants 501 to 1500 g. Am J Obstet Gynecol 1995; (173):263.

42.- En una consulta prenatal de rutina, una mujer de 28 aos de edad, G5 P4, con 28 SDG,
refiere que no ha sentido movimiento fetal durante los ltimos 2 das. Su embarazo ha sido
complicado debido a que padece hipertensin crnica, para lo cual se le recetaron tabletas
de alfa-metildopa 2 veces al da. Al examen, su FU es de 30cm, y las maniobras de Leopold
demuestran que el feto se encuentra en situacin transversa. TA:145/85mmHg. A la
colocacin de de Doppler no se encuentra latido cardiaco.. El paso ms apropiado a seguir
en el manejo de esta paciente es:

a)
b)
c)
d)

Realizar un test sin estrs


Amniocentesis
USG
Beta-HCG (cuantitativa)

Probable bito:
Sintomatologa y diagnstico Signos funcionales: No se perciben movimientos fetales por
12-24 horas. Disminucin o ausencia de sntomas y/o signos como nauseas vmito,
hipertensin, albuminuria) Paraclnicos: *ecografa: diagnstico precoz y exacto: Doppler.
*Radiologa: hay 3 signos: +deformacin del crneo +curvatura y torsin de la columna
+presencia de gas en el feto *lquido amnitico: puede estar meconiado, o sanguinolento
Signos locales: en los senos hay secrecin calostral, sangrado leve y oscuro por vagina, el

feto se vuelve blando a la palpacin, fetocardia (-), puede haber detencin y/o disminucin
de la altura uterina, bajo peso corporal, entre otros

Bibliografa: 1. OBSTETRICIA, Schwarcz R, editorial El ateneo, 2003. 2. Sociedad


espaola de ginecologa y obstetricia, junio 2002. 3. OBSTETRICIA CLNICA, Llaca V,
edicin 2000, captulo 24; Pg, 315-316.

43.- Se trata de femenino de 30 aos de edad, la cual inicia con hiperemesis gravdica de
difcil control, as como sangrado trasvaginal. Se realiza el diagnstico de mola hidatiforme.
El tratamiento inicial indicado en esta patologa es?

a)
b)
c)
d)

Histerectoma total abdominal.


Legrado por aspiracin.
Metotrexate y seguimientos radiogrficos.
Vigilancia expectante

La enfermedad trofoblstica gestacional agrupa a diferentes entidades interrelacionadas:


mola completa, generalmente diploide con origen cromosmico paterno, mola parcial
generalmente triploide, tumor trofoblstico del lecho placentario y coriocarcinoma, con
tendencias variables a la invasin local y a las metstasis, cuyo denominador comn es la
hipersecrecin de hCG. El coriocarcinoma es diploide y proviene de ambos progenitores,
excluyendo probablemente su origen directo en la mola completa. El tumor trofoblstico
del lecho placentario est constituido por trofoblasto mononuclear intermedio no
conteniendo vellosidades corinicas e inmunohistoquimicamente caracterizado por expresar
muchas de sus clulas hPL y unas pocas hCG
Tratamiento
Hay que tratar las complicaciones como la hipermesis, anemia, hipertensin, y alteraciones
electrolticas, coagulopatas, alteraciones cardio-respiratorias y preeclampsia, procediendo
a evacuar la mola lo antes posible, con lo que se producirn menos malignizaciones.
La evacuacin del contenido uterino se realiza mediante dilatacin, y legrado por aspiracin.
Adems se pauta profilaxis antibitica y oxitcicos.3
La histerectoma, con la mola en su interior, est indicada en pacientes de edad superior a
40 aos o en mujeres con ms de tres hijos, ya que en ambos grupos se ha demostrado una
mayor incidencia de malignizacin.

Tras la ciruga, se mide la concentracin de gonadotropina corinica humana para


determinar si la extirpacin ha sido completa. Si es as, el valor de esta hormona vuelve a la
normalidad, en unas 8 semanas, y se mantiene en esos valores. Si una mujer a la que se le ha
extirpado una mola queda embarazada, es difcil interpretar un valor alto de gonadotropina
corinica humana, porque podra estar causado tanto por el embarazo como por una parte
de la mola que no se ha extirpado. En consecuencia, a las mujeres a las que se les ha
extirpado una mola se les recomienda no quedar embarazadas durante un ao. Las molas
hidatiformes benignas no necesitan quimioterapia, pero las malignas s. Los frmacos que se
usan para este tratamiento son el metotrexato, la dactinomicina o una combinacin de
ambos.
Silverman L,Romero Zambrano F, Saldao S. Enfermedad molar. Diagnstico, tratamiento y
seguimiento, 1987.
4- Puertas A, Lpez Fernandez J et al. Enfermedad trofoblstica. Casustica del Hospital
Virgen de las Nieves de
Granada. Cln Invest Gin Obs 1993; 20: 98-103.
5- Enfermedad trofoblstica gestacional. Propuesta Normativa Perinatolgica y
Ginecolgica de Alto Riesgo.
Ministerio de Asuntos Sociales. Tucumn. 1996-1997; 19:171-183.
6- Jones. Enfermedad Trofoblstica Gestacional: qu hemos aprendido en la ltima dcada.
Am J Gynecol Obstet
1990;162: 1286-1292.
7- Resmen del simposio Enfermedad Troblstica Gestacional del Segundo Congreso
Nacional de AGORA, 1990.
8- Gonzalez Merlo et al. Protocolos de diagnstico y tratamiento en Obstetricia y
Ginecologa, Barcelona: editorial
Salvat, 7:35-45.

44.- Femenino de 31 aos se enva de alta con diagnstico de enfermedad inflamatoria


plvica, regresa a los 15 das con datos de dolor abdominal intenso, signo de blumberg
positivo, datos de irritacin peritoneal y mal estado general. Al tacto vaginal demuestra
fondos de saco vaginales abombados y dolorosos. SV : temperatura de 38.5 c, Fc 88 x,
Fr 22 x t/a 110/70 mmhg . El diagnstico ms probable es:

a) Endometritis
b) Hidrosalpinx
c) Absceso tubo-ovrico
d) Ooforitis aguda

La enfermedad inflamatoria plvica (EIPA) es un sndrome clnico caracterizado por la


infeccin del tracto genital superior que se produce casi siempre por va ascendente desde
el cuello uterino. El impacto que la infeccin plvica ejerce sobre la condicin fsica de la
mujer va desde la infeccin asintomtica o silente a una mayor morbilidad que en algunos
casos puede llegar hasta la muerte. Incluye una variedad de condiciones inflamatorias que
afectan el tracto genital superior. Los Centros de Control de Enfermedades (C .D. E.) la
definen como un sndrome agudo debido al ascenso de microorganismos de la vagina o el

cuello uterino al endometrio, trompas uterinas y en ocasiones a las estructuras vecinas


(ovarios, peritoneo y cavidad pelvianas).
En el momento actual se incluyen como principales agentes etiolgicos de la E.I.P.A la
Neisseria gonorrhedae, las clamydias y los anaerobios. Otros microorganismos como los
microplasmas y los actinomices se estn observando con frecuencia.
La presencia de anaerobios as como de bacterias aerobias puede deberse a un fenmeno
de sobre infeccin secundaria. Hay autores que sealan que excepto para el gonococo y la
Clamydia trachormatis, no existen datos suficientes que permitan afirmar que otras
bacterias tengan un papel primario en la infeccin de unas trompas sanas. Una vez alterada
la integridad anatmica de la trompa, se producira la infeccin mixta o poli microbiana.

El absceso tubo ovrico es una formacin inflamatoria que compromete el ovario y la


trompa y puede ser uni o bilateral. En este absceso las estructuras comprometidas
estn infectadas y contienen pus. Este proceso inflamatorio es secundario a un proceso
infeccioso de la pelvis, habitualmente producido por grmenes muy patgenos, que llegan
al tracto genital a travs de una relacin sexual, es decir corresponde a una complicacin
severa de una enfermedad de transmisin sexual.
Se caracteriza por aumento de volumen del ovario y trompa, los que se encuentran
adheridos entre s producto de esta infeccin, adems el proceso infeccioso se extiende
habitualmente a otras estructuras y rganos pelvianos, los que estn muy inflamados y
adheridos formando lo que se denomina plastrn.
El tratamiento se inicia mdicamente con antibiticos de amplio espectro para cubrir
tanto grmenes aerbicos como anaerbicos, generalmente requiere de hospitalizacin
para iniciar una terapia agresiva endovenosa con los antibiticos y para monitorizar
adecuadamente a la paciente, pues la infeccin produce compromiso del estado general
pudiendo llegar hasta la sepsis generalizada.
Diagnstico
El cuadro clnico se sospecha cunado una paciente consulta por dolor abdominal intenso,
progresivo, fiebre y compromiso de su estado general, habitualmente en el examen se
encuentra un distensin abdominal y a la palpacin del abdomen hay dolor, y signo de
blumberg positivo o irritacin peritoneal. El Tacto vaginal demuestra fondos de saco
vaginales abombados y dolorosos y habitualmente el cuello del tero lateralizado y
doloroso a la movilizacin si el compromiso es unilateral, adems de palpar una masa para
uterina irregular y sensible.

BIBLIOGRAFA.
1.
2.
3.
4.
5.
6.
7.
8.

Botella Llusi,J.Clavero Nez,J.A:Tratado de Ginecologa.14 edicin. Ed.Diaz de


Santos.pg 833-844. Madrid,1993.
Brunham,R,C:Infectionin woman and ectopic pregnancy. Am J Obstet
Gynecol.67:722,1999.
Cates,W,Wasserheit,J,N:Genital Infection Epidemiology and sequeale.Am J Obstet
Gynecol 164-1771,1998.
Keit,L,G; Berger,G,S:On the causation of pelvic inflammatory disease. Am J Obstet
Gynecol 149-215,2002.
Muller,B,R;Allen,J,et al.Pelvic Inflamatory disease after histerosalpingography.Brit
J Obstet Gynecol,91-1181,1999.
Toth,A,O Leary,W,M: Evidence of microbial transfer by espermatozoo.Am J O
bstet Gynecol 59-556,2003.
Varela,R,et col:Abceso Tuboovrico,Acta mdica Portuguesa ,p:537-542,Vol.
8,2001.
Sopper,D,E:Pelvic Inflamatory disease.Infections disease.Clin of North America
.831-840,vol 8;n 4.Dec 2003.

45.- Se trata de paciente femenino de 68 aos, refiere intenso prurito vulvar y sensacin
quemante, actualmente presenta disuria. Al examen, el introito vaginal se encuentra
estentico, con fisuras superficiales. Sin otro antecedente patolgico. De los siguientes el
tratamiento apropiado para la paciente es:

a)
b)
c)
d)

5-fluoracilo
Estrgeno tpico
Testosterona tpica
Corticoesteroides fluorados

Vulvovaginitis atrfica

El hipoestrogenismo conduce a atrofia de la vagina y el vestbulo vulvar, que los hace


fcilmente irritables y susceptibles a infecciones secundarias. Las pacientes refieren
sensacin de quemadura, prurito, disuria, hipersensibilidad y dispareunia. Puede
encontrarse al examen fsico atrofia, fisuras superficiales, y un flujo vaginal acuoso1. Hay
disminucin del tamao del introito2, prdida de la rugosidad y la vagina toma una apariencia
lisa y brillante.
Los hallazgos histolgicos revelan un epitelio vaginal delgado, disminucin de los lechos
capilares, y la citologa muestra, a medida que la atrofia progresa, aumento de las clulas
basales y disminucin o ausencia de las clulas superficiales2.
Se aconseja evitar el uso de jabones y dems irritantes de la piel. Se pueden utilizar
lubricantes simultneamente con los estrgenos o como terapia nica, si hay alguna
contraindicacin a las hormonas.
El tratamiento con estrgenos por va sistmica o transvaginal mejora y restaura los signos
y sntomas, y una a dos semanas despus de iniciar el tratamiento los cambios de atrofia
empiezan a mejorar rpidamente, se reduce el pH y se induce maduracin vaginal y de la
mucosa uretral, reduciendo la frecuencia de las infecciones urinarias3. La dosis y va de
administracin debe ser debidamente individualizada4. Contraindicaciones al tratamiento
con estrgenos, incluyen: la presencia de tumores estrgenosensibles, falla heptica
terminal y antecedentes de tromboembolizacin relacionada con ellos.
Menopausia y Piel. Parte II:
Manifestaciones clnicas
dermatolgicas durante la menopausia
MARA ISABEL BARONA C. Docente adjunto. Dermatloga Universidad del Valle-Cali.

46.- Mujer de 26 aos, es atendida en consulta en la clnica de displasias por papanicolaou


con lesin NIC I. Antecedentes: menarca 14 aos, ritmo 30x5 eumenorreica, inicio de vida
sexual a los 15 aos, 2 parejas sexuales, mtodo de planificacin familiar oclusin tubaria
bilateral, gestas 3 partos 3, crvix con lesin acetoblanca con extensin lineal de 2 cm.
Para confirmar el diagnstico se debe realizar:

a)
b)
c)
d)

Crioterapia de lesin.
Biopsia de la lesin.
Captura de hbridos.
Repetir colposcopa

9.5.2 Las pacientes a quienes se les realiz citologa cervical, cuyo resultado es LEIBG
(infeccin por VPH, displasia leve o NIC 1); LEIAG (displasia moderada y grave o NIC 2 y 3)
o cncer deben enviarse a una clnica de colposcopa, para realizar estudio colposcpico.
9.5.3 Si el resultado de la citologa es LEIBG, la colposcopa es satisfactoria y sin
evidencia de LEIBG, se realizar control citolgico en un ao (Apndice Normativo A)

9.5.4 Si la citologa es de LEIBG, la colposcopa es satisfactoria y existe evidencia de


lesin, se debe tomar una biopsia dirigida.
9.5.4.1 Si la biopsia dirigida es negativa, se realizar nueva colposcopa para verificar el
diagnstico y en caso necesario, tomar nueva biopsia dirigida y revalorar.
9.5.4.2 Si la biopsia dirigida es reportada como LEIBG se podr dar tratamiento
conservador: criociruga, electrociruga o laserterapia (slo si cumple con las condiciones
referidas en el Apndice 1) o se podr mantener a la paciente en vigilancia en la clnica de
colposcopa, con colposcopa y estudio citolgico cada seis meses, durante 24 meses.
Jueves 31 de mayo de 2007 DIARIO OFICIAL (Primera Seccin)
9.5.4.3 Si la biopsia dirigida es reportada como LEIAG (Lesin Intraepitelial Escamosa de
Alto Grado) se realizar tratamiento conservador (electrociruga o laserterapia). En las
mujeres posmenopusicas, dependiendo de las condiciones anatmicas del crvix, se
realizar tratamiento conservador en la clnica de colposcopa o tratamiento quirrgico
(histerectoma extrafascial) en el servicio que corresponda.
9.5.4.4 Si la biopsia dirigida reporta cncer microinvasor o invasor, la paciente se
transferir a un Servicio o Centro Oncolgico para su tratamiento correspondiente.
9.5.4.5 Si la citologa reporta LEIBG y la colposcopa es no satisfactoria, se tomar
cepillado endocervical (Apndice Normativo A)
9.6 En caso de colposcopa no satisfactoria, negativa a LEIBG y con cepillado endocervical
negativo, se continuar su control en la clnica de colposcopa en seis meses, con colposcopa
y citologa.
9.6.1.1 Si el cepillado endocervical reporta LEIBG se tratar a la paciente como LEIAG,
con mtodos conservadores escisionales.

Jueves 31 de mayo de 2007 DIARIO OFICIAL (Primera Seccin)


Modificacin a la Norma Oficial Mexicana NOM-014-SSA2-1994, Para la prevencin,
deteccin, diagnstico, tratamiento, control y vigilancia epidemiolgica del cncer
crvico uterino.
Al margen un sello con el Escudo Nacional, que dice: Estados Unidos Mexicanos.- Secretara
de Salud.
MODIFICACION A LA NORMA OFICIAL MEXICANA NOM-014-SSA2-1994, PARA LA
PREVENCION,
DETECCION,
DIAGNOSTICO,
TRATAMIENTO,
CONTROL
Y
VIGILANCIA
EPIDEMIOLOGICA DEL CANCER CERVICO UTERINO.

7.3 El resultado del estudio citolgico es descriptivo y debe ser informado de la siguiente
manera:
a.- Negativo a cncer.
b.- Negativo con proceso inflamatorio.
c.- Displasia leve (NIC 1).
d.- Displasia moderada (NIC 2).

e.- Displasia grave (NIC 3).


f.- Cncer del cuello del tero in situ (NIC 3).
g.- Cncer microinvasor e invasor.
h.- Adenocarcinoma.
i.- Maligno no especificado.

47.- Femenino de 32 aos segunda gesta a trmino sin anormalidades en el transcurso de


ste. Inicia trabajo de parto de forma espontnea, con evolucin normal hasta que se
rompe la bolsa, con una dilatacin de 4 cm. A partir de entonces, comienza con hemorragia
de sangre roja, en moderada cantidad y aparecen signos de sufrimiento fetal agudo. El
estado general de la mujer es bueno y la dinmica uterina es normal. Este cuadro
corresponde a:
a) Abruptio placentae.
b) Rotura uterina.
c) Placenta previa central
d) Rotura de vasa previa.

La vasa previa es una condicin de alto riesgo obsttrico en la cual vasos fetales o
placentarios cruzan el segmento uterino por debajo de presentacin.
Estos vasos estn desprotegidos de gelatina de Wharton o tejido placentario, lo que los
hace altamente vulnerables y susceptibles de ruptura o laceraciones en cualquier perodo
del embarazo, principalmente en el momento del parto. Tambin es frecuente la compresin
de estos vasos, especialmente durante el tercer trimestre de la gestacin, lo que puede
condicionar asfixia y muerte fetal.

Esta condicin ocurre como resultado de que vasos velamentosos cruzan por el segmento
uterino debido a una insercin velamentosa del cordn, situacin en la cual el cordn
umbilical se inserta en las membranas ovulares en vez del tejido placentario (vasa previa
tipo I), o por el cruce de vasos fetalesentre uno o ms lbulos accesorios de la placenta
(vasa previa tipo II) (Figura 1).

Figura 1. A, Vasa previa tipo I, debida a insercin velamentosa.


B, Vasa previa tipo II, debida a cotiledones aberrrantes. Reproducido de Daly-Jones y cols.
Ultrasound 2008.

Vasa previa se puede presentar si existe alguna (o ninguna) de las siguientes condiciones:
placenta baja (que puede ser causa de abortos previos seguidos por legrado o por
operaciones uterinas, que provocan cicatrices en el tero), placenta bilobada o de lbulo
succensuriado, embarazos resultado de fertilizacin in vitro, o embarazos mltiples (5-6).
El sangrado por vasa previa no es doloroso. Otros sangrados por complicaciones o por
nacimiento no necesariamente son sin dolor.

Referencias:
1. Oyalese Y, Smulian JC. Placenta previa, placenta acreta, and vasa previa. Obstet Gynecol
2006; 107: 927-941.
2. Oyalesse KO, Turner M, Less C, Campbell S. Vasa previa: an avoidable obstetric tragedy.
Obstet Gynecol Surv 1999; 54: 138-145.
3. Seplveda W, Sebire NJ, Harris R, Nyberg DA. The placenta, umbilical cord, and
membranas. In Diagnostic Imaging of Fetal Anomalies, Nyberg DA, MaGahan JP, Pretorius
DH, Pilu G (eds). Philadelphia, PA: Lippicont Williams & Wilkins 2003, 85-132.
4. Daly-Jones E, John A, Leahy A, McKenna C, Sepulveda W. Vasa praevia; a preventable
tragedy. Ultrasound 2006; 16: 8-14.
5. Derbala Y, Grochal F, Jeanty P. Vasa previa. J Prenat Med 2007; 1: 2-13.
6. Fung TY, Lau TK. Poor perinatal outcome associated with vasa previa. It is preventable?
A report of three cases and review of the literature. Ultrasound Obstet Gynecol 1998; 12:
430-433.
7. Robert JA, Sepulveda W. Fetal exsanguination from ruptured vasa previa: still a
catastrophic event in modern obstetrics. J Obstet Gynaecol 2003; 23: 574.
8. Cordero DR, Helfgott AW, Landy HJ, et al. A non-hemorrhagic manifestation of vasa
previa: a clinicopathologic case report. Obstet Gynecol 1993; 82: 698-700.
9. Schachter M, Tovbin Y, Arieli S, et al. In vitro fertilization as a risk factor for vasa
previa. Fertil Steril 2002; 78: 642-643.

48.- Se trata de femenino de 26 aos, que cursa con 14 semanas de gestacin, acude al
servicio por presentar tenesmo vesical, disuria y escalofro. Por su estado actual, cul de
los siguientes antimicrobianos recomendara?
a)
b)
c)
d)

Ampicilina
Trimetoprim sulfametoxazol
Tetraciclina
Levofloxacina

El Metronidazol no ha mostrado efectos txicos en humanos, pero es teratognico en


modelo animal. Las tetraciclinas ocasionan coloracin anormal de los dientes,
hepatotoxicidad y alteracin en el desarrollo de huesos. Las sulfas podran tener un efecto
deletreo en el primer trimestre dada su actividad como antimetabolitos, y en los ltimos
meses pueden favorecer kernicterus en el recin nacido si es que tiene alteraciones
metablicas que favorezcan anemia hemoltica. Las quinolonas se han asociado a
malformaciones seas en modelos animales, y se recomienda evitarlas si existen mejores
opciones.
Kasper DL, Braunwald E, Fauci AS, Hauser SL, Longo DL, Jameson JL. Harrisons
Principles of Internal Medicine. McGraw Hill. 16 Ed. 789-806 pp.

49.- Mujer de 26 aos, G-1, que cursa con embarazo de 37 semanas de gestacin, presenta
prdida del estado de alerta posterior a crisis convulsivas tnico-clnicas, signos vitales
con T-A 170.120mmhg Fc 95x, reflejos osteotendinosos aumentados, se aprecia una Fc
fetal de 132x y edema importante de miembros inferiores, no se aprecian datos de
trabajo de parto ni modificaciones cervicales. El diagnstico ms probable es:

a) Eclampsia
b) Pre eclampsia severa
c) Crisis epileptica de gran mal
d) Hipertensin inducida por el embarazo

CUADRO 1. DIAGNSTICO*
Preeclampsia Leve: Se presenta despus de la semana 20 de gestacin, durante el parto, o
en las
primeras 6 semanas despus de ste
Presin sistlica a 140 mm Hg o presin diastlica 90 mm Hg
Proteinuria a 300 mg / orina de 24 hrs o su equivalente en tira reactiva
Preeclampsia Severa: Se presenta despus de la semana 20 de gestacin, durante el
parto, o en las primeras 6 semanas despus de ste
Presin sistlica a 160 mm Hg o presin diastlica 110 mm Hg

Proteinuria a 2 gr en orina de 24 horas o su equivalente en tira reactiva


Creatinina srica > a 1.2 mg/dl
Trombocitopenia 150 000 cel/mm3
Incremento de la deshidrogenasa lctica a 600 UI
Elevacin al doble de los valores de TGO/AST o TGP/ALT
Cefalea, alteraciones visuales o auditivas
Epigastralgia
Oliguria a 500 ml en 24 horas 7
Edema agudo de pulmn
Dolor en hipocondrio derecho
Restriccin en el crecimiento intrauterino
Oligohidramnios
Eclampsia Preeclampsia mas convulsiones sin otra causa. Se presenta despus de la semana
20 de gestacin, durante el parto, o en las primeras 6 semanas despus de ste.
Sndrome de HELLP Criterios para establecer el diagnstico del sndrome de HELLP:
Plaquetas < 100 000/mm3 TGO/AST 70U/L DHL 600U/LBilirrubina total > 1.2 mg/dl
Se presenta despus de la semana 20 de gestacin, durante el parto, o en las primeras 6
semanas despus de ste.
Hipertensin Crnica: Se diagnostica cuando existe hipertensin arterial a 140/90 mm
Hg antes de la semana 20 de gestacin o si persiste despus de doce semanas posteriores
al parto.
Las pacientes con hipertensin crnica deben ser evaluadas antes del embarazo para
determinar la severidad de la hipertensin y facilitar la planeacin de un embarazo
mediante el cambio de medicamentos y de hbitos higinicos y dietticos para evitar
complicaciones.
Hipertensin Gestacional: Presencia de hipertensin arterial a 140/90 mm Hg despus
de la semana 20 de gestacin y se mantiene hasta las doce semanas despus del parto
Ausencia de proteinuria
Presencia o no de cefalea, acfenos y fosfenos
Despus de 12 semanas de la interrupcin del embarazo se revalorar la presencia de
hipertensin, si contina, se reclasifica como hipertensin crnica: es un diagnstico
retrospectivo. 8 Si no hay, se clasifica como hipertensin transitoria.

Bibliografa:
1. Aagaard-Tillery KM, Belfort MA. Eclampsia: morbidity, mortality, and management. Clin
Obstet Gynecol 48:12-23, 2005.
2. Atallah AN, Hofmeyr GJ, Duley L. Calcium supplementation during pregnancy for
preventing hypertensive disorders and related problems. Cochrane Database Syst Rev
1:CD001059, 2001.
3. Barton JR, Sibai BM. Diagnosis and management of hemolysis, elevated liver enzymes,
and low platelets syndrome. Clin Perinatol 31:807-33, 2004.
4. Baxter JK, Weinstein L. HELLP syndrome: the state of the art. Obstet Gynecol Surv
59:838-45, 2004.
5. Cetin A. Eclampsia. In Mohler III ER, Townsend RR. Advanced therapy in hypertension
and vascular disease. Ontario: B.C. Decker Inc. pp. 407-15, 2006.
6. Cetin A. Hemolysis, elevated liver enzymes, and low platelets (HELLP). In Mohler III ER,
Townsend RR. Advanced therapy in hypertension and vascular disease. Ontario: B.C. Decker
Inc. pp. 416-20, 2006.
7. Chappell LC, Seed PT, Briley AL, Kelly FJ, Lee R, Hunt BJ, Parmar K, Bewley SJ, Shennan

AH, Steer PJ, Poston L. Effect of antioxidants on the occurrence of pre-eclampsia in


women at increased risk: a randomised trial. Lancet 354:810-16, 1999.

50.- En una paciente que cursa con 12 semanas de gestacin que lleva tres das sangrando
por genitales, con nuseas continuas y constantes, tero mayor aumentado de tamao y
valores de beta HCG muy elevados, debemos sospechar de:

a) Amenaza de aborto.
b) Aborto diferido.
c) Mola hidatdica.
d) Amenaza de aborto en un tero con miomas.

La enfermedad trofoblstica gestacional agrupa a diferentes entidades interrelacionadas:


mola completa, generalmente diploide con origen cromosmico paterno, mola parcial
generalmente triploide, tumor trofoblstico del lecho placentario y coriocarcinoma, con
tendencias variables a la invasin local y a las metstasis, cuyo denominador comn es la
hipersecrecin de hCG. El coriocarcinoma es diploide y proviene de ambos progenitores,
excluyendo probablemente su origen directo en la mola completa. El tumor trofoblstico
del lecho placentario est constituido por trofoblasto mononuclear intermedio no
conteniendo vellosidades corinicas e inmunohistoquimicamente caracterizado por expresar
muchas de sus clulas hPL y unas pocas hCG.

Cuadro clnico

Tras un periodo de amenorrea, y a partir del segundo mes, hay un aumento de los sntomas
subjetivos del embarazo, sobre todo nuseas y vmitos o hipermesis en un 30%. Hay
metrorragias irregulares en el 96% de los casos, en principio de escasa cantidad, pero que
se va incrementando. Dicha hemorragia no es continua sino que se repite cada dos o tres
das, de color roja o negruzca, que se produce por la ruptura de vasos maternos al
separarse las vesculas de la decidua.
Ocasionalmente, hay expulsin de restos molares, que lo refiere la paciente como expulsin
de vesculas en el 11 % de los casos y que es patognomnico pero aparece tardamente.
Tambin puede presentar mal estado general, dolor difuso en hipogastrio y anemia. Esta
ltima en relacin con las metrorragias.
Pueden haber signos y sntomas de hipertiroidismo, como taquicardia, sudoracin, y
temblores, en el 7%, y es debido a que la fraccin -hCG es similar a la hormona TSH.
Existen signos de preeclampsia o hipertensin gestacional del primer trimestre hasta en el
50% de los casos, y signos de insuficiencia respiratoria aguda en el 2% por embolismo
pulmonar de las clulas trofoblsticas, o por la asociacin entre hipertiroidismo e
hipertensin arterial. Como complicaciones pueden aparecer coagulopatas y metstasis.
A la exploracin el tero esta aumentado de tamao, en el 50% de los casos, por encima de
lo que correspondera a la edad gestacional,3 de consistencia blanda, sin signos de actividad
fetal a partir de la semana 12 y siempre que se trate de una mola total. Tambin puede que
la paciente presente un tamao uterino menor al esperado para la edad gestacional.3 El
cuello est cerrado, con metrorragia en cantidad variable, y raramente se observa la
expulsin de vesculas. Pueden haber quistes teca-lutenicos bilaterales en ovario en cerca
del 20% de los casos,3 debido al estmulo de la -hCG.
La enfermedad trofoblstica maligna va a cursar con metrorragias por lo general intensas,
hay una elevacin de la -hCG y en la ecografa se objetiva la cavidad uterina con signos de
ocupacin atpica.

Diagnstico
Por la clnica, y pruebas complementarias como la determinacin de la -hCG y la ecografa.
La determinacin de la -hCG se basa en que el trofoblasto produce la hormona
gonadotropina corinica, presentando cifras elevadas, y su cuantificacin va a servir para
diagnstico, valorar el pronstico, y el seguimiento postratamiento. La ecografa revela un
tero aumentado de tamao que no corresponde con la amenorrea, con ecos en su interior,
puntiformes que corresponderan a las vesculas y que asemejan copos de nieve o panal de
abeja. No se aprecia saco gestacional ni estructuras fetales y, en ambos ovarios se
aprecian quistes teca-lutenicos como formaciones ovricas redondas, econegativas, con
mltiples tabiques en su interior

BIBLIOGRAFA
1.

Mazur MT, Kurman RJ. Gestational trophoblastic disease and related lesions. En:
Kurman RJ editor. Blaunsteins pathology of the female genital tract. 4th ed. New
York: Springer-Verlag. 1994, p. 1049-93.
2. Kurman RJ, Young RH, Norris HJ, Main CS, Lawrence WD, Scully RE.
Immunocytochemical localization of placental lactogen and chorionic gonadotrophin
in the normal placenta and trophoblastic tumors, with emphasis on intermediate
trophoblast and the placental site trophoblastic tumor. Int J Gynecol Pathol 1984;
3: 101-21.
3. Berkowitz RS, Golstein DP. The management of molar pregnancy and gestational
trophoblastic tumours. En Knapp RC, Berkowitz RS, editores. Gynecologic Oncology,
2nd ed. New York: Mc Graw-Hill 1992, p. 328-38.

4. De Agustn P, Ruiz A, Lpez F, Contreras F. Patologa de la enfermedad


trofoblstica. Simposio Enfermedad Trofoblstica 1972; 79-98.
5. Salem S. Ultrasound diagnosis of trophoblastic disease. En: Sanders RC, James
AE(Jr) editores. Ultrasonography in Obstetrics and Gynaecology. New York:
Appleton-Century Crofts: 1977; p. 255-66.
6. Silverberg SG, Kurman RJ. Tumors of the uterine corpus and gestational
trophoblastic disease. En: Rosai J, Sobin LJ, editores. Atlas of tumor pathology:
tumors of the uterine corpus and gestational trophoblastic disease, fasc. 3, ser. 3.
Washington DC: Armed Forces Institute of Pathology; 1992, p. 219-85.
7. Szulman AE, Surti U The syndromes of hydatiform mole II. Morphologic evolution
of the complete and partial mole. Am J Obstet Gynecol. 1978; 132: 20-7.

51.- Mujer de 23 aos diagnosticada de E. Ectpico a nivel ampular, con saco gestacional
menor de 3 cm., sin actividad cardaca embrionaria, asintomtica y con niveles de B-HCG
menores para su edad gestacional. El tratamiento ms indicado es:
a) Salpingocentesis
b) Reseccin segmentaria
c) Tratamiento mdico con Metrotexate.
d) Histerectoma total con doble anexectoma.

TRATAMIENTO







METOTREXATE
cido 4 amino 10 metil folnico, antagonista del cido flico
Glucosa hiperosmolar
Prostaglandina F2a
Actinomicina D
Mifespristona

TRATAMIENTO CON METOTREXATO


 Mayor xito:




Embarazo menor a 6 SDG


Masa tubaria menor de 3.5cm
Feto sin latido cardaco

PROTOCOLO CON DOSIS NICA DE METOTREXATE


Da 0
Da 1
Da 4
Da 7

hGC, Biometra hemtica, transaminasas, creatinina, grupo


hGC
hGC
hGC

sanguneo






Disminucin de hCG <15%. Segunda dosis


Si la hCG declina seguirla semanalmente
Si la hCG est en meseta o en ascenso, segunda dosis
El raspado endometrial se realiza slo en pacientes con hCG <2000 mIU/mL al
momento de comenzar el tratamiento

Graczykowski JW, Mishell DR. Methotrexate prophylaxis for persistent ectopic pregnancy
after conservative treatment by salpingostomy. Obstet Gynecol.

52.- Al realizar el diagnstico de hirsutismo asociado a ovario poliqustico, usted elige el


siguiente frmaco para su tratamiento por ser el ms adecuado:

a) Clomifeno
b) Estrgenos
c) Corticoide
d) Acetato de ciproterona

El acetato de ciproterona parece ser ms efectivo que otros frmacos para el hirsutismo
en mujeres causado por la produccin ovrica excesiva de andrgenos
Una de las causas de hirsutismo (crecimiento piloso excesivo) en mujeres es la
hiperproduccin de andrgenos a partir del ovario. Varios frmacos pueden utilizarse para
contrarrestar los efectos del andrgeno. El acetato de ciproterona es un frmaco
antiandrognico. Los efectos adversos informados con su uso fueron aumento de peso,
depresin, fatiga, sntomas mamarios y disfuncin sexual. La revisin de los ensayos
encontr que el acetato de ciproterona parece ejercer un efecto en el hirsutismo similar a
otros frmacos utilizados para el tratamiento del hirsutismo por exceso de andrgenos. No
existen pruebas suficientes para comparar los efectos adversos de las opciones de
tratamiento.

Van der Spuy ZM, le Roux PA. Acetato de ciproterona para el hirsutismo (Revisin
Cochrane traducida). En: La Biblioteca Cochrane Plus, nmero 4, 2007. Oxford, Update
Software Ltd. Disponible en: http://www.update-software.com. (Traducida de The
Cochrane Library, 2007 Issue 4. Chichester, UK: John Wiley & Sons, Ltd.).

53.- Femenino de 25 aos la cual presenta un ndulo mamario palpable de aparicin brusca.
La ecografa revela un ndulo anecognico, de limites muy precisos, morfologa regular y
refuerzo posterior, nico de 3.5 cms. de dimetro. El diagnstico ms probable es :
a)
b)
c)
d)

Cncer.
Quiste
Displasia fibrosa.
Fibroadenoma.

Quistes. Los quistes mamarios son fciles de detectar con la ecosonografa. Pueden ser
lesiones nicas o mltiples que se observan como imgenes redondeadas, anecognicas, de
paredes delgadas, contornos bien definidos, con importante reforzamiento acstico
posterior y sombras laterales delgadas. Pueden presentar septos intraqusticos y, en
ocasiones, se pueden observar ecos internos que sugieren detritus celulares o proceso
inflamatorio. Se debe descartar la presencia de lesiones intraqusticas o la coexistencia de
otras alteraciones benignas o malignas. En caso de ser sintomticos, el tratamiento
adecuado es la puncin y aspiracin de la lesin con aguja guiada por palpacin o
ecosonografa de acuerdo con el tamao, profundidad y caractersticas del contenido. El
uso del ultrasonido garantiza el vaciamiento completo.

REFERENCIAS:
1.

Barth V, Prechtel K. Mama normal. En: Barth V, Prechtel K, editores. Atlas de


patologa de la glndula mamaria.
2da ed. Madrid: Editorial Mdica Panamericana, 1991.
2. Bush H, McCredie A. Carcinoma of the breast during pregnancy and lactation. In:
Allen HH, Nisker JA. Cancer in pregnancy. New York: Futura Publishing Co. Inc.,
1986.
3. Byrd BF, Bayer DS, Robertson JC, Stephenson JE Jr.
4. Treatment of breast tumor associated with pregnancy and lactation. Ann Surg.
1962; 155:940-7.

54.- Es el caso de paciente femenino de 17 aos la cual presenta amenorrea acompaada


de profundas alteraciones del olfato. De los siguientes diagnsticos el diagnstico ms
probable es:
a) Sndrome de amenorrea-galactorrea.
b) Amenorrea de causa uterina.
c) Sndrome de ovario poliqustico.
d) Amenorrea por alteracin hipotalmica.

AMENORREA HIPOTALAMICA. (Hipogonadotrpica)


Psicgena. (Stress emocional)
Anorexia nerviosa. (Deficiencia nutricional)
Ejercicio excesivo. (Carrera de fondo, natacin, gimnasia, ballet)
Frmacos.
(Fenotiazina,
reserpina,
bloqueadores
ganglionares,
anticonceptivos)
Pseudocisis.
Sndrome de Kallman (Deficiencia selectiva de gonadotropinas y anosmia).
El ejemplo clsico de la alteracin hipotalmica que lleva a desrdenes del ciclo menstrual
es el Sndrome de Kallman.

Lectura Recomendada:
Etiopatogenia de la amenorrea hipotalmica funcional Interaccin de las respuestas
hormonales del Sistema Nervioso Central y Neuropptidos Perifricos
Revista Argentina de Endocrinologa y Metabolismo
Copyright 2008 por la Sociedad Argentina de Endocrinologa y Metabolismo
Vol 45 No. 2

55.- Femenino de 22 aos de edad, acude al servicio de urgencias por referir malestar
general, fosfenos, nusea y vmito. Antecedentes: cursa con embarazo de 36.5 SDG, G2,
A1, C0. Exploracin fsica: TA 185/110 mmHg, FC 120 lpm, FR 35 x, T37.9, alerta,
inquieta, aprecia ictericia, cardiopulmonar sin compromiso, abdomen globoso a expensas de
tero grvido, se detecta PUVI, longitudinal, ceflico, dorso a la derecha, FCF 142 x, al
tacto genital crvix, central, formado, cerrado, extraccin de guantes sin evidencia de
prdidas genitales, extremidades inferiores edema +++.
El dato de laboratorio que apoya el diagnstico en esta paciente es:
a)
b)
c)
d)

Hematuria.
Coluria.
Piuria.
Proteinuria.

Referencia:
La preeclampsia se define por aumento de la presin arterial y la presencia de proteinuria
durante el embarazo.
1) Leve: las pacientes suelen tener unas cuantas manifestaciones, y su presin arterial
diastlica es menor de 110 mmHg. En ocasiones hay edema. La cifra de plaquetas es
mayor de 100 000/l.
2) Grave: los sntomas son ms notorios y persistentes. La presin arterial casi
siempre es con niveles mayores de 160/110 mmHg. Puede haber trombocitopenia
que avance hasta coagulacin intravascular diseminada.
Se requiere hospitalizacin para las mujeres con preeclampsia; debe obtenerse una
biometra hemtica completa con cifra de plaquetas y determinacin de electrolitos, que
adems incluya enzimas hepticas. Se obtiene una muestra de orina de 24 horas para
determinar la depuracin de creatinina y protenas totales al ingreso hospitalario.
Se debe controlar la hipertensin arterial, para evitar sufrimiento fetal, as como
empeoramiento de la paciente.
Bibliografa:
1.

McPhee S, Papadakis M, et. al. Diagnstico Clnico y Tratamiento 2010. Lange, McGraw
Hill,
49 edicin, Mxico, 2010.
2. Sibai BM, Diagnosis, prevention, and management of eclampsia. Obstet Gynecol. 2005.
Feb; 105;: 402 410.

56.- A woman presents with painless vaginal bleeding at 37 weeks gestation. The fetal
heart rate is stable in the 150 pbm. Which of the following is indicated?
a)
b)
c)
d)

Ultrasound examination
Nonstress test
Induction of labor
Digital examination of the cervix

Diagnstico por USG.

Sangrados 2da Mitad Embarazo


Diagnstico diferencial (1)
DPPNI
DOLOR
HIPOGASTRICO

PLACENTA PREVIA

PRESENTE

AUSENTE

AUMENTADO

RELAJADO

COMPROMISO
FETAL

FRECUENTE

INFRECUENTE

U.S.G.

DIAGNOSTICA

TONO UTERINO

DIAGNOSTICA

57.- Una mujer de 33 aos de edad G4 P3, acude a consulta en busca de informacin acerca
de los mtodos anticonceptivos. No tiene APP de importancia, ni toma medicamentos. Ha
mantenido una relacin mongama con su pareja por los ltimos 9 aos. No quiere volver a
embarazarse, pero se niega a que se le practique una salpingoclasia. Su mdico recomienda
un DIU. Cul de los siguientes es ms probable que ocurra con este mtodo
anticonceptivo?

a)
b)
c)
d)

Amenorrea
Embarazo ectpico
Embarazo intrauterino
Aumento de peso

Ventajas del DIU: Sumamente eficaz. No se requieren suministros constantes. Fcil de


usar. No interfiere con el acto sexual. La fertilidad regresa pronto.
Desventajas del DIU: PRESCRIPCIN MDICA para iniciar y discontinuar el uso.
Puede causar dolor o molestia leves durante los procedimientos de insercin y
extraccin. Puede expulsarse. No protege contra las ETS / VIH
Efectos secundarios: Dolor o calambres durante la menstruacin.

Sangrado menstrual prolongado y abundante.

Dolor en la parte baja del abdomen, flujo vaginal, fiebre Embarzo ectpico
DIU
2 por cada 1000 usuarias.

ETIOLOGIA

III.1 Factores que dificultan el camino del huevo. Este se implanta all donde se encuentra
en el 6-7 da postfecundacin.
A/ Procesos inflamatorios, causa ms frecuente de embarazo ectpico. Las salpingitis
deterioran la actividad ciliar y la motilidad tubrica. Hay otros procesos inflamatorios,
como, apendicitis que afectan secundariamente a las trompas, con produccin de
adherencias y acodaduras.
B/ Alteraciones de la motilidad tubrica, como ocurre con la administracin de gestgenos
a bajas dosis de forma continua (minipildora) o el empleo de contracepcin postcoital con
estrgenos.
C/ DIUs. 9-17% de embarazos en portadoras de DIU son ectpicos, lo que se explica
porque el DIU protege ms eficazmente frente a la gestacin intrauterina (995% de
seguridad) que frente al tubrico (95% de seguridad) con lo que aumenta el riesgo relativo
de E.E.
D/ Tratamientos de esterilidad, sobre todo los quirrgicos, cirugia conservadora de E.E
tubricos, ligadura tubrica o tcnicas de fecundacin asistida (in vitro y transferencia
embrionaria intrauterina)
III.2 Factores que favorecen la nidacin en la trompa: endometriosis tubrica, por un
cierto quimiotactismo del endometrio ectpico, as como, por la estenosis y adherencias que
se producen incluso con rganos vecinos.
III.3 Factores dependientes del propio huevo. Las anomalas genticas del huevo, mediante
alteracin en el momento de implantacin, o quiz por un mayor volumen, puede favorecer la
anidacin ectpica.
BIBLIOGRAFA EMBARAZO ECTOPICO

eCabero Roura, Ll y cols. Protocolos de Medicina Materno-fetal (Perinatologa), 2 edicin.


Ed. Ergon, S.A. Madrid. 2000. 120-122.

eBajo Arenas, J.M, Castellanos Bolado, P. Embarazo ectpico. En: Manual de asistencia a la
patologa obsttrica. Fabre E, Ed. INO: Zaragoza. 1997. 89 -107.
eCarrera Macia, J.M y cols. En: Protocolos de obstetricia y Medicina perinatal del I. U.
Dexeus. 3 Edicin. Masson. Barcelona. 2000. 135-139.

58.- Femenino de 22 aos que cursa con amenorrea secundaria la cual presenta
menstruacin posterior a la administracin de progestgenos, lo ms probable es que tenga:

a)
b)
c)
d)

Estrgenos bajos
Estrgenos normales
Progesterona elevada
Gonadotropinas altas

La presencia de estrgenos se puede establecer de dos maneras: con la prueba de desafo


con acetato de medroxiprogesterona (AMP) y el ndice de maduracin vaginal. Si hay
estrgenos, luego de la administracin de 10 mg de AMP diarios por 10 das debera
presentarse un sangrado por vagina. Y de esta forma corroborar los niveles de estrgenos
serian normales.
La ausencia de esta metrorragia sugiere una obstruccin o un
hipogonadismo. La otra alternativa es realizar el ndice de madurez vaginal que es un
procedimiento sencillo que puede obtenerse en el consultorio. Usando una esptula como las
de Papanicolaou el gineclogo extrae una muestra de la vagina y la extiende en un
portaobjeto. Este portaobjeto se procesa y se estudia en el microscopio. Un extendido
maduro tendr clulas epiteliales superficiales grandes y numerosas con un ncleo rodeado
por una gran cantidad de citoplasma. Por el contrario, una muestra no estrognica tendr un
mayor nmero de clulas parabasales y basales con ncleo grande rodeado por escaso
citoplasma.
Speroff L, Glass RH, Kase NG, eds. Clinical Gynecologic Endocrinology and Infertility, 5th
ed. Baltimore: Williams & Wilkins; 1994:334-335.

59.- Se trata de femenino de 34 aos de edad, ingresada a quirfano por una laparoscopa
diagnstica debido a dolor plvico as como en cuadrante superior izquierdo crnicos
(durante los ltimos 2 aos). No tiene alteraciones funcionales vesicales o intestinales.
Tiene antecedente de 2 episodios de gonorrea previos. Bebe una cerveza al da. Labs: HCG
urinaria negativa; Hto 39%; Leuc. T 8 000; Plt 200 000; AST 12; ALT 14.
Intraoperatoriamente se observan adhesiones densas que involucran los oviductos, ovarios
y tero. Tambin se observan adhesiones perihepticas que se extienden desde la
superficie heptica hacia el diafragma. El l diagnstico ms probable es:
a)
b)
c)
d)

Sx de Fitz-Hugh-Curtis
Hepatitis
Carcinoma hepatocelular
Sx Wolff-Parkinson-White

El sndrome de Fitz-Hugh-Curtis se define como la presencia de una perihepatitis asociada


a salpingitis. Los agentes etiolgicos reconocidos hasta la fecha son Chlamydia trachomatis
y Neisseria gonorrhoeae. El cuadro clnico de este sndrome es inespecfico y puede ser
confundido con procesos inflamatorios o infecciosos del tubo digestivo, aparato urinario y
respiratorio, en los cuales la manifestacin sintomtica fundamental es el dolor en
hipocondrio derecho. El diagnstico debe de sospecharse en aquella mujer joven con vida
sexual activa que tenga antecedentes de promiscuidad en ella o en su pareja, que se queje
de dolor subcostal derecho. Es ms probable el diagnstico si se cuenta con el antecedente
de enfermedad plvica inflamatoria y ms an, si se tiene evidencia de que sta sea
causada por Neisseria gonorrehoeae y/o Chlamydia trachomatis. El diagnstico definitivo
se realiza con la visualizacin directa de la adherencia periheptica por laparoscopa o
laparotoma. Se recomienda la primera. El tratamiento mdico es a base de cefalosporinas y
dicloxacilina y en algunos casos se requiere de la extirpacin quirrgica del proceso
adherencial para mitigar el dolor.
El Sndrome de Fitz-Hugh-Curtis. Causa frecuente de error de diagnstico en hepatologa y
gastroenterologa / The Fitz-Hugh-Curtis Syndrome. a frequent misdiagnosis in hepatology
and gastroenterology
Rev. gastroenterol. Mx;60(4):223-8, oct.-dic. 1995.

60.- Se trata de femenino de 26 aos, es atendida en consulta por secrecin lctea


bilateral hace 4 meses y menstruaciones cada 21 das, en escasa cantidad desde hace 6
meses. Antecedentes: hace 6 meses padece gastritis tratada con HBP y metoclopramida
durante 2 meses y posteriormente ha tomado el medicamento de forma irregular. a.g.o.:
menarca 12 aos, ritmo 30/7, nbil. e.f.: talla 1.63, peso 54 kg. al efectuar compresin en
glndulas mamarias se produce salida de secrecin lactea bilateral, resto normal.
El estudio de eleccin en esta paciente para confirmar el diagnstico es:
a)
b)
c)
d)

Perfil hormonal ginecolgico.


Prolactina srica.
Radiografa de silla turca.
Ultrasonido mamario.

La hiperprolactinemia (hiperprl) es un trastorno frecuente, ms en la mujer que en el


hombre, y puede ser la manifestacin de un adenoma hipofisario (prolactinoma). La
medicin de prolactina (prl) en la sangre es ahora un procedimiento de rutina en la
deteccin de trastornos de la menstruacin y galactorrea que se resuelven efectivamente
con el uso de frmacos con actividad dopaminrgica (bromocriptina y cabergolina). La causa
ms frecuente de hiperprl es por el uso de frmacos que inhiben la accin de dopamina,
pero se debe descartar inicialmente hipotiroidismo y un prolactinoma mediante resonancia
magntica.
Los prolactinomas son fcilmente controlados con dopaminrgicos y excepcionalmente se
requiere ciruga. En caso de infertilidad por exceso de prl existe una respuesta favorable

con bromocriptina o cabergolina, aunque en la actualidad se prefiere la ltima por su


efectividad y menos efectos indeseables.

hiperprolactinemia. Gua teraputica y diagnstica arturo zrate*


* Unidad de investigacin de endocrinologa, diabetes y metabolismo,
Centro mdico nacional imss y hospital ngeles Mxico.
Revisin metoclopramida y prolactinemia..

61.- Mujer de 26 aos, con embarazo de trmino, sin antecedentes de control prenatal. G
3. C-1. Se ingresa al servicio de obstetricia por presentar actividad uterina regular y
dolorosa. Ef.: Deambulante, tranquila, adecuada coloracin de tegumentos, abdomen con
fondo uterino a 32 cm. con producto nico vivo en situacin transversa dorso inferior FCF
144, al tacto vaginal crvix dilatado a 3 cm. y membranas ntegras. Se realiza cesrea con
retencin de placenta e invasin a vejiga.
Estudio de gabinete de eleccin previo a ste evento obsttrico es:
a)
b)
c)
d)

prueba sin estrs


perfil biofsico
simple de abdomen
ultrasonido doppler

PLACENTA:
 Se divide en:
 Placenta acreta.
 Placenta increta.
 Placenta percreta.


ACRETA: Las vellosidades se adhieren al miometrio.

INCRETA: Penetran ms de la mitad del espesor del miometrio.

PERCRETA: Atraviesa todo el espesor del miometrio, llegando a la


serosa, incluso atravesndola y adhirindose a rganos vecinos.

Factores:
 Endometrisis previa.
 Tumores submucosos. (Miomas)
 Cicatrz uterina previa. (Cesrea, miomectoma)
 Implantacin baja. (Placenta previa)
 Malformaciones placentarias. (Placenta extracorial)
 Legrado enrgico previo.
 Extraccin manual previa de una placenta.

Diagnstico Pre-parto


Ultrasonografa Dppler.

Resonancia Magntica.

Diagnstico transparto-:
 Placenta retenida por ms de 20 minutos.
 Imposibilidad para encontrar un plano de separacin placentaria cuando se intenta
su extraccin manual.
 Hemorragia incontrolable despus de la pseudoextraccin.
 El diagnstico histopatolgico corrobora el diagnstico clnico.
 Escenario menos deseable.
Tratamiento:
 Histerectoma Obsttrica.
 Constituye una ciruga no planeada y secundaria al hallazgo del acretismo
placentario con sangrado incohercible.


Cesrea-Histerectoma. (Con diagnstico previo)


 Ciruga planificada ante un correcto diagnstico prenatal.

Recomendacin ACOG:
 Maduracin pulmonar intrauterina.
 Inyectar al cordn umbilical 50 mg de metrotexate.
 Ligar el cordn en el nacimiento placentario y dejar la placenta in-situ.
 Embolizacin inmediata de arterias uterinas bilaterales, as como de ramas
de la divisin anterior de la arteria iliaca interna con alcohol polivinlico.
 Continuar con 5 dosis I.M. de 50 mg de metrotexate y cuantificar niveles
de hCG.
 Programar Histerectoma Total Radical Abdominal y/o Cistectoma parcial
y/o reseccin pared anterior recto.

Lee et al. Conservative Management of Placenta Percreta. Obstet Gynecol, 112(2):421-424

62.- Femenino de 29 aos aos, con tumor anexial de 6 cm lquido, dolor abdominal, fiebre,
leucorrea, con historia de cervicovaginitis de repeticin y dispareunia crnica, ltima
menstruacin hace una semana.
El manejo ms adecuado es:
a)
b)
c)
d)

Histerocopa
Colposcopa
Histerosalpingografa
Laparoscopa

CLASIFICACION LAPAROSCOPICA DE LA ENFERMEDAD PELVICA INFLAMATORIA


El stndar de oro para el diagnstico de EPI es la laparoscopia, ya que adems de visualizar
directamente los rganos plvicos, permite la toma de muestras para estudios
bacteriolgicos.
Los criterios laparoscpicos para el diagnstico de EPI, se describen en la tabla No.3.
Tabla No. 3
CLASIFICACION LAPAROSCOPICA DE LA EPI
Leve

Eritema, edema, las trompas se mueven


libremente. No hay exudado purulento.

Moderada

Eritema, edema ms marcado, material


purulento evidente. No hay movimiento libre
de las trompas. La fimbria puede no ser
evidente.

Severa

Presencia de pioslpinx y/o absceso

EPI
Diagnstico
Dolor abdominal bajo y/o dolor a la movilizacin cervical
y anexial y:
Fiebre de 38 C o mayor
leucocitosis > 10, 500 mm3, eritrosedimentacin y PCR elevada
Bacterias o leucocitos en liquido peritoneal
Masa anexial palpable o por USG
Laparoscopia
Sndrome de Fitz- Hugh-Curtis

63.- Se trata de femenino de 33 aos nuligesta pero con actividad sexual regular, sin
mtodo de planificacin familiar, con ciclos regulares, sin leucorrea, refiere dispareunia
profunda, sangrado intermestrual y dismenorrea secundaria ocacionalmente presenta
urgencia urinaria, posterior a confirmar el diagnstico, el tratamiento mdico es
a)
b)
c)
d)

Doxicilclina
Ceftriaxona
Leuprolide
Tibolona

Endometriosis
Tratamiento
Anlogos de GnRH por 6 meses (Leuprolide, Nafarelina,
Triptorelina, Goserelina.

Disminuye la secrecin de FSH y LH .


Danazol
ACO por 9 meses
Laparoscopia
Radical con HTA +SOB
Cncer?
Clulas claras
Endometroide de ovario

64.- Se trata de paciente de 48 aos que presenta sinusorrragia de 3 meses, sin


alteraciones menstruales previas, por lo dems asintomtica.
La alteracin ms probable a encontrar es:
a) Adenomiosis
b) Malformaciones Mllerianas
c) Plipo endometrial
d) Plipo endocervical

Plipo es toda formacin ssil o pediculada que se fija a la cavidad uterina. Las
formaciones polipoideas dentro del aparato genital femenino se dividen en
endometriales y cervicales. Dentro de los plipos cevicales se diferencian los
ectocervicales y los endocervicales; estos ltimos son considerados como los verdaderos
por estar constituidos por epitelio cilndricco. No se conocen con certeza los factores
asociados a su histognesis (teora ms admitida: hiperplasia focal de la mucosa

endocervical por estmulo estrognico) (1). Los plipos cervicales se consideran la forma
ms frecuente de tumor benigno de crvix.
Constituyen de un 3 a un 10% de las consultas ginecolgicas; presentan mxima
incidencia entre la 4 y 5 dcada de la vida. Algunos estudios demuestran que hasta en
el 56% de casos de mujeres postmenopusicas se encuentran asociados plipos
cervicales y endometriales; es mucho menor esta frecuencia antes de la menopausia (2).
La clnica de presentacin ms comn es la hemorragia intermenstrual, espontnea o
durante el coito (3). Tambin pueden ser asintomticos (hallazgo casual en estudio
ecogrfico) o, en ocasiones, alcanzar grandes tamaos: llegan a aflorar por el orificio
cervical externo; se denominan "plipos gigantes" cuando superan 2 cm (4). Para el
diagnstico habitualmente se utiliza la ecografa. Los casos sintomticos se consideran
de indicacin quirrgica por su riesgo de degeneracin a adenocarcinoma cervical.
Los plipos cervicales constituyen una patologa frecuente en la consulta ginecolgica,
sobre todo en mujeres mayores de 20 aos que han tenido hijos y son poco comunes antes
de la menarquia. En la mayora de los casos, solamente se presenta un plipo, aunque en
ocasiones se pueden encuentrar 2 3. La poca ms frecuente es al final de su vida
reproductiva y despus de la menopausia. En las primeras, las lesiones suelen ser ms
grandes, difciles de tratar y con frecuencia recidivan.
La causa de los plipos cervicales an no se ha comprendido completamente, pero con
frecuencia son el resultado de una infeccin crnica, una respuesta local anormal a los
niveles de estrgeno o a una congestin local de los vasos sanguneoscervicales.
Clnicamente se manifiesta con sangrado genital fuera de la menstruacin, sobre todo
durante la relacin sexual. Son benignos, no se malignizan pero deben ser extirpados para
controlar la sinusorragia.

65.- Femenino de 39 aos con embarazo de 32 semanas. Ingresa al servicio de urgencias


por presentar cuadro de 1 da de evolucin por sangrado transvaginal leve, rojo brillante,
sin contractilidad uterina. A.G.O.: G-3, P-0, C-2. E.F.: IMC: 30 kg/m2, F.U. de 28 cm, fcf
presente, especuloscopa crvix cerrado con huellas de sangrado. USG reporta producto
nico vivo con fetometra normal, lquido amnitico normal y placenta anterior que cubre
parcialmente el orificio cervical interno.

El siguiente paso en la atencin de esta paciente es:


a)
b)
c)
d)

manejo por consulta externa y vigilar sangrado


manejo por consulta externa con uteroinhibidores
ingreso a hospital e inductores de maduracion pulmonar
ingreso a hospital y cesrea.

66.- Femenino de 22 aos de edad, acude al servicio de urgencias por referir malestar
general, fosfenos, nusea y vmito. Antecedentes: cursa con embarazo de 36.5 SDG, G2,
A1, C0. Exploracin fsica: TA 185/110 mmHg, FC 120 lpm, FR 35 x, T37.9, alerta,
inquieta, aprecia ictericia, cardiopulmonar sin compromiso, abdomen globoso a expensas de
tero grvido, se detecta PUVI, longitudinal, ceflico, dorso a la derecha, FCF 142 x, al
tacto genital crvix, central, formado, cerrado, extraccin de guantes sin evidencia de
prdidas por genitales, extremidades inferiores edema +++.
La medida teraputica inicial en esta paciente es:
a)
b)
c)
d)

Diurticos.
Sedantes.
Antihipertensivos.
Analgsicos.

Referencia:
La preeclampsia se define por aumento de la presin arterial y la presencia de proteinuria
durante el embarazo.
1) Leve: las pacientes suelen tener unas cuantas manifestaciones, y su presin arterial
diastlica es menor de 110 mmHg. En ocasiones hay edema. La cifra de plaquetas es
mayor de 100 000/l.
2) Grave: los sntomas son ms notorios y persistentes. La presin arterial casi
siempre es con niveles mayores de 160/110 mmHg. Puede haber trombocitopenia
que avance hasta coagulacin intravascular diseminada.
Se requiere hospitalizacin para las mujeres con preeclampsia; debe obtenerse una
biometra hemtica completa con cifra de plaquetas y determinacin de electrolitos, que
adems incluya enzimas hepticas. Se obtiene una muestra de orina de 24 horas para
determinar la depuracin de creatinina y protenas totales al ingreso hospitalario.
Se debe controlar la hipertensin arterial, para evitar sufrimiento fetal, as como
empeoramiento de la paciente.
Bibliografa:
1.

McPhee S, Papadakis M, et. al. Diagnstico Clnico y Tratamiento 2010. Lange, McGraw
Hill,
49 edicin, Mxico, 2010.
2. Sibai BM, Diagnosis, prevention, and management of eclampsia. Obstet Gynecol. 2005.
Feb; 105;: 402 410.

67.- Se trata de femenino de 41 aos, portadora de DM tipo II e HTAS, controlada.


Acude a consulta refiriendo descarga vaginal prurtica y blanquecina, Cul de los
siguientes exmenes de diagnsticos es ms til para identificar el patgeno?
a)
b)
c)
d)

Wet prep.
Tincion de Gram
PH
KOH

Prueba del KOH es un procedimiento en el cual hidrxido del potasio (KOH) se utiliza
detectar hongos disolviendo las clulas humanas en una cultura. La diferencia en la
composicin de la pared de clula de clulas humanas y de clulas fungicidas permite que
este procedimiento ayude a distinguir las dos clulas. El KOH desnaturaliza las protenas en
la clula humana; solamente sigue habiendo las clulas fungicidas ser considerado debajo
del microscopio.
1. Bernal B. Fisiologa y ecologa de la vagina. Rev Chil Obstet Ginecol 1986; 51:56-60.

2. Ibrcena E. Vaginosis bacteriana; diagnstico y prevalencia. XII Congreso Peruano de


Obstetricia y Ginecologa 1996; 204-6.
3. Scapini JC, Guzmn CA. Deteccin de bacilos Gram negativos curvos anaerobios en
pacientes con vaginosis. Obstet Ginecol Latinoam 1986; 44: 320-5.
4. Soihet S. El flujo vaginal en la consulta ginecolgica. Ginecol Obstet (Per) 1986; 30: 5060.
5. Linaldi CAS, Urbina JR, Castaeda JL. Vaginitis por Gardnerella vaginalis en nias y
adolescentes. Bol Med Infant Mex 1988; 45: 101-3.
6. Sobel J. Vaginitis and vaginal flora. Controversies abound. Current Opin Infect Dis 1996;
9: 42-7.
7. Faro S. Leucorrea. Causas infecciosas e imbalances en el ecosistema vaginal. Tribuna
Mdica. Cahners Healthcare ed 1996; 1 10.

68.- Femenino de 29 aos. Acude al servicio de urgencias por presentar salida de lquido
vaginal. Antecedentes: G2, P1, cursa embarazo de 36 semanas de gestacin exploracin
fsica: cervix con 10% de borramiento, 1 cm de dilatacin y Tarnier positivo.
La complicacin ms frecuente en esta paciente es?

a)

Corioamnioitis.

b)

Parto pre trmino.

c)

Sepsis neonatal.

d)

Endometritis.

Corioamnioitis: El diagnstico de la infeccin intraamnitica (IIA) es bsicamente clnico.


La corioamnionitis se debe descartar en toda gestante que presente fiebre sin foco
aparente, sobre todo si se sospecha o se ha confirmado una rotura de membranas. Los
criterios ms empleados para el diagnstico son: fiebre materna y, al menos, 2 de los
siguientes signos: taquicardia materna, taquicardia fetal, irritabilidad uterina, leucocitosis
materna o lquido amnitico purulento o maloliente.
Progresos de obstetricia y ginecologa: revista oficial de la Sociedad espaola de
ginecologa y obstetricia, ISSN 0304-5013, Vol. 48, N. 6, 2005 , pags. 316-317

69.- Se trata de paciente de 34 aos que cursa con 39 SDG; a la exploracin fsica
reflejos patelares hiperactivos, inquieta, se reportan cifras de TA 145/95, se realiza
laboratorio que reporta proteinuria 2+.
El tratamiento ms adecuado para esta paciente es:

a)
b)
c)
d)

Reposo en cama
Sulfato de magnesio oral
Propranolol
Interrupcin del embarazo.

Interrupcin del embarazo


La mayor parte de las guas de prctica clnica que contestan esta pregunta se basan en
estudios retrospectivos o recomendaciones de los comits de expertos (Nivel III/IV). La
interrupcin del embarazo se considera la mejor opcin de tratamiento para la
preeclampsia y, sin duda, lo es para la embarazada al prevenir la preeclampsia severa y la
eclampsia. Sin embargo, para el feto no siempre es la mejor opcin. Dos estudios
observacionales retrospectivos mostraron que a las 37 semanas la terminacin del
embarazo es la mejor opcin, tomando en cuenta que si existen condiciones cervicales
favorables, la induccin del parto es una via segura y de bajo riesgo para el feto (Nivel
III). En embarazos pretrmino debe considerarse la prolongacin del embarazo hasta
alcanzar el mayor peso y madurez fetal.
Atencin conservadora de pacientes con preeclampsia leve
Cuando el embarazo cursa entre las 28 y 34 semanas, el manejo conservador puede reducir
las complicaciones propias de la prematuridad. La decisin debe basarse en la estabilidad
del estado materno y fetal, as como en la capacidad del centro hospitalario para brindar
atencin optima a un recin nacido prematuro (Nivel III). Durante el periodo del monitoreo
ambulatorio se indica a la paciente que se realice en forma externa una medicin diaria de
la presin arterial, y que acuda semanalmente a la consulta para descartar evolucin o
agravamiento de la enfermedad (Nivel IV). No existen estudios controlados que hayan
determinado la magnitudde los riesgos maternos y fetales durante el tratamiento
conservador de la paciente con preeclampsia leve.
Recomendaciones
La paciente con preeclampsia leve, seleccionada y clasificada despus de su estancia
hospitalaria, puede atenderse en forma ambulatoria, con vigilancia semanal en la consulta,
como una medida efectiva y segura.

(Grado de recomendacin B)
En las pacientes con embarazo 37 semanas y preeclampsia leve, la terminacin del
embarazo es la mejor opcin, valorando la induccin del parto como una medida de bajo
riesgo para el feto.

(Grado de recomendacin C)
La atencin conservadora del embarazo menor de 34 semanas, complicado por preclampsia
leve, mejora el pronstico del feto.

(Grado de recomendacin C)

Ginecol Obstet Mex 2010;78(6):S461-S525


Guas de prctica clnica
Diagnstico y tratamiento de la
preeclampsia-eclampsia
Fecha de busqueda de informacion: marzo 2009
Fecha de elaboracion: septiembre 2009
Fecha de actualizacion: junio 2012
Institucin responsable: Colegio Mexicano de Especialistas en Ginecologa y Obstetricia.
Coordinador del grupo
Dra. Maria Teresa Leis Marquez

Especialista en Ginecologa y Obstetricia y Medicina Materno Fetal. Certificada por el


Consejo Mexicano de Ginecologa y Obstetricia. Jefa de la Clnica de Medicina Materno
Fetal del Hospital ngeles Lomas.
Coordinadora del Comit para la elaboracin de las Guas de Prctica Clnica del Colegio
Mexicano de
Especialistas en Ginecologa y Obstetricia, periodo 2008-2010. Miembro de la Junta de
Gobierno y del
Comit de Exmenes del Consejo Mexicano de Ginecologa y Obstetricia. Vicepresidenta
para Amrica del Norte de la Sociedad Iberoamericana de Diagnstico y Tratamiento
Prenatal.

70.- Mujer de 32 aos G/4, P/0 con 6 SDG acude a su primera visita prenatal. Su historia
obsttrica pasada es importante porque tiene tres prdidas de producto en el segundo
trimestre. Refiere que en las tres ocasiones al presentarse al hospital presentaba dilacin
cervical completa. No recuerda haber tenido contracciones dolorosas. Niega antecedentes
mdicos y quirrgicos. El examen fsico es normal incluyendo un examen plvico que
muestra un cervix largo y cerrado. Despus de una larga discusin con la paciente ella pide
que se le practique un cerclaje durante este embarazo. Cul de los siguientes es el
momento ms apropiado para realizarlo:

a)
b)
c)
d)

Inmediatamente
12 a 16 semana
24 a 28 semanas
32 a 36 semanas

El cerclaje cervical tiene sus indicaciones en la profilaxis y tratamiento de la incompetencia


cervical.
La incompetencia o insuficiencia cervical representa un 10% de las causas de parto
pretrmino y est asociada a una importante morbimortalidad neonatal.
Las modificaciones cervicales en el segundo trimestre de gestacin son causa de parto
prematuro y pueden deberse a:
1) Incompetencia cervical.
2) Prdida de tejido conectivo tras una ciruga cervical (conizacin).
3) Defectos congnitos como la hipoplasia cervical tras exposicin a dietilestilbestrol.

4) Infeccin intrauterina. Hasta un 51.5% de las pacientes con clnica compatible con
incompetencia cervical enmascaran un cuadro de infeccin intraamnitica subclnica.
Diferenciamos tres tipos de cerclaje:
1. El cerclaje se considera profilctico o electivo (o primario) cuando se realiza de forma
electiva por historia previa de incompetencia cervical antes de evidenciar cambios en el
cerviz y generalmente suele realizarse entre las 13 y 16 semanas de gestacin.
2. El cerclaje teraputico secundario que se realiza tras la deteccin, en el seguimiento
obsttrico, de modificaciones en el crvix antes de las 26 semanas de gestacin. Se realiza
en pacientes con un riesgo potencial de parto pretrmino.
3. El cerclaje teraputico terciario, en caliente, de rescate o emergent cerclage que
se realiza en pacientes que presentan la membrana amnitica visible a travs del orificio
cervical externo o en vagina.

GUIA CLNICA:
INDICACIONES DEL CERCLAJE
Unitat de Prematuritat. Servei de Medicina Maternofetal.
Institut Clnic de Ginecologia, Obstetrcia i Neonatologia, Hospital Clnic de Barcelona
Responsables del protocolo: T.Cobo, M. Lpez, M. Palacio
Creacin: 24/01/07
Modificaciones: 05/09/07
ltima actualizacin: 17/01/10

71.- Femenino de 40 aos de edad G.3 P.1 C 2, la cual es diagnosticada por miomatosis
uterina de pequeos y medianos elementos sintomticos, sus antecedentes refieren
cirugas plvicas previas, el tratamiento de eleccin es:

a)
b)
c)
d)

Histerectoma total abdominal.


Progesterona.
Observacin
Anlogos de GnRH.

CUADRO CLINICO
La miomatosis uterina muestra manifestaciones clnicas en menos del 50%, de estas las ms
frecuentes son:
1. Hemorragia uterina anormal.
2. Dolor.
3. Distensin abdominal.

4. Compresin genitouterina.
5. Compresin gastrointestinal.
6. Compresin plvica.
10. DIAGNOSTICO
El diagnstico se realiza a travs de imagenologa:
1-Ecografa.
2-TAC
3-Rayos X
4-Histeroscopia.

11. TRATAMIENTO
La miomatosis uterina debe ser tratada cuando produzca cualquiera de las manifestaciones
clnicas anotadas, toda paciente que se programe para histerectoma debe tener
previamente legrado biopsia
Lo podemos dividir en Conservador o Radical.
1.

CONSERVADOR:

Este tratamiento se puede instaurar en pacientes con deseo de preservar el tero.


Igualmente se puede subdividir:
-Expectante
-Quirrgico: miomectoma
-Medico
Tratamiento expectante: Esta indicado en pacientes, cuyos sntomas son leves y no deseen
o tengan alguna contraindicacin medica para tratamiento quirrgico. En ellas se
recomienda controles clnicos y ecogrficos cada 6 meses a 1 ao.
Tratamiento mdico.
AINES
Anlogos GnRH:
Progestgenos:
Andrgenos.
Antiandrgenos

Tratamiento quirrgico:
MIOMECTOMIA:

1. Criterios del ACOG en pacientes infecundas.


Procedimientos:
Va endoscpica:
Laparoscopia: Miomas subserosos sesiles o pediculados < 5cm.
Histeroscopia: miomas submucosos
Va laparotoma
Aquellos miomas que se salgan de las caractersticas anteriores.
Indicaciones:





Hemorragia anormal.
Perdida reproductiva.
Infecundidad.
Dolor.

El tratamiento previo con anlogos esta indicado cuando se desee disminuir el tamao del
mioma para prevenir sangrado quirrgico.

Contraindicaciones:





Embarazo.
Cncer endometrial.
Infecciones.
Dificultad tcnica.

2. Criterios del ACOG para Miomectoma en pacientes que desean conservar el tero.
Procedimiento:

Va endoscpica.
Va abdominal.
Va vaginal.

Indicacin:

a.

Presencia de uno o dos Leiomiomas asintomticos de tamao tal que se pueden palpar
por va abdominal y constituyen una preocupacin para la paciente.

b.

Pacientes ovulatorias con miomas como posible causa de hemorragia uterina excesiva,
demostrada por cualquiera de las siguientes circunstancias:
Hemorragia profusa: de duracin mayor de 8 das.
Anemia por prdida sangunea aguda o crnica.

2. RADICAL:

HISTERECTOMIA
Para pacientes post menopusicas, con paridad satisfecha o sin deseo de preservar el
tero.
Criterios del ACOG para Histerectoma por miomas.
1.

Presencia de 1, 2, o 3 Miomas asintomticos de tamao tal que son palpables por va


abdominal y preocupan a la paciente.

2. Hemorragia uterina excesiva.

Duracin mayor de 8 das.


Anemia por prdida sangunea aguda o crnica.

1.

Molestias plvicas producidas por los miomas: signos compresivos.

Contraindicaciones:
1. Deseo de conservar la fecundidad.
2. Miomas asintomticos.
3. Contraindicacin mdica o dificultades tcnicas para la ciruga.

Guarnaccia M. and Rein M. Traditional Surgical Approaches to Uterine Fibroids


Abdominal. Myomectomy and Hysterectomy. Clinical Obstetrics and Gynecology
2001. 44.2. 385-400
- Milad. M and Sankpal R. Laparoscopic Approaches to Uterine Leiomyomas.
Clinical Obstetrics and Gynecology. 2001. 44-2. 401-411
- Carlson K. et al. Indications for Hysterectomy. N. Engl. J. Med. 1993. 328(12) 56-

72.- Se trata de femenino de 31 aos nuligesta pero con actividad sexual regular, sin
mtodo de planificacin familiar, con ciclos regulares, sin leucorrea, refiere dispareunia
profunda, sangrado intermestrual y dismenorrea secundaria ocacionalmente presenta
urgencia urinaria, el mtodo de eleccin para confirmar el diagnstico clnico de sta
paciente es:

a)
b)
c)
d)

Cultivo de exudado vaginal


Ultrasonido
Biopsia de endometrio
Laparoscopia con biopsias dirigidas

Endometriosis
Examinacin bimanual
Dolor en los ligamentos utero-sacros
Nodularidad en el fondo de saco
Induracin del septo rectovaginal
tero en retroversin fijo
Masa anexial

Endometriosis
Laparoscopia
visualizacin directa
estadificacin
toma de muestras

Estadio I Minimo
Estadio II Leve
Estadio III Moderado
Estadio IV Severo

Referencias bibliogrficas:

1. Ruiz V. Endometriosis y fertilidad. Ed. Acosta y Warman, pp. 99


2. Lpes,VH. Palomo E. Incidencias de endometriosis en una poblacin infrtil. XXI
Congreso nacional de Ginecologa y Obtetricia. Guatemala, 1993.
3. El-Eoley, et al. Danazol but not ginadotropin releasing hormone agonists suppresses
autoantibodies in endomeriosis. Fertil Steril 1990; 54:725
4. Acosta AA. Buttram VC Jr. Besch PK, Malinak LR, Van Der Heyden J. A.proposed
classfication of pelvic endometriosis. Obstet Gynecol 1973;42:19.
5. Buttran VC Jr. Evolution of the revised American Fertility classification of
endometriosis. Fert. Steril 1985; 43: 347
6. Lpez VH. Tratamiento mdico-quirrgico de la endometriosis. Simposio El rostro
cambiante de la endometriosis panam 3. 12. 1993.
7. Steinleitner A. Heterolous transplation of activated murine peritonel macrophages
inhibitis gamete interaction in vivo; A paradigm fo endometriosis associted subfertility.
Fertil Steril 1990; 54:725.
8. Damewood M. Effect of serum from patients with minimal to mild endometriosis on
mouse embryo growth. Fertil Steril 1990; 54: 917
9. Proug S. Peritoneal fluid fracctions from patients with endometriosis do not promote
two-cell mouse embryo growth. Fertil Steril 1990; 54: 927.

73.- Se trata de paciente femenino de 37 aos de edad, G-3, C-2. Es ingresada a hospital
presentando cefalea, acfenos, fosfenos y epigastralgia en barra con embarazo de 34
semanas. E.F T/A 160/110, FC 84 x, FR 18 x, no presenta fiebre, somnolienta, sin
agregados cardioventilatorios, hepatalgia. F.U. de 25 cm. Producto nico vivo. FCF 110 lpm,
genitales sin prdidas ni modificaciones cervicales. Laboratorio: hb 9.8 g/dl, plaquetas de
54 mil, TP 11 seg TPT 27, TGO 160 ng/dl TGP 160 ng/dl, hiperbilirrubinemia indirecta,
albuminuria 300 mg/dl, cido rico de 8.1 mg/dl, creatinina de 1.5 mg/dl.
El diagnstico ms probable es:

a)
b)
c)
d)

Sx anticuerpos antifisfolpidos
Prpura trombocitopnica trombtica
Sindrome de hellp.
Hgado graso

DEFINICIN:
Es una complicacin de la preeclampsia en la cual adems de la Hipertensin Arterial y
proteinuria hay presencia de anemia hemoltica, enzimas hepticas elevadas y recuento
bajo de plaquetas

MANIFESTACIONES CLINICAS:
Malestar general, fatiga y molestias inespecficas 90%
Cefalea 70%
Epigastralgia 64%
Vmito 22%
Fosfenos 15%
Visin Borrosa 11%
Acfenos 3%
Ictericia
Anemia no explicada
Oliguria

BIBLIOGRAFIA:
Sibai baha, El sndrome HELLP. Universidad de Valencia , revista quincenal de Obstetricia
clnica y ginecologa, Octubre 2003.
V. Cararach, Sndrome de HELLP y Repercusiones maternas. X curso intensivo de
formacin continuada materno fetal. Enero de 2003.
Toirac, Abelardo. Sndrome de Weistein HELLP Hospital Ginecoobstetrico Tamara Bunke.
Junio 2002
De la Fuente, David. Sndrome HELLP. Medicina Universitria 2003; 5 (19): 101 -9
Andrea G. Witlin, DO, Baha M. Sibai, MD. Diagnosis and Management of women with
Hemolysis Elevate Liver Enzymes, and Pletelet Count (HELLP) syndrome. Hospital Physician.
Febrero 1999.
CIFUENTES B, Rodrigo. Ginecologa y obstetricia bas

74.- Femenino de 33 aos que cursa con 38.5 semanas de gestacin que ingresa al servicio
con trabajo de parto. En el transcurso de trabajo de parto durante la dilatacin presenta
dolor intenso y brusco. A la exploracin usted observa metrorragia escasa y aumento del
tono uterino a la palpacin abdominal que resulta muy doloroso. El diagnstico ms
probable es:
a)
b)
c)
d)

Placenta previa.
Rotura de vasos previos.
Desprendimiento de placenta.
Crioamnionitis hemorrgica.

Fisiopatologa de la hemorragia
La hemorragia es el signo fundamental que domina el cuadro clnico de la placenta previa.
Para explicar su mecanismo existen distintas teoras:
Mecanismo de Jacquemier: Se produce crecimiento armnico de la placenta y del tero
hasta la semana 26, 28. Despus el segmento inferior crece ms deprisa y favorece el
despegamiento lo que origina la hemorragia en el embarazo.
Mecanismo de Schroeder: Las contracciones uterinas en el parto traccionan del segmento
inferior hacia arriba y empujan al feto hacia abajo despegando la placenta.
Mecanismo de Pinard: Explica las hemorragias gestacionales y del parto. El estiramiento de
las membranas de la zona de menor radio (orificio interno cervical) como consecuencia de
las contracciones tira de la placenta y la desprenden.
Mecanismo de Bartholomew: Explica la hemorragia en los casos de placenta previa central.
La zona placentaria que reviste el orificio interno es un rea isqumica ya que no recibe
vasos deciduales. A este nivel disminuye la presin sangunea, por lo que la sangre tiende a
dirigirse hacia esta zona y escapa por la cara materna.
En el alumbramiento tambin puede haber una hemorragia importante producida por un
doble mecanismo:
-desprendimiento parcial antes de la expulsin en los casos de placenta oclusiva.
-atona uterina en la zona de insercin despus de expulsada la placenta y vascularizacin
anmala.
Manifestaciones clnicas de la placenta previa en el embarazo
- Sntomas: Principalmente la hemorragia. Toda hemorragia vaginal acontecida en el tercer
trimestre debe hacer pensar en una placenta previa. Las hemorragias suelen ser
espontneas, no acompaadas de dolor, de sangre roja y se presentan de forma
intermitente, con intervalos variables entre las mismas. Progresivamente se van haciendo
ms frecuentes y ms graves. La primera hemorragia suele aparecer en forma inesperada
generalmente nocturna, cesando en menos de media hora. Las hemorragias ulteriores son
ms graves y ms precoces.
- Signos: La consecuencia fundamental es la anemia materna que depende de la cuanta de la
hemorragia (la sangre es de origen materna ya que procede de espacios intervellosos).
- Exploracin:
Exploracin general para valorar la existencia de signos de anemia.
Exploracin obsttrica: valorar el tamao del tero (adecuado para la edad gestacional), es
blando e indoloro. A menudo la esttica fetal est alterada (transverso, oblicuo, nalgas). No
debe efectuarse tacto vaginal cuando haya existido hemorragia en embarazo avanzado por
el riesgo de infeccin y de despegamiento y aumentar as la hemorragia.
Auscultacin fetal normal.
Exploracin ecogrfica: es una tcnica fundamental en el diagnstico de la placenta previa.
Permite determinar la localizacin placentaria y la variedad de la placenta previa. En
general, la placenta puede identificarse a partir de la 9 semana. No obstante, a lo largo de
la gestacin por crecimiento uterino se produce un cambio en sus relaciones con el tero
("emigracin placentaria", imagen de desplazamiento). As el diagnstico de certeza de
lmites placentarios solo puede establecerse hacia la semana 34. Siempre ser necesario
hacer una adecuada identificacin del orificio cervical interno (ms fcil con sonda

transvaginal). Aadiendo al estudio Doppler color se observa la vascularizacin y las zonas


que sangran.

75.- Femenino de 19 aos, atendida en sala de urgencias ginecoobsttricas, Antecedente:


cursa embarazo de 38 SDG. Exploracin Fsica: en trabajo de parto. Repentinamente
presenta sangrado profuso transvaginal y dolor abdominal.
El manejo apropiado en esta paciente es:

a)
b)
c)
d)

Tocolisis con un agonista beta adrenrgico


Continuar el monitoreo de la madre y el producto
Amniotoma
Cesrea

El manejo de stas pacientes es estricto y urgente:


1- Ingreso inmediato de la paciente en vigilancia a sala de partos.
2- Venopuntura. Si existen signos de shock, instaurar preferentemente dos vas, una
central y otra perifrica.
3- Solucin de Ringer.
4- Monitoro de signos vitales cada 15.
5- Sonda vesical y monitoro de diuresis. Diuresis de 50 ml/h asegura perfusin perifrica
correcta.
Evitar diuresis inferiores a 30 ml/h.
6- Monitoro contnuo de LCF.
7- Laboratorio de urgencia: hemograma, ionograma, urea y creatinina, coagulograma.
8- Sangre como mnimo 4U para reserva (concentrado de glbulos rojos o sangre entera).
9- Ecografa obsttrica para localizar y medir el hematoma y descartar otras causas de
sangrado.
Dependiendo del grado de desprendimiento realizar:
1- Desprendimiento leve: sin compromiso materno ni fetal y el embarazo es de pretrmino,
conducta expectante. Uteroinhibicin y maduracin pulmonar fetal.
2- Desprendimientos moderados a severos: si hay compromiso materno o fetal u bito
fetal, terminar inmediatamente la gestacin.
3- Independientemente de la va del parto, es preceptiva la prctica de la amniotoma, para
reducir la presin intraamnitica y disminuir el paso de tromboplastina hstica a la
circulacin y la extravasacin de sangre al miometrio.
Las principales complicaciones son:
1- Hipotensin, shock hipovolmico.
2- Coagulacin intravascular diseminada.
3- Necrosis cortical y necrosis tubular aguda renal.
Obstetricia. Scwarcz, Sala, Duverges. 7 edic. Edit. El Ateneo. (Biblioteca Fac.
Med. UNNE).

76.- A 23-year-old nulligravid female has not menstruated in the past 4 months. Previously,
her menstrual cycles were regular. She is otherwise well and denies recent onset of
stress, change in exercise routine, headaches, visual field alterations, or galactorrhea..
She has a body mass index of 24, blood pressure of 120/78 mm Hg, and does not appear
hirsute. No adnexal masses can be palpated. Laboratory investigations reveal a negative
beta human chorionic gonadotropin (-hCG), normal thyroid-stimulating hormone (TSH),
and prolactin levels.
What is the next best step in the management of this patient?
a)

Measurement of luteinizing hormone.

b)

CT scan of the sella turca.

c)

Prescribe oral estrogen for 21 days followed by 7 days of medroxyprgesterona and

reevaluate.
d)

Preescribe 7 days of medroxyprgesterona and reevaluate.

La Prueba de respuesta a progestgenos se basa en la observacin de que el tratamiento


con progestgeno (acetato de medroxiprogesterona 10 mg por 5 a 6 das) solo induce la
menstruacin en las mujeres con concentraciones normales de estrgenos circulantes. Una
prueba positiva (hemorragia despus de concluir el tratamiento con progestgenos) seala
cifras normales de produccin de estrgenos y una prueba negativa (sin hemorragia por
privacin), hipogonadismo franco.
Danforth, Tratado de Obstetricia y Ginecologia, 9 Edicin, Ed. Mc Graw Hill
Interamericana, Pg 668

77.- Mujer de 27 aos, es atendida en consulta en la clnica de displasias por papanicolau


con lesin NIC I. Antecedentes: menarca 14 aos, ritmo 30x5 eumenorreica, inicio de vida
sexual a los 15 aos, 2 parejas sexuales, mtodo de planificacin familiar oclusin tubaria
bilateral, gestas 3 partos 3, crvix con lesin acetoblanca con extensin lineal de 2 cm. Se
realiza biopsia de la lesin, en caso de corroborarse el diagnstico, el siguiente paso en el
manejo de esta paciente es realizar:

a)
b)
c)
d)

biopsia.
electrociruga.
cepillado de canal.
ultrasonido endovaginal.

9.5.2 Las pacientes a quienes se les realiz citologa cervical, cuyo resultado es LEIBG
(infeccin por VPH, displasia leve o NIC 1); LEIAG (displasia moderada y grave o NIC 2 y 3)
o cncer deben enviarse a una clnica de colposcopa, para realizar estudio colposcpico.
9.5.3 Si el resultado de la citologa es LEIBG, la colposcopa es satisfactoria y sin
evidencia de LEIBG, se realizar control citolgico en un ao (Apndice Normativo A)
9.5.4 Si la citologa es de LEIBG, la colposcopa es satisfactoria y existe evidencia de
lesin, se debe tomar una biopsia dirigida.
9.5.4.1 Si la biopsia dirigida es negativa, se realizar nueva colposcopa para verificar el
diagnstico y en caso necesario, tomar nueva biopsia dirigida y revalorar.
9.5.4.2 Si la biopsia dirigida es reportada como LEIBG se podr dar tratamiento
conservador: criociruga, electrociruga o laserterapia (slo si cumple con las condiciones
referidas en el Apndice 1) o se podr mantener a la paciente en vigilancia en la clnica de
colposcopa, con colposcopa y estudio citolgico cada seis meses, durante 24 meses.
Jueves 31 de mayo de 2007 DIARIO OFICIAL (Primera Seccin)
9.5.4.3 Si la biopsia dirigida es reportada como LEIAG (Lesin Intraepitelial Escamosa de
Alto Grado) se realizar tratamiento conservador (electrociruga o laserterapia). En las
mujeres posmenopusicas, dependiendo de las condiciones anatmicas del crvix, se
realizar tratamiento conservador en la clnica de colposcopa o tratamiento quirrgico
(histerectoma extrafascial) en el servicio que corresponda.
9.5.4.4 Si la biopsia dirigida reporta cncer microinvasor o invasor, la paciente se
transferir a un Servicio o Centro Oncolgico para su tratamiento correspondiente.
9.5.4.5 Si la citologa reporta LEIBG y la colposcopa es no satisfactoria, se tomar
cepillado endocervical (Apndice Normativo A)
9.6 En caso de colposcopa no satisfactoria, negativa a LEIBG y con cepillado endocervical
negativo, se continuar su control en la clnica de colposcopa en seis meses, con colposcopa
y citologa.
9.6.1.1 Si el cepillado endocervical reporta LEIBG se tratar a la paciente como LEIAG,
con mtodos conservadores escisionales.

Jueves 31 de mayo de 2007 DIARIO OFICIAL (Primera Seccin)


Modificacin a la Norma Oficial Mexicana NOM-014-SSA2-1994, Para la prevencin,
deteccin, diagnstico, tratamiento, control y vigilancia epidemiolgica del cncer
crvico uterino.
Al margen un sello con el Escudo Nacional, que dice: Estados Unidos Mexicanos.- Secretara
de Salud.
MODIFICACION A LA NORMA OFICIAL MEXICANA NOM-014-SSA2-1994, PARA LA
PREVENCION,
DETECCION,
DIAGNOSTICO,
TRATAMIENTO,
CONTROL
Y
VIGILANCIA
EPIDEMIOLOGICA DEL CANCER CERVICO UTERINO.

78.- Femenino de 36 aos, con presencia de hiper y polimenorreas o prdidas de sangre


contnuas sin conservacin del ciclo, se sospecha de miomatosis uterina, ste tipo
alteracin se presenta con mayor frecuencia en los miomas de localizacin:

a) Submucoso
b) Intramural
d) Intraligamentaria
e) Cervical

Descripcin
Los miomas son tumores monoclonales benignos de las clulas del msculo liso del
miometrio. Estn compuestos por grandes cantidades de matriz extracelular que contiene
colgeno, fibronectina y proteoglicanos. El colgeno tipo I y tipo II es abundante, pero las
fibrillas de colgeno son anormales y estn desorganizadas, de modo similar a lo que se
observa en la formacin de queloides.
Los miomas submucosos son los menos frecuentes, constituyendo nicamente el 5% de la
totalidad de los miomas, a menudo producen un aumento del sangrado menstrual en forma
de hiper y polimenorreas, e incluso hemorragias importantes que exigen tratamiento de
urgencia. Las metrorragias tambin son habituales en este tipo de miomas. Segn Novak, el
peligro de degeneracin sarcomatosa es mucho mayor en los miomas submucosos, e
igualmente es causa de dismenorreas ms intensas y frecuentes.

BIBLIOGRAFA
1. De la Fuente U. Tratado de Obstetricia y Ginecologa. Mc Graw-Hill. (Madrid). 1998.
Volumen II.
2. DI SAIA S. Tratado de Obstetricia y Ginecologa de Danforth. Sexta. Mc Graw-Hill.
(Nueva York). 1990. Sexta Edicin.
3. Gonzlez-Merlo J. Tratado de Obstetricia y Ginecologa. Salvat Editores S.A.
(Barcelona). 1990. Quinta Edicin.
4. Aller J., Pages G. Obstetricia Moderna. Tercera Edicin. Mc Graw-Hill. (Caracas). 1999.
5. Formacin Mdica Continuada en Atencin Primaria. Marzo 1995. Vo. 5, N (3).

79.- Se trata de femenino de 54 aos con antecedentes de G.3,P.3, presenta prolapso


uterino III e incontinencia urinaria de esfuerzo, el procedimiento de eleccin en esta
paciente es:

a)
b)
c)
d)

Histerectomia total abdominal.


Histerectoma vaginal reconstructiva
Colpoperineoplasta.
Uterosuspensin.

El prolapso genital y su tratamiento ha sido siempre un importante captulo de la


ginecologa. No existe consenso clnico del concepto que define al prolapso genital como
patolgico. Cierto grado de descenso y relajacin de la pared vaginal es considerado normal
en la gran mayora de las mujeres, siendo ms frecuente en mujeres de mayor edad. En la
poblacin general solo un 3% presenta prolapso genital severo, entendiendo como tal al de

III y IV grado. Es probable que alrededor de 3 a 6% de la poblacin femenina desarrolle


un prolapso de esas caractersticas en algn momento de su vida
La intervencin quirrgica por excelencia para el tratamiento quirrgico del prolapso
uterino es la histerectoma vaginal. Esta intervencin se clasifica como "limpia-contaminada

INDICACIONES PARA LA HISTERECTOMA VAGINAL


Prolapso uterino
Hemorragia uterina disfuncional
Carcinoma cervico-uterino "in situ"
Miomatosis uterina poco voluminosa
Hiperplasia endometrial
Piometra

VENTAJAS DE LA HISTERECTOMA VAGINAL


No deja cicatriz abdominal
Mnimo trauma abdominal
Escasa hemorragia transoperatoria
Mnima manipulacin intestinal
Menos dolor postoperatorio

CUADRO
CONTRAINDICACIONES PARA LA HISTERECTOMA VAGINAL
Impericia
tero muy voluminoso
Tumores ovricos
Endometriosis
Enfermedad plvica inflamatoria
Ciruga previa en tero, trompas y ovarios
Cncer de endometrio
Histerectoma obsttrica

80.- The anemic condition in a chronic renal failure patient is caused by defficence:

a)
b)
c)
d)

Glucose-6-phosphate dehydrogenase
Erytropoyetin
e1- antitripsine
B12 vitamin

La anemia en la IRC se caracteriza por ser normoctica y normocroma.


Puede detectarse con FG <60 ml/min, hacindose ms severa a medida que empeora la
funcin renal.
El dficit en la secrecin de EPO es el principal mecanismo patognico.
Otros factores mltiples contribuyen al desarrollo de la anemia renal. La vida media del
hemate est acortada. Algunas molculas del grupo de poliaminas, como la espermina y
espermidina, se comportan como toxinas urmicas; inhibiendo la eritropoyesis. Por otra
parte, en la IRC puede presentarse dficit de hierro y vitaminas, prdidas hemticas,
intoxicacin por aluminio y fibrosis de la mdula sea secundaria a hiperparatiroidismo.
La EPO es una glicoprotena sintetizada por las clulas intersticiales peritubulares renales
en el individuo adulto. La hipoxia estimula su secrecin, con el fin de conservar la masa de
hemates para satisfacer la demanda tisular de oxgeno. En la IRC se observa una respuesta
inapropiada. Los niveles plasmticos son anormalmente normales en relacin a los niveles de
bajos de hematocrito o hemoglobina.
La anemia, adems de la sintomatologa propia de cualquier anemia crnica, tiene
repercusiones sobre las funciones cognitivas, el sistema cardiovascular, la trombopata
urmica, la nutricin, la inmunidad y la disfuncin sexual. Tiene una significacin especial la
relacin de la anemia con la miocardiopata urmica. La anemia contribuye de forma
importante al desarrollo de hipertrofia ventricular izquierda (HVI).
En los ltimos aos, se conocen otros efectos de la EPO, ms all de los relacionados con la
eritropoyesis. La EPO es un potente regulador de la proliferacin y diferenciacin de las
clulas progenitoras endoteliales (EPCs). Las clulas madre CD34+ de la mdula sea pueden
diferenciarse en dos vas, la hematopoytica y la endotelial. La EPO, adems de estimular la
produccin de hemates, tendra la funcin de movilizar la circulacin de EPCs y actuar
sobre las EPCs maduras que expresan receptores de EPO. Estas clulas tendran una
particular relevancia en los fenmenos reparadores de neoangiognesis.

Vous aimerez peut-être aussi